Jump to content

Wikipedia:Reference desk/Science

From Wikipedia, the free encyclopedia

This is an old revision of this page, as edited by 98.26.182.245 (talk) at 05:38, 8 August 2008 (→‎Why does quantum mechanics only work on a small scale?: re). The present address (URL) is a permanent link to this revision, which may differ significantly from the current revision.

Welcome to the science section
of the Wikipedia reference desk.
Select a section:
Want a faster answer?

Main page: Help searching Wikipedia

   

How can I get my question answered?

  • Select the section of the desk that best fits the general topic of your question (see the navigation column to the right).
  • Post your question to only one section, providing a short header that gives the topic of your question.
  • Type '~~~~' (that is, four tilde characters) at the end – this signs and dates your contribution so we know who wrote what and when.
  • Don't post personal contact information – it will be removed. Any answers will be provided here.
  • Please be as specific as possible, and include all relevant context – the usefulness of answers may depend on the context.
  • Note:
    • We don't answer (and may remove) questions that require medical diagnosis or legal advice.
    • We don't answer requests for opinions, predictions or debate.
    • We don't do your homework for you, though we'll help you past the stuck point.
    • We don't conduct original research or provide a free source of ideas, but we'll help you find information you need.



How do I answer a question?

Main page: Wikipedia:Reference desk/Guidelines

  • The best answers address the question directly, and back up facts with wikilinks and links to sources. Do not edit others' comments and do not give any medical or legal advice.
See also:


August 1

Raindrop

What is the weight of a typical free falling raindrop? —Preceding unsigned comment added by 79.76.185.178 (talk) 00:57, 1 August 2008 (UTC)[reply]

According to rain, typical raindrops are approximately spherical, with a diameter of 1-2mm. Thus they have a volume of about 5×10−10–4×10−9m3, a mass of about 5×10−7–4×10−6kg, and a weight of about 5×10−6–4×10−5N. Algebraist 01:08, 1 August 2008 (UTC)[reply]
Or in more readable units (no offense Algebraist), a raindrop has a mass of about a milligram (and therefore a weight of about 10 micronewtons). —Keenan Pepper 03:35, 1 August 2008 (UTC)[reply]
Wait does a free falling raindrop have any weight? I mean it has mass..but does it have weight? As the raindrop basically no longer exists when it goes * splat * does a raindrop ever have any weight to begin with? —Preceding unsigned comment added by PvT (talkcontribs) 08:27, 1 August 2008 (UTC)[reply]
Weight is the amount of gravitational force pulling on an object. A raindrop may be in freefall falling, but that's because gravity is acting on it. --Bowlhover (talk) 08:38, 1 August 2008 (UTC)[reply]
Everything with mass in a gravitational field has weight. --98.217.8.46 (talk) 13:51, 1 August 2008 (UTC)[reply]
Bowlhover, raindrops aren't in freefall. Their terminal velocity is 9 m/s at the highest. After a few seconds, they pretty much stop accelerating at all. — DanielLC 17:09, 1 August 2008 (UTC)[reply]
Oops, I used "freefall" to mean "in a fall". --Bowlhover (talk) 18:28, 1 August 2008 (UTC)[reply]

Insect identification

What is this insect? It was in Vancouver. Sancho 07:24, 1 August 2008 (UTC)[reply]

Oh, I think it's a Firebrat. Sancho 07:36, 1 August 2008 (UTC)[reply]

Copper Sulfate as an algacide

I have some copper sulfate I purchased from Southern States Coop and I used it in my pool as an algacide. I was amazed at the quickness which it cleared the pool(overnight) with only about 2 teaspoons of CuSo4. I still have not added any shock in over two months. My pool has 14500gals of water. I have a terrible problem now with algae taking over my small (test)aquarium.I have one fish (left)an he's a Jack Dempsey, who gradually killed the other fish.1) Can I dose the aquarium with him in it? 2)If I must keep him out, for how long? 3) should I just leave him in and OVERdose it for his murder of the other fish? (just kidding) (e-mail removed as per guidelines)—Preceding unsigned comment added by 72.84.241.243 (talk) 12:17, 1 August 2008 (UTC)[reply]

You can use it - though it is pH sensitive - there are many guidlines on the web eg http://www.fishyfarmacy.com/Q&A/all_about_copper.html or search 'copper sulphate aquarium', it's also poisonous to snails. http://www.sydneycichlid.com/algae.htm says 0.3ppm 87.102.86.73 (talk) 12:41, 1 August 2008 (UTC)[reply]

Why does loop quantum gravity predict that the speed of light depends on its energy? Do all LQG theories make this prediction? Thanks, *Max* (talk) 15:44, 1 August 2008 (UTC).[reply]

I'm no expert, but the article section "LQG and string nets" mention that some researchers try to derive photons as analoges to phonons. Phonons are vibrations in a medium that propagate at the Speed of sound, which can in fact depend on frequency in certain media. (And high frequency means high energy). So by the analogue, the speed of photons could depend on their energy too. If I read the article correctly, this is an effect of their model and for instance the "preon approach" would presumably not predict it, so not all LQG theories predict this. EverGreg (talk) 22:02, 1 August 2008 (UTC)[reply]

Robert I of Parma's children

Does someone know what was the mental illness of Duke Robert I of Parma's children by his first marriage with princess Maria Pia of the Two Sicilies? His children by his second marriage are all healthy. Maybe the problem is of the Princess Maria Pia? Help me because it's important..... --84.222.154.178 (talk) 17:24, 1 August 2008 (UTC)[reply]

The article says mental retardation - that's an intellectual disability. (also read this for a list of possible causes)87.102.86.73 (talk) 17:53, 1 August 2008 (UTC)[reply]
It's worth nothing we have no way of knowing if these claims are accurate at least from the information I've seen in the articles. It could be for example that the youngest son or someone else conspired to have them declared mentally retarded for his, her or their own interest. Given that this occured in 1907, I would say it's possible. Also Infanta Maria Antonia of Portugal says two of her children have "issues" (no idea what that means although most of Maria Pia's children has "issues" as well) and one of them was deaf Nil Einne (talk) 18:14, 1 August 2008 (UTC)[reply]
Had 'issues' usually means had offspring - not health issues.87.102.86.73 (talk) 18:19, 1 August 2008 (UTC)[reply]
See Issue and Issue (legal)87.102.86.73 (talk) 18:21, 1 August 2008 (UTC)[reply]
(edit conflict) The problem could be with the union and not specifically with the woman.
This condition can be, but is not necessarily, inherited (have a genetic cause) see http://specialed.freeyellow.com/YAdaptUse.html
There is also the possibilty of bad luck.
I'll leave as message at the humanities desk to see if anyone knows more about this specifically.87.102.86.73 (talk) 18:19, 1 August 2008 (UTC)[reply]
I'm not certain that there was a modern diagnosis-I haven't seen one in reading about it; one could speculate that there was an Rh blood factor incompatibility in the first marriage (and not in the second), or a congenital toxoplasmosis infection (such as that responsible for mental retardation in the French royal pretenders), or any of a number of other possibilities. - Nunh-huh 01:24, 2 August 2008 (UTC)[reply]

Gull strength

For its size, is a gull (say, your basic Herring Gull) physically stronger than a human? --Kurt Shaped Box (talk) 19:49, 1 August 2008 (UTC)[reply]

Well I can't fly - so at a guess - yes?87.102.86.73 (talk) 20:04, 1 August 2008 (UTC)[reply]
I expect that if Kurt were shrunk to the size of a gull, he could kick its ass, (but couldn't fly) and if the gull were enlarged to Kurt's size it would be unable to support its own weight and would fall over (and also be unable to fly). See Square-cube law and "On Being the Right Size " by HaldaneEdison (talk) 20:18, 1 August 2008 (UTC)[reply]
I hope not - if Kurt were gull-sized I'd imagine that he would find its claws to be as big as kitchen knives...87.102.86.73 (talk) 21:43, 1 August 2008 (UTC)[reply]
I don't know for certain, but I suspect from observation that gulls have faster reflexes and reaction times than humans too. I don't know how effective punches and kicks would be against thick feathers either... --Kurt Shaped Box (talk) 22:09, 1 August 2008 (UTC)[reply]
Plus gulls have beaks. —Lowellian (reply) 23:51, 1 August 2008 (UTC)[reply]
Could this be adequately modeled by a fight between gull and equally sized primate, such as a small monkey? A Tamarin is approximately the same mass as a gull (at about 1 kg), and they look pretty human-like to me. Nimur (talk) 00:02, 2 August 2008 (UTC)[reply]
Due to the shorter nerve paths, Kurt's reflexes and responses to stimuli would become faster. As for the beaks, his musculature is designed to operate an organism many times larger. Read the Haldane ref and consider how things like strength and mass vary by squares and cubes as size is scaled by an order of magnitude. A human, scaled down, would be far stronger than a small primate or a bird. I personaly would pay big bucks to watch the fight on pay per view TV. Edison (talk) 04:19, 2 August 2008 (UTC)[reply]
Kurt-shaped or not, a box can't beat a gull in a fight. Not even boxing. — DanielLC 17:26, 2 August 2008 (UTC)[reply]

Am I the only one who gets unsettled?

Question restored per Wikipedia talk:Reference desk#Am I the only one who gets unsettled?

When discovering that one's pet parrot not only has sexual thoughts, but has sexual thoughts about humans, namely me? Just to look at a parrot, you wouldn't think it. I'm quite religious and this disturbs me a little. --81.79.21.14 (talk) 19:24, 30 July 2008 (UTC)[reply]

If you're asking do parrots exhibit sexual feelings towards their owners, I'd imagine the answer would be yes, in some circumstances, as parrots often form close, affectionate bonds with their owners. There are several interesting answers from this yahoo! answers post on a similar question, and this article also makes for an interesting read. 20I.170.20 (talk) 19:56, 1 August 2008 (UTC)[reply]
It seems a bit of a leap assume that thought in parrots is equivalent to thought in humans. I have no idea if they do think, but whatever thinking they do is probably pretty parrot-y. Even if the bird is becoming sexually excited, psittacine morality is likely quite different from your own. This is not to say "you're wrong," only to suggest that you may be applying standards to the parrot that apply mainly to humans.
Do parrots looking at their reflection in a mirror act as though they're seeing themselves, or another parrot? I'm thinking of the elephant self-awareness discussed here (an elephant touched a painted mark on her forehead that she could only see in a mirror; she did not touch a similar mark made with "colorless paint"). Lack of such awareness would suggest the parrot isn't all that responsible for its actions. OtherDave (talk) 20:17, 1 August 2008 (UTC)[reply]
I'll tell you what unsettles me. "If the insects had hit on a plan for driving air through their tissues instead of letting it soak in, they might well have become as large as lobsters,..." from the essay linked earlier about animal sizes. Insects the size of lobsters? Gahh. That's really unsettling. Imagine walking through the forest and seeing a mosquito the size of a small raccoon... Nimur (talk) 23:52, 1 August 2008 (UTC)[reply]
Heh, insects were that big during the Carboniferous period, due to higher oxygen levels. ;) —Lowellian (reply) 00:42, 2 August 2008 (UTC)[reply]
Imagine if a mosquito the size of a small raccoon decided that it would like to feed on your blood! Man! It would be the similarly scaled-up centipedes that I'd have a real problem with - I think I've mentioned before that I have a phobia of those particular creatures. I'd have to carry a machete with me everywhere (handguns are illegal here) - just in case I came face-to-face with a skittering 4-foot length of legs, armour-plating and malevolence... --Kurt Shaped Box (talk) 11:26, 2 August 2008 (UTC)[reply]
Male budgies will certainly attempt to copulate with their human's hands or head (if their human is their closest flockmate). It's not something I encourage. I know that some owners find it amusing - but to me, it's a bit like bestiality-lite. I certainly don't know many dog-owners who would willingly allow their pet to hump their leg to the point of climax... --Kurt Shaped Box (talk) 11:33, 2 August 2008 (UTC)[reply]
Many animals, birds included, have very simple sexual triggers, so anyone making the right gestures or sounds, or having the right smell, "turns them on". Maybe this is what happened in this case. It's similar to how baby birds decide that anyone nearby who happens to be handy is their mother. StuRat (talk) 13:22, 3 August 2008 (UTC)[reply]

Astronomy - Solar Eclipse

I was reading an article on the solar eclipse presently underway, and it discussed the rarity of solar eclipses in the universe. It explained that the only reason that we have a precise solar eclipse is that the "...moon is 100 moon diameters away from Earth and the sun is 100 sun diameters from Earth and that that is how it works."

The Solar Eclipse article adds weight to this statement of rarity by discussing how, because of tidal acceleration (the moon's orbit slowly increasing) and that the sun will slowly increase in size over this timescale (presumably because the spent nuclear fuel reduces it's mass and therefore gravitational pull?) full eclipses will no longer occur on earth in slightly less than 600 million years, then never again.

So I wonder if anyone has any comments on the likelihood of this rare little spot in time and space that we happen to be in. How did the moon form at its' present size? Has any statistical work been done on estimating the chance that a randomly selected planet-moon pair in the universe would have these properties? (I know how imprecise planet estimates are.)

I just found this all very interesting. 24.68.246.113 (talk) 20:15, 1 August 2008 (UTC)[reply]

To the best of my knowledge, the approximately "same" (~ 30 arc-minutes) angular diameter of Earth's moon and the sun we orbit is nothing more than "dumb luck", but you can probably expand a variety of Drake equation style estimates to determine likelihood. As far as I am concerned, the likelihood is empirically measured as, "100% of known inhabited planets exhibit this coincidental phenomenon." Alternatively, consider the anthropic approach, "100% of intelligent life evolved visual acuity that could not perceive or distinguish the apparent difference of angular diameter between the two largest astronomical objects visible in the sky of the planet on which it developed." Nimur (talk) 00:07, 2 August 2008 (UTC)[reply]
The chances that a solar eclipse would occur on other planets with moons is quite high. The unusual part is that our Moon and Sun have almost the same apparent diameter, which makes for a very interesting solar eclipse where the body of the Sun is blocked, but the corona is still visible. StuRat (talk) 03:52, 2 August 2008 (UTC)[reply]
Every planet has its own unique characteristics. Mercury's inclination is 7.005 degrees, and 7.005 is very nearly a whole number. Mars is covered with iron oxide and has the solar system's tallest mountain. Jupiter has 4 Gallilean moons with the very distinctive characteristic of having around the same, very high, brightness; the 5th brightest moon is much dimmer. Neptune has the only major retrograde moon, Triton.
It may not be a coincidence, though, that Earth has a large moon. Tides on the early Earth may have created isolated potholes of water rich in organic molecules. This water, when it evaporated, would concentrate these molecules to a much greater extent than was possible in the open ocean. The molecules would react to form more complex structures, which would in turn be carried into the ocean, and so repeats the process until life formed. --Bowlhover (talk) 04:38, 6 August 2008 (UTC)[reply]
Very interesting last insight there, Bowlhover. I think that if this were Yahoo Answers, I would award best answer to "...100% of known inhabited planets exhibit this coincidental phenomenon." Nice NByz (talk) 21:43, 6 August 2008 (UTC)[reply]

Sunburn

What causes the outermost layer of the skin to peel off after sunburn? JIP | Talk 21:28, 1 August 2008 (UTC)[reply]

Apoptosis: programmed cell death. In essence, the body decides that the top layer(s) of skin cells (keratinocytes) are irreversibly damaged by solar radiation and those cells are killed. Some time later, they slough off. TenOfAllTrades(talk) 21:47, 1 August 2008 (UTC)[reply]
A non-sourced photo caption in sunburn says, "the dehydration of the epidermis causes the top layer to flake off." I am going to guess that the cells, which have presumably died all around the same time as TenOfAllTrades described above, shrink, lose their flexibility, and detach from the lower layers as they dry. The combination causes them to flake off in big sheets rather than in small pieces like normal. 71.77.4.75 (talk) 22:11, 1 August 2008 (UTC)[reply]

creation of the universe

why is this universe created? —Preceding unsigned comment added by Pramod tiwari (talkcontribs) 21:28, 1 August 2008 (UTC)[reply]

Wouldn't this be better listed on the Humanities desk? Nyttend (talk) 21:33, 1 August 2008 (UTC)[reply]
Possibly.87.102.86.73 (talk) 22:09, 1 August 2008 (UTC)[reply]
I asked God to come by here and give an answer. If he's not too busy playing mmorgs, perhaps we'll get an answer soon. -- kainaw 22:03, 1 August 2008 (UTC)[reply]
(I'm not too busy)87.102.86.73 (talk) 22:09, 1 August 2008 (UTC)[reply]
There are many responses to this question but none I've heard that could be described as an final and only answer.
One response is basically summed up by the response "It must".
Alternatively you may consider that the universe is never created, if you meant 'why is it created' as being different from 'why does it exist'.
Finally I can suggest that the answer (if it exists) may be beyond human comprehension.
There are other responses, no doubt, that I hope others will be able to link to for you to read - we have numerous articles relating to this - both scientific, spiritual and religious.
I hope that some one does know - for this I leave the space below blank:87.102.86.73 (talk) 22:09, 1 August 2008 (UTC)[reply]
There are several possible answers in our article on creation myths. You may also find a number of useful links in our article on cosmogeny. TenOfAllTrades(talk) 22:14, 1 August 2008 (UTC)[reply]
You need to make the question more specific. Then again for every interpretation of your question I can think of except why god created the universe (in which case there is no generally accepted answer, and AFAIK few religions give an answer at all), the question is pretty much meaningless and there is no reason. If there where laws of metaphysics that explained the laws of physics, then you could just count them as the laws of physics and you'd be back to square one. If you were going to use infinite recursion and have meta-meta-physics, meta-meta-meta-physics, ... for ever, it just wouldn't be logically sound. — DanielLC 17:16, 2 August 2008 (UTC)[reply]
Usually the grand origin of the universe is credited to a quantum-mechanical separation of energy and anti-energy, enough of which is created to put us into a stable false vacuum state and start the Big Bang. That's at least according to every colloquium I've been to on the topic, though while interesting much of the debate has degraded into a philosophical discussion of the logical validity of arguments such as the Anthropic principle. SamuelRiv (talk) 02:32, 3 August 2008 (UTC)[reply]

Some have suggested that the Large Hadron Collider could create small black holes. If this does happen, what could this cause? 67.150.168.139 (talk) 23:20, 1 August 2008 (UTC)[reply]

Micro black hole talks about this issue. "According to the standard calculations these (micro black holes) are harmless because they would quickly decay by Hawking radiation." Someone else with more understanding in this area will have to explain what sorts of particles/products a decaying micro-black-hole would produce... 71.77.4.75 (talk) 23:31, 1 August 2008 (UTC)[reply]
The rule for quantum mechanical decays is "everything not forbidden is compulsory", i.e. any decay that doesn't violate any conservation law has a nonzero probability of happening. A free muon can decay into an electron plus neutrinos because all the conserved properties add up correctly, but a free electron can't decay into a muon plus neutrinos because that would violate the conservation of mass-energy, and it can't decay into particles lighter than itself because, for one thing, all the lighter particles have no electric charge. That's why the electron is stable and the muon isn't. A micro black hole, because of its large mass, would have the whole Standard Model particle zoo to choose from, and presumably would decay pretty quickly into some assortment of everyday particles, there being no conservation law to prevent it. If small black holes dramatically violate the approximate conservation laws, as large black holes are believed to, they might be easy to spot. Otherwise, they would behave much like any other quantum particle with the same mass, electric charge, etc, and I'm not sure how you would even tell that they were black holes. My secret hope is that particles and black holes are actually the same thing, and Standard Model decays are the same thing as evaporation by Hawking radiation. This isn't a completely kooky idea; it has been investigated by real physicists. It runs into serious theoretical difficulties, like the fact that the electron violates by many orders of magnitude, but I still hope it's somehow true. At any rate, thinking of the electrons and nucleons that make up your body as tiny stable black holes might help some people get over their fear of a tiny stable black hole destroying the Earth/Universe. -- BenRG (talk) 01:35, 2 August 2008 (UTC)[reply]
Or perhaps the fact that if the LHC could create it, then it's already been created countless times by cosmic rays in the atmosphere, which often collide at much much higher energies. —Preceding unsigned comment added by SamuelRiv (talkcontribs) 02:35, 3 August 2008 (UTC)[reply]


August 2

Body part focus of cell phone health studies

Most studies I have heard about on cell phones and health focus on brain diseases. But most people I know carry their cell phone in a trousers pocket or a purse or have it hanging on the belt most of the day; people don't put their cell phone next to their head unless they're talking on it, and most people spend more time during the day carrying a cell phone then actually talking on it. So shouldn't the torso, waist, and pelvis be more affected than the head by the radiation? If so, why are the studies focusing on the head and brain; isn't their focus misplaced? —Lowellian (reply) 00:00, 2 August 2008 (UTC)[reply]

Cell phones are usually not transmitting, especially not when they are in your pocket and "inactive" - the device may be on, but the transmit radio is almost entirely off. (They may periodically send a short pulse to the local cell tower to retain link connection). However, during conversation, the mobile device is actively transmitting (at "maximum power"). This necessarily increases the exposure to electromagnetic radiation while a telephone call is active. The assumption is that the cell phone is held near the head (ear) during conversation, thus, during peak radiation levels, so this is the time period of interest to many scientists. Nimur (talk) 00:13, 2 August 2008 (UTC)[reply]
Addendum: As we get more "internet"-ish cell phones, which use persistent data connections, the connectivity model I just described may become less accurate as a descriptor of the cell phone transmitter/radio usage; however, the historical technology trend has been a move towards lower-power radio signals with more sophisticated digital modulation to preserve signal integrity. This will almost certainly be true of any persistent data connections; if for no other reason, it's usually better for battery life, since even complex digital circuits use less power than a nasty high-power narrow-band transmitter. Nimur (talk) 00:22, 2 August 2008 (UTC)[reply]
Considering how many people keep their cell phones in their pockets or otherwise attached to the waistband of their pants/shorts while talking using hands-free sets though, studies on other parts of the body besides the head would still seem useful. —Lowellian (reply) 00:33, 2 August 2008 (UTC)[reply]
Also, you wrote "This necessarily increases the exposure to electromagnetic radiation while a telephone call is active." Another question: how much of an increase? Two-to-one? Ten-to-one? A hundred-to-one? —Lowellian (reply) 00:38, 2 August 2008 (UTC)[reply]
Well, if the transmitter is off, it generates 0 (zero) watts, and if it is on, it generates about 1 (one) watt. You could look at a specific chip used on a specific telephone to determine more precise values. Even when "off", it seems plausible that a minute amount of leakage may occur, but I'd guess this is on the order of tens or hundreds of decibels below the active transmit level. (If you aren't familiar with decibel scales, that would be a factor of hundreds of times more power). Nimur (talk) 17:13, 3 August 2008 (UTC)[reply]
Keep in mind, that while EM radiation is indeed "radiation", it is not ionizing radiation. It is not the same kind of "radiation" that one means when talking about nuclear weapons or reactors or things like that. Whether there are long-term health effects that can be traced to it or not (I suspect not, other than distraction-related injuries), one should be aware that the terminology can get slippery if one is not careful. --98.217.8.46 (talk) 00:23, 2 August 2008 (UTC)[reply]
Ionizing radiation and radiation may help clarify. Of course, though ionizing radiation is generally considered worse, it is not valid to use this classification as the sole distinction between dangerous and not-dangerous. Nimur (talk) 00:25, 2 August 2008 (UTC)[reply]
I wasn't implying that it was. But "radiation" in the common parlance is something very different than the "radiation" in cell phones, or light bulbs, for that matter. It's entirely true that light bulbs output LOTS OF RADIATION!!! but that doesn't really mean "radiation" in the sense that most people know it. When most people think of "radiation", they mean ionizing radiation—they are thinking about genetic damage, cell damage, etc., not the sorts of things one gets at the microwave level, e.g. heating. --98.217.8.46 (talk) 00:27, 2 August 2008 (UTC)[reply]
Research has been done in mammals and through observational methods in humans, see this and all of it's linked papers and this study on mammals. Nanonic (talk) 00:25, 2 August 2008 (UTC)[reply]

Also, how much radiation are wireless cards generating in comparison to cellphones? —Lowellian (reply) 00:34, 2 August 2008 (UTC)[reply]

Cellphone: 2 W
WiFi card: 0.1 W
These are the regulatory limits [1], but the devices don't operate flat-out all the time. --Heron (talk) 14:11, 2 August 2008 (UTC)[reply]

A Cell phone transmits at "maximum power" only when the cell signal is weak. The cell tower interacts with the cell phone to restrict the power what is actually needed. Thus, you are exposed to more power when you are in a weak signal area. this is why your battery runs down faster when you are in a weak signal area.If you are worried about radiation from your cell phone and it your hours or office gets a weak signal, you might want to install a femtocell. -Arch dude (talk) 16:20, 2 August 2008 (UTC)[reply]

Won't that replace the radiation from your cell phone with radiation from a base-unit? It may even increase your exposure levels. Nimur (talk) 17:15, 3 August 2008 (UTC)[reply]
That depends how close you are to the femtocell is guess. Remember the primary issue is that people tend to have phones very close to their heads/brains. While a femtocell obviously transmits with greater power, unless you are within say a metre of one (random guess) you're probably going to receive less EM radiation. Nil Einne (talk) 19:35, 6 August 2008 (UTC)[reply]

Predatory Dinos

I remember reading a book on predatory dinosaurs a long time ago, which wass mainly about theropods, but had some passing references to 2 other types of carnivorous dinos, herreravians, which I don't know how to spell, and another which I forget. Google turns up nothing. they might have undergone a name change. Does anyone have an info on these "herreravians"? Sliver Slave (talk) 02:13, 2 August 2008 (UTC)[reply]

Herrerasauridae ? They are technically theropods AFAIR. --Dr Dima (talk) 03:09, 2 August 2008 (UTC)[reply]
Predatory Dinosaurs of the World by Gregory S. Paul is the book you're thinking of. Paul invented his own classification scheme which is a bit outdated and was not widely accepted by others. Herreravia was one group that lumped together primitive types like Herrerasauridae, as Dr Dima guessed. The 'avia' part may have come from his including Protoavis as a herrerasaur--he thought they were highly convergant with birds because of this, seperate from his "Protoavia" (roughly the modern Maniraptora minus birds). I do recall he had another, even more primitive group than Herreravia (I don't have the book with me here so I can't check on the name and exact content). IIRC, it contained mainly Lagosuchians and relatives, now considered an assemblage of ancestral dinosauromorphs and not true dinosaurs (I think Bakker in Dinosaur Heresies also included lagosuchians as dinosaurs. Under modern cladistic methods, they fall outside this group because they did not descend from the common ancestor of Megalosaurus and Iguanodon--that is, they're neither Saurichians nor Ornithischians and therefore aren't true dinosaurs). Hope this helps! Dinoguy2 (talk) 00:30, 4 August 2008 (UTC)[reply]

Allylic halides and nucleophilic substitutions

How do you know when an allylic halide would rather undergo SN1 rather than SN2? Is there some sort of rule? I ask because my organic chemistry textbook just says that allylic halides can undergo both but doesn't mention specific conditions for either. On top of that, one of the exercises just claims "SN2 conditions!" with no explanation of why it's SN2-favoring conditions. --M1ss1ontomars2k4 (talk) 03:35, 2 August 2008 (UTC)[reply]

There may be something specific to sp2-hybridized carbons (in which I'd be quite interested), but ignoring that, I think the conditions favoring SN1 or SN2 mechanisms are the same as for sp3-hybridized carbons: Polar solvents, more-substituted carbons, and stearically hindered attack angles (opposite the leaving group) favor SN1, the opposite conditions favor SN2. —Keenan Pepper 04:47, 2 August 2008 (UTC)[reply]
See Neighbouring group participation which speeds up both SN1 and SN2 reactions (quite a lot)87.102.86.73 (talk)
Thanks! I actually found it in my notes already. I never remember a thing about my notes once I've taken them; it turns out that nonpolar solvents tend to favor SN1, whereas polar aprotic solvents encourage SN2. --M1ss1ontomars2k4 (talk) 04:54, 2 August 2008 (UTC)[reply]
I think it's the other way round - polar solvents should favour SN1 !87.102.86.73 (talk) 12:38, 2 August 2008 (UTC)[reply]
Yes there is something special about allylic halides assuming you mean XCH2-CH=CH2 etc (and not CH2-CH=CHX which are vinylic haldide technically)
SN1 is favoured compared to a normal alkylic halide due to stabilisation of the carbon cation CH2+-CH=CH2 by resonance
eg CH2+-CH=CH2 and CH2=CH-CH2+
Additionally SN2 can occur at the gamma carbon too eg
XCRH-CH=CH2 + Y- >>> CRH=CH-CH2Y + X-
Watch out for the above reaction when the question says SN2 conditions. —Preceding unsigned comment added by 87.102.86.73 (talk) 12:36, 2 August 2008 (UTC)[reply]
Actually Neighbouring group participation points out that allyl groups have faster SN2 reactions as well...87.102.86.73 (talk) 13:40, 2 August 2008 (UTC)[reply]

Can you explain this 'major discovery'?

What was the novelty the MIT team in fact discovered in this press release? [2]. When electricity -- whether from a photovoltaic cell, a wind turbine or any other source -- runs through the electrode, the cobalt and phosphate form a thin film on the electrode, and oxygen gas is produced. Well, I thought the production of oxygen and hydrogen from water would happen with any electrode, just using the electricity.

So, they convert surplus electricity to H2/O2 for storage. Where is the deal? --Ayacop (talk) 07:05, 2 August 2008 (UTC)[reply]

See User talk:SteveBaker#Interesting? for discussions made by one of the greatest (ex-)RD contributors on this topic. --antilivedT | C | G 07:44, 2 August 2008 (UTC)[reply]
Thanks. To summarize, splitting water with normal electrodes isn't 100 percent efficient (probably producing too much heat), so that can still be optimized which they apparently did. I hope the PR writer will never edit Wikipedia. --Ayacop (talk) 07:52, 2 August 2008 (UTC)[reply]
Confirmed. As I understand it, using two platinum electrodes is about 60% to 70% efficient [3], with the rest lost as heat. Nearly all of the loss is at the anode where O2 is made [4]. Nocera's catalyst fixes the anode problem, increasing the overall efficiency to "near 100%" (can't find the exact figure), but still requires a platinum cathode to make the H2. The next step is to find a cheap cathode material. --Heron (talk) 13:59, 2 August 2008 (UTC)[reply]

Life time of hydrogen economy

A question that bothers me for a good time, already: How long can a hydrogen economy survive, given nonzero leaking of hydrogen into the atmosphere plus nonzero leaking of hydrogen from atmosphere into space? What's the order of magnitude of lifetime if you assume 5% leakage on earth, and the normal rate into space? --Ayacop (talk) 08:03, 2 August 2008 (UTC)[reply]

I assume by hydrogen economy you mean using hydrogen as main carrier of energy. But where would that hydrogen come from in first place? From water, consuming lots of energy, leaving O2, or from hydrogen carbons, consuming lots of energy, leaving, perhaps graphite or soot? I admit I never understood the benefits. As for the leakage, I guess H2 would be oxidized pretty fast and the leakage into space would be quite that from natural water vapor. 93.132.136.138 (talk) 11:29, 2 August 2008 (UTC)[reply]
As far as I'm aware, most proposals for a hydrogen economy have it coming from seawater since we have it in abudance. 5% leakage seems rather high to me but even so, I expect it would last several million years or more. I don't think anyone seriously thinks were going to run out of seawater. The main problem is finding a way to produce the energy needed to produce said hydrogen. Nil Einne (talk) 12:31, 2 August 2008 (UTC)[reply]
A quick calculation shows that, at current world energy consumption levels, even if we assume that all the hydrogen we burn immediately vanishes into space by magic, we still have 30 million years or so before we run out. We are not going to run out of ocean any time soon. Algebraist 12:50, 2 August 2008 (UTC)[reply]

The article hydrogen economy states. "As an energy carrier, hydrogen could substitute for dwindling supplies of petroleum and provide energy independence to countries without oil resources."

How could that be? 93.132.136.138 (talk) 13:12, 2 August 2008 (UTC)[reply]

I think the principle is Nuclear+Wind+Hydro etc >>> Energy and then Energy+Water >>> Hydrogen
Thus avoiding any petroleum in the chain.
However I can't imagine Hydrogen power ships/planes/trains anytime soon - so I think the emphasis is on could87.102.86.73 (talk) 13:37, 2 August 2008 (UTC)[reply]
So the energy independence really comes from "Nuclear+Wind+Hydro etc". Why is the article falsely implying hydrogen alone could lead to energy independence? 93.132.136.138 (talk) 13:55, 2 August 2008 (UTC)[reply]
It's slightly misleading perhaps "As an energy carrier, hydrogen could substitute for dwindling supplies of petroleum and provide energy independence to countries without oil resources when coupled with alternative energy sources"
Is that better - make the change yourself if you wish.87.102.86.73 (talk) 14:12, 2 August 2008 (UTC)[reply]
It would still be subtly but massively misleading. Energy independence does not depend on the energy carrier but on the energy resources. For renewable energies, this is mostly land, and not an alternative (though a valuable supplement) for highly populated countries. 93.132.136.138 (talk) 14:49, 2 August 2008 (UTC)[reply]
In the current world, you need both an energy carrier and energy resources. While trains in particular can use electricity most other forms of transport including air, sea and trucks+cars and a lot of other stuff (e.g. farm machinery) can't for long distance at least given the current state of battery technology (fuel cells may change this although I suspect it will be a long, long time before we're using electric planes). Also I don't think it purely depends on land. A desert country for example can get a lot more out of solar then say, Scotland. Some countries have good access to geothermal, others don't. Inland countries have no access to tidal power. Wind is better in some countries then others. Hydro depends on rainfall, rivers etc. And some countries have a lot of uranium, others not much. Tropical countries can potentially produce an equivalent amount of biofuels with less land usage then temperate countries (although again this varies a lot from location to location) although of course if you are producing biofuels you may not need hydrogen as an energy carrier. Fresh water may be a limiting resource for biofuel production in some cases Nil Einne (talk) 16:46, 2 August 2008 (UTC)[reply]
I tweaked the article in an attempt to make it less ambiguous. --Heron (talk) 14:44, 2 August 2008 (UTC)[reply]
Note that hydrogen could potentially be used for energy storage as well as an energy carrier. Energy storage is particularly vital for renewable sources which (barring hydrogen and perhaps geothermal to some extent) tend to be unreliable and also can't respond to demand & may also be useful for nuclear which can only respond very slowly to demand. Currently the only energy storage I'm aware of suitable for power stations is pumped hydro. (Non-pumped hydro can of course also respond on demand and can be used to supplement other renewable sources and nuclear but isn't an energy storage system.) Energy storage may also be deemed necessary for security reasons (e.g. if a big chunk of your power is coming from solar and your solar power stations are mostly in one area that may be deemed a big security risk.) Hydrogen could also potentially be used as an energy carrier over long distance where simply high tension wires are not deemed feasible e.g. desert countries could produce hydrogen via solar power and sell it to countries with less available solar power (obviously this doesn't help energy independence), some ideas appear to be here [5]. Of course all these are simply possibilities what actually happens is anyone's guess (I personally believe the hydrogen economy idea is overated) Nil Einne (talk) 16:59, 2 August 2008 (UTC)[reply]
Pumped air and NaS batteries have also been demonstrated for energy storage at the power station scale. Dragons flight (talk) 16:52, 2 August 2008 (UTC)[reply]

Zenker's Diverticulum is well know among Gastroenterologists, who was Zenker and where did he publish his new disorder?

As a retired medico, I have encountered patients with Zenker's Diverticulum. Most embarrassing when I had to admit to one of them that I didn't know who Zenker was. That was before Wikipedia, of course! —Preceding unsigned comment added by 77.127.125.172 (talk) 12:12, 2 August 2008 (UTC)[reply]

Um now that this is after Wikipedia, did you try looking at Zenker's Diverticulum? Seems the obvious place to look... Nil Einne (talk) 12:25, 2 August 2008 (UTC)[reply]

Applied maths

Which type of engineering uses the most maths and which uses the least? I would also liek to ask the same question for physics. Clover345 (talk) 17:54, 2 August 2008 (UTC)[reply]

How do you measure maths and physics? Without knowing that, there's no way of telling who uses most. 93.132.136.138 (talk) 19:18, 2 August 2008 (UTC)[reply]
As a gross generalization someone majoring in biophysics will have probably have taken fewer math courses than someone majoring in theoretical particle physics for example. Physics pretty much entails doing math, though the complexity of the math varies from problem to problem and depends on how difficult you want to make it and what sort of answer you are looking for. If this question relates to picking a major, it is probably best to look at the major requirements are at whatever institution is relevent. If you are asking which type of physics or engineering is the "harder science" (vs. the "softer science"), I don't think you can get an answer everyone will agree on. —Preceding unsigned comment added by 71.16.107.2 (talk) 19:58, 2 August 2008 (UTC)[reply]
Probably the heaviest use of mathematics in physics is in particle physics, especially string theory (a lot of which is indistinguishable from pure mathematics). In engineering I have no idea—aerospace? The least mathematical is harder to judge because of the fuzzy boundaries of both subjects. Building Lego structures could count as engineering and juggling could count as physics. -- BenRG (talk) 20:00, 2 August 2008 (UTC)[reply]

I think this question can be answered, although I can't answer it myself. The engineering disciplines differ in how much they rely on experience and professional intuition, rather than using theory and math in practice. My gut feeling is that electrical engineers are relatively reliant on the math and civil engineers are relatively reliant in experience and intuition. I'd love to hear from others on this. ike9898 (talk) 17:18, 3 August 2008 (UTC)[reply]

Each discipline uses different styles of analysis (and accordingly, they use different types of mathematics). Electrical engineers use transform theory and complex analysis. Mechanical and structural engineers use partial differential equations. Environmental engineers use statistics and stoichiometry. Computer engineers use number theory and generalized algebra representations. You could survey a lot of engineers and ask them how often they "do math", but you are going to have to be pretty specific - do you mean number-crunching, or analysis, or theory? Nimur (talk) 17:24, 3 August 2008 (UTC)[reply]

Why didn't man evolve wings?

I thought that every life form evolved based on need. Don't get me wrong, I believe in evolution , but it seems odd that such a useful trait wasn't given to us. Instead it was given to birds, and insects (As if they need it. There's too damn many of em, already!)

But seriously, imagine how useful it would be.- Gas crisis? Solved! Road congestion? Solved! Elevator repairmen? Sadly, unemployed. Oh well, there's always a trade off. Hey, I'm Just Curious (talk) 18:18, 2 August 2008 (UTC)[reply]

Evolution doesn't work based on "need" in the sense that it can make major changes to body plans based on what might eventually be helpful. To slightly modify an example of yours, if the gas crisis is going to affect human evolution, it would favor those who can run with speed and stamina approaching that of the cars we can no longer use, not cause people to start growing wings. Even if some amazing mutation caused us to develop angel-type wings(that is, a new pair of limbs) and hollow bones, it'd probably be debilitating to the rest of the body, and therefore not very helpful(you might be interested in the story "The Fliers of Gy" by Ursula Le Guin, which is about humanoid aliens who occasionally have a wing-sprouting mutation, which they consider a deformity). 207.233.84.97 (talk) 19:37, 5 August 2008 (UTC)[reply]
If "that would be handy" was the only factor in the natural selection of animals, birds would long ago have evolved into F-15 Strike Eagles. There are costs as well: flying needs one to be light (which generally means small, with thin honeycombed bones) and strong (a winged human would have impossibly massive chest and back muscles), and (for larger animals at least) flight is very expensive (energy wise). Animals all live in a complex multivariate world - you have to survive drought and famine and flood and you have to exploit times of plenty when they come, and you have to find a niche in life that isn't occupied by something that's better at living there than you are, and doesn't make you lunch for too many other things. And (returning to the lighter-hearted tone of your question) wings are for chickens. -- Finlay McWalter | Talk 18:34, 2 August 2008 (UTC)[reply]
(EC) If you're talking about traits being 'given', you don't understand evolution. If you think life forms evolve base on 'need' you also don't understand evolution. I don't say this to put this down, simply suggest that you may want to read our articles, e.g. Introduction to evolution, Evolution & Modern evolutionary synthesis since it will help you answer this question for yourself and a lot more besides like asking 'why' something didn't evolve is pretty pointless. But to put it simply, evolution doesn't happen based on need. 'Nature' doesn't choose what traits would be best for an organism and add them because it 'know's' the organism 'needs' them (that's a design view not evolution). Instead, random/spontaenous mutations give rise to different traits. If a trait is advantageous in a organism population's current niche, then it is likely this trait will become more common in that population over time (see Adaptation). There is no guarantee a trait is going to arise, and even if it does, there is no guarantee it will be fixed in the population even if it's advantageous. Also, flight requires a lot of tradeoffs (e.g. light body) and it's unlikely to be advantageous for humans to have natural flight in any case IMHO. (Not that this matters since as I've said, traits aren't selected in any way). And BTW, flight is itself a very complicated trait and only arose after a lot of mutations and their corresponding different traits lead there. Since as I've said, there is no overall purpose (i.e. there was no consideration, we need flight and we need to do this to get there), understanding how & why flight evolved in any given organism that has flight is not so simple as it may seem. (Although it's worth understanding not every single step happened because it was advantageous, see genetic drift and that we can never know for sure why something evolved, we can only speculate on possibilities.) This is a greatly simplified view of evolution and not entirely accurate so I suggest you read the linked articles if you want to learn more. Nil Einne (talk) 19:00, 2 August 2008 (UTC)[reply]
If you read Last and First Men you'll see that we will eventually have wings, we just haven't got there yet. Dmcq (talk) 19:45, 2 August 2008 (UTC)[reply]
If we had wings we wouldn't have arms. If we had wings we'd be birds, and we'd be wondering why we don't have arms, as we have to use our beaks and feet for everything. --M1ss1ontomars2k4 (talk) 19:51, 2 August 2008 (UTC)[reply]
Those before me have posted useful links, another would be Common_misconceptions#Evolution. --Mark PEA (talk) 21:50, 2 August 2008 (UTC)[reply]
I may be wrong, but didn't Leonardo da Vinci come up with a few sketches of what an angel would look like if it had wings? I seem to recall the rib cage and muscular structure would have to be grotesque in their enormity, and you would have to spend every waking hour consuming food to generate the energy to beat them for sustained flight. --russ (talk) 22:57, 2 August 2008 (UTC)[reply]
It's not impossible for a large animal to be able to fly. Pterosaurs are quite a bit larger than humans. As for the original question, just because being able to fly would be useful doesn't mean we are guaranteed to evolve it. It would be useful to fly. It would also be useful if we were stronger, more intelligent, had eyes in the back of our heads, could absorb energy from the sun directly, had four arms instead of just two, were immune to disease, had stronger teeth, had skin as hard as iron, etc. Mutations are random. Those mutations just never came up, therefore natural selection couldn't select those traits. Of course, if you wanted those traits, there's nothing stopping us from genetically engineering them... It would be extremely difficult though. ScienceApe (talk) 23:55, 2 August 2008 (UTC)[reply]
It's not size that's important, it's weight. That article says: "Pterosaur bones were hollow and air filled, like the bones of birds." Human bones are not hollow and air filled. You would have to significant lighten the human body in order to make it feasible for it to fly under its own power, and that has it's drawbacks (it would make us weaker, for a start - we break bones often enough as it is). --Tango (talk) 02:25, 3 August 2008 (UTC)[reply]
To be more precise, it's thrust to weight ratio that's important. Some of the largest pterosaurs such as Hatzegopteryx, were heavier than humans, but their large wingspan, and power allowed them to fly. For their size, they were quite lightweight due to hollow bones. ScienceApe (talk) 15:37, 3 August 2008 (UTC)[reply]

Synthesis problem

How might this synthesis be accomplished? I'm guessing that butadiene becomes the central part of the new molecule, but I don't see how to do the addition. --M1ss1ontomars2k4 (talk) 19:41, 2 August 2008 (UTC)[reply]

What does the text say eg "from any SM and S-C" - What does that mean.?87.102.86.73 (talk) 20:10, 2 August 2008 (UTC)[reply]
The little note says "from butadiene and any Starting Material 5 Carbons or less". --20:20, 2 August 2008 (UTC)
Oh, I was thinking that if I started with cyclopentene and used a bromine radical to abstract an allylic hydrogen, I might add that way, with 1 π electron from each double bond moving inward to form the new double bond and the other π electron moving outward to form a new bond with the allylic radical. --M1ss1ontomars2k4 (talk) 19:47, 2 August 2008 (UTC)[reply]
We cannot do homework for people on the ref desk. We can help point you in the right direction though if this is indeed a homework question. You will need to extend the butadiene by forming new carbon-carbon bonds. You may need to add some more reactive functional groups to it first. Note that the end product is symmetrical, this might make things easier. --Russoc4 (talk) 19:50, 2 August 2008 (UTC)[reply]
I'm not asking you to do my homework. I've already thought about it, and several classmates have already spent a great deal of time thinking about it (or so they've led me to believe). I've been here probably just as long as you, and I don't appreciate the fact that you assumed I was trying to get others to do my homework for me. This problem is neither homework nor required; I ask because there is an exam on Monday and this showed up in the professor's review handout (which has no solutions, just questions), and I'm assuming that we are expected to know how to do it, when I can't find anything in my notes or the text that seems to help with extending the butadiene or usefully adding functional groups.
The closest I've come so far is this: first hydroxylize the double bonds (monohydroxylation with BH3), use PCC without water to get to the aldehyde, convert cyclopentadiene into an organocuprate, then mix them up. Get rid of the carbonyl oxygen with whatever's handy, and you end up with something similar to the product but lacking a double bond. Besides the obvious problem of not making the desired molecule, this synthesis is far longer than our professor usually expects of us. Also, this is a class, not real life, so we're not expected to use BH3 as that was featured in another class. --M1ss1ontomars2k4 (talk) 20:19, 2 August 2008 (UTC)[reply]
I don't know if it is relevant to this problem, but don't forget the possibility of designing a reaction so that the rings of the molecule close up on their own. 71.16.107.2 (talk) 20:08, 2 August 2008 (UTC)[reply]
(convert to di-epoxide, react with 2x (3-metallo-cyclopenta-1-enyl) grignard, convert to 1,2 dioxide to an alkene...? There must be a better method)87.102.86.73 (talk) 20:15, 2 August 2008 (UTC)[reply]
As I've said elsewhere, this is a class, not real life, so while that may be a viable method of synthesis, converting alkenes to epoxides and using Grignards on them is probably not part of the solution. Pretty much the only tools we have are carbonyls and delocalized pi systems. --M1ss1ontomars2k4 (talk) 20:25, 2 August 2008 (UTC)[reply]
It says use any starting materials 5 or less C chains. What's wrong with Grinards? Is this what we've come up with so far, because it's all I can think of. I can't quite figure out how to get the double bond back without disrupting the cyclopentenes. And you aren't allowed to use reactions you've previously learned?? --Russoc4 (talk) 20:31, 2 August 2008 (UTC)[reply]
Update: is the last step here possible? --Russoc4 (talk) 20:46, 2 August 2008 (UTC)[reply]
Well I'm certainly allowed to use reactions I've previously learned, but all previous test questions I've done so far for this course haven't involved as much usage of "old stuff" as we've been using here, and this is from a review guide thing for the chapter on delocalized π systems. Plus, the teacher specifically mentioned that stuff from our previous course would not be tested in this course (except for basic things like ozonolysis, hyrdogenation of double and triple bonds, and oxidation with PCC and/or chromate) and that our syntheses should use stuff from this course. That's why I'm hesitant to use other stuff. As for the first image, that was indeed my thoughts, but like we've said, it doesn't quite work. The other reaction you've suggested can afford the desired product if ??? is PCC or chromate followed by Wolff-Kishner reduction, I think. Like I said though, this question is supposed to be somehow related to delocalized π systems... --21:24, 2 August 2008 (UTC)
P.S. Those are nice drawings; I would have done the same but 1. ChemSketch isn't cooperating with Wine and 2. I haven't really figured out how to make a nice reaction sequence like that; the arrows are always too long, too short, or not straight. --M1ss1ontomars2k4 (talk) 21:24, 2 August 2008 (UTC)[reply]
(IF I was teaching I'd assume that epoxidation and grignard reactions were basic things)
Have you done pericyclic reactions yet - I can't help wondering if there is a 'clever' way to do this using a cycloaddition followed by rearrangement - but nothing I can think of makes any sense...87.102.86.73 (talk) 21:39, 2 August 2008 (UTC)[reply]
Oh, Grignards are quite often used in this course, just not in this particular chapter. So they're less likely to be involved in the solution, but it's certainly possible. Epoxidation has never even been mentioned in this course, so that's most likely out of the picture... We have indeed done pericyclic reactions (specifically, electrocyclic reactions and cycloadditions) and that stuff is also in the chapter that this problem came from. That was my first though too, given the context of the problem, but I couldn't see any way to do it either. --M1ss1ontomars2k4 (talk) 22:06, 2 August 2008 (UTC)[reply]
Mmmh if you split the molecule down the middle you might get (C5H7)-CH2-CHO (you could dimerise this with a few steps..) Know any good ring contraction reactions (6 to 5) ???87.102.86.73 (talk) 22:27, 2 August 2008 (UTC)[reply]
ChemSketch can be tough to use. There is a toolbar button for reaction arrows. Anyway, you say it probably focuses on pi systems. The final product has no delocalized pi bonds, so going from a system with them to a system without them is going to take a considerable amount of energy. The only thing I can think of right now that comes close would be a pseudo-Freidel-Crafts reaction, but I'm not sure how to work it in here.--Russoc4 (talk) 22:30, 2 August 2008 (UTC)[reply]
Well, I tried to incorporate a Diels-Alder using butadiene, but that gets you to a 6 carbon chain. It would be quite the task to add only 8 carbons when the rings contain 5. Here's how I did it. Though all the transformations are theoretically possible, I definitely don't recommend using it. --Russoc4 (talk) 23:07, 2 August 2008 (UTC)[reply]
Ozonolysis of cyclohexene above will easily get you the six carbon chain. If you could cleave cyclobutene, you would get the four carbon chain that we got earlier, and then you could carry on. The only thing is that to get cyclobutene from butadiene is not a favored reaction, and would be pointless when you could just turn the double bonds of butadiene into aldehydes. --Russoc4 (talk) 23:34, 2 August 2008 (UTC)[reply]
According to our article on Grignards, Grignards also attack other electrophiles, including alkyl halides. So I think it might be possible for the allylic Grignard in the upper right corner of your image to make a carbon-carbon bond at those beta positions with bromines. My professor's hint (I emailed him with my thoughts and your idea about how to get the double bond in there) was that cyclopentene Grignard is correct; "Identify where the butadiene is and figure out what that needs to be if you are reacting it with grignards." I take this to mean that 1,4-dibromo-2-butene should be made from 1,3-butadiene, because that would certainly create the desired molecule; the creation of 1,4-dibromo-2-butene from 1,3-butadiene is actually given in the chapter. You mix the butadiene with bromine gas in carbon tetrachloride at 20 degrees C; unfortunately this only gives 46% of the desired dibromobutene. However, this seems far simpler than any of the other syntheses we've come up with, despite it not having much to do with pretty much anything in the chapter. Unfortunately, I don't think Grignards actually react with alkyl halides, contrary to what the article says, as otherwise they would react with themselves during preparation of the reagent. It looks like an organocuprate reagent has to be used instead. --M1ss1ontomars2k4 (talk) 23:55, 2 August 2008 (UTC)[reply]
Indeed. Making 1,4-dibromo-2-butene from butadiene would require a 1,4-addition to a diene. I was trying to avoid diene reactions because it could undergo 1,2-addition as well. The reaction conditions would have to be just right, and as you say, the yield is only 46%. I do not think that Grignards can attack alykl halides. This is why I would turn butadiene into the double-ended aldehyde. But, if you choose not to go this route, and would rather aim for 1,4-addition of bromine, I can tell you after some double checking that Gilman reagents do react with alkyl halides in the way you mention. --Russoc4 (talk) 00:12, 3 August 2008 (UTC)[reply]

human cloning

in human cloning, can it be done to make someone at desired age or start from a baby? will their life style be similar to ours, lifespan,health? should all the parts be stitched into them? when life is put into them, will they feel totally blank or will they remember anything, atleast can their brain adapt soon to its ambience? —Preceding unsigned comment added by Shiniga (talkcontribs) 19:44, 2 August 2008 (UTC)[reply]

At the moment it's nearly impossible to clone humans. Once we do, however, it would have to start from a baby, and it would hopefully have similar lifespan and health. They won't remember anything, as we don't know how to transfer memories. Of course, it's possible that in the future we'll come up completely new techniques to transfer memories, start clones off as adults (or make them mature faster), or whatever. It's hard to say for sure, as we're currently stuck in the present, and we can't even solve the problem of getting an embryo to develop properly. --M1ss1ontomars2k4 (talk) 19:50, 2 August 2008 (UTC)[reply]
Another thing to take into account is that clones will develop differently from the "original" because it is impossible to place them in indentical environments and chance events in development will result in them having non-identical physical and mental features. Clones should theoretically have about the same level of similarity that identical twins have with each other (they are not completely identical mentally or even physically). One problem with clones produced from mature cells is that they really don't have the same DNA. Cells modify their DNA epigenetically in order to regulate the activity of genes and to differentiate themselves from each other. If you use the DNA of a skin cell to create a clone, it already has been modifed. The electric shock is intended to strip some of these modifications off, but this is kind of like using an axe to cut a slice of bread, it's not very clean or precise and some modifications may stick around or the DNA could be damaged. Geneticists suspect this can lead to some down the road health problems, so clones produced from already differentiated cells may have reduced health compared to "originals".
If a clone is produced like a normal baby, then it has a "clean memory slate" and should develop and behave like any non-cloned human will. Thus a clone will be fully "human" in that it has an independent conciousness and will and is a totally different human being than the original, the same way identical twins are separate people. A clone of Gandhi would not be Gandhi: raised in a normal environment the clone is no more likely to become a great political spiritual leader than any other human on earth.
Memory and consiousness is created by the physical layout of the neurons in the brain, the structure and other stuff going on inside of the neurons themselves, how they connect to each other, and how they signal to one another. To reproduce someone's memory and personality (this is speculation by the way), identical neurons would have to be stuck in the same places, connected the same way, and induced to signal each other just like the originals. Brain development relies in part on chance events that even two genetically identical brains in identical conditions will not develop exactly the same. It would probably be easier to try to brainwash someone to believe that they had a past experience than to try to reproduce a memory by controlling the brain's development or creating a copy of a brain. 71.16.107.2 (talk) 20:37, 2 August 2008 (UTC)[reply]
I wouldn't say a clone of Ghandi is no more likely than the average person to behave like Ghandi. At least a portion of our personality comes from our genetics. And, if he were told he was a clone of Ghandi, this might make him more likely to behave like Ghandi than to run a convenience store. StuRat (talk) 02:41, 3 August 2008 (UTC)[reply]

Instead of thinking of cloning as "copying", think of it as making a baby that happens to have the same DNA as someone already alive, instead of new DNA from a mother and father. --98.217.8.46 (talk) 23:43, 2 August 2008 (UTC)[reply]

Well your age has nothing to do with your physical size. Afterall, humans define age relative to childbirth, thus you have to realize the biases that exist in discussing the cloning issue. A successful living human clone would think of hisself or herself the same way you think of your self. The clone would start out as a single cell, just like you or I would. Some humans (not clones) already feel memories of past lives and I've met someone who truely thinks that they were in WW2 in a past life. So if a clone had similar memories, are they more valid? Sentriclecub (talk) 02:54, 3 August 2008 (UTC)[reply]
Any clone would start from age zero, just like anyone else. However, there are ways to alter the rate at which the clone would grow. Dolly the cloned sheep, for example, may have been inadvertantly made to age more rapidly by the cloning process. StuRat (talk) 16:51, 3 August 2008 (UTC)[reply]

Pigs on the wing

How difficult would it be for humans to genetically engineer a pig with wings and the ability to actually fly? —Preceding unsigned comment added by 84.66.29.34 (talk) 22:26, 2 August 2008 (UTC)[reply]

Very - a pig is far too heavy to fly under its own power without massive changes. You would probably be better off starting with a bird and engineering it to look like a pig. --Tango (talk) 23:27, 2 August 2008 (UTC)[reply]
It would involve more than one gene. The pig, if you removed enough genes and substituted them with the minimum amount of alleles to accomplish the goal, then you may have removed so many genes that the pig lost some of its familiar phenotype (e.g. can it lose its curly tail and still be called a pig?) Speciation is a wreckless art, and one of the few agreed upon criteria is the count of chromosomes. That said, its probably possible, but I dont know how you can "program" the pig to know how to fly? I mean, what if we gave him the genes to grow some spectacular 30 feet wings made out of a super-light biologically feasible polymer? It might need a "mother" to teach the pig how to use it. i.e. like how those otters can crack shells on a rock--its learned from other otters. But I'd say its definitely possible. Look how far nature has came? A tyrannosaurus from a bacteria? or a flying pig from a regular pig given the earth is populated with a flood of nobel laureates in biomedical engineering (and hopefully this one some day ;) I think its less of a challenge to make enough genetic modifications to pig to accomplish your goal, than what nature has already done (t-rex from a prokaryotic bacteria, one or two incremental gene modifications at each offspring level). The main bottle neck is that we don't have computers than can predict the tertiary and quaternary structure of proteins given a sequence of amino acids. (the sequence of amino acids is the unit of a gene) Sentriclecub (talk) 02:34, 3 August 2008 (UTC)[reply]
The answer I've heard used for this type of question is another question: "How many corners can you add to a circle to make look like a square?" None - it is impossible. Once you add a corner (or four of them) to a circle, it is no longer a circle. So, it can never be a circle that looks like a square. Similarly, if you replace a pig's front legs with wings, it will not longer be a pig, so it will never be a pig that flies. If you want a serious discussion on this topic, try something like, "How many (or what specific) body parts in a human can be replaced with artificial parts without making the human non-human (ie: a robot)?" -- kainaw 03:36, 3 August 2008 (UTC)[reply]
Genetic engineering is only about changing the pig's DNA so that a 1-celled embryo with the modified genes grows up into a fully developed organism with his stipulated phenotype. Replacing tissue doesn't count since it would then answer a question that asked can I surgically put wings onto a pig? Sentriclecub (talk) 03:57, 3 August 2008 (UTC)[reply]
After carefully reviewing the semantic structure of the OP’s question, I believe that it can be logically deduced that s/he is talking about STARTING with a pig. This is a good example of the sorites paradox by the way. --S.dedalus (talk) 05:07, 3 August 2008 (UTC)[reply]
Yes, I was thinking about achieving the end result of a pig that looks like a pig - but with wings on its back. Which would mean giving it six limbs. --84.71.59.70 (talk) 22:16, 3 August 2008 (UTC)[reply]
While we are speaking hypothetically, would a different technique (not genetic modification) be easier to accomplish the task at hand? (I.E. structural reformative surgery, or something drastically different?) Nimur (talk) 17:27, 3 August 2008 (UTC)[reply]
Just attach it to an airplane. We can even put a chip in the brain of the pig and allow it to pilot the airplane with its mind. If you're only counting adding biological tissue, I'm not sure if we can do it now, but it's still definitely much easier than genetic modification. It's much easier to graft wings onto a pig than to modify its genes so they interact with each other in in such a way that perfectly functional wings sprout out of the pig's back. — DanielLC 03:23, 4 August 2008 (UTC)[reply]

Risk of contracting MRSA

Would it be possible to contract MRSA by picking your nose, and eating it? --russ (talk) 22:46, 2 August 2008 (UTC)[reply]

Nostrils are one site for MRSA infection. If you picked your nose, and what you picked has MRSA in it, you are likely to already have contracted the disease.--Fangz (talk) 23:02, 2 August 2008 (UTC)[reply]
If you picked someone else's nose and ate it, maybe. You can't contract a disease from yourself, since it would require you to already have it. --Tango (talk) 23:28, 2 August 2008 (UTC)[reply]
You wouldn't be contracting a disease from yourself though. You would be allowing a way for the bacteria that always live on your skin to enter your body and initiate the infection. --Russoc4 (talk) 00:00, 3 August 2008 (UTC)[reply]
If it's already in your nose, I think you're probably already infected. Bacteria can get on the skin from all kinds of places, but getting into the nose would probably require you to either inhale it, or for it to come from the inside, either way, you're already infected. In fact, even without that, eating it isn't that likely to cause an infection - stomach acid is pretty good at killing bacteria. You're more likely to get infected due to scratching the inside of your nose and letting the bacteria in that way. --Tango (talk) 02:21, 3 August 2008 (UTC)[reply]
The mrsa is more likely to infect you by crossing the thin wet linings of epithelial tissue in your nose, than risk the harsh environment of the stomache. Indeed scratching your nose might invoke tissue damage and weakness. 99x more likely to contract it in the local proximity of your nasal cavity than your mouth/throat/stomach/intestines. Sentriclecub (talk) 02:49, 3 August 2008 (UTC)[reply]

I'm not satisfied with the answers so far. In fact, staphylococci in those parts of your nose you can get at with fingers aren't something to be worried about, it's perfectly normal. That's why a good doctor, when checking for systemic MRSA, will make smear tests of your nose and your throat because if you have something in your sinus it will show in a throat smear.

Now, to the bogey eating part, it's not the worst you can do, as long as you have stomach acid and a good digestion. Actually, it's even clever, because your immune system will see the digested parts and learn from that. The worst would be trying to clean your nose the usual way, i.e., by pressuring your nose while holding a kerchief (and possibly closing the nose opening, that's the really worst), because this way you push the MRSA into your paranasal sinus where it will thrive and go into the other sinuses. --Ayacop (talk) 07:38, 3 August 2008 (UTC)[reply]

You lost me at a good doctor, when checking for systemic MRSA, will make smear tests of your nose and your throat because if you have something in your sinus it will show in a throat smear.
In my unscientific opinion, MRSA is just a fancy staph infection. The question could be answered if the OP inserted the word staph infection for MRSA. The only difference being that MRSA is linked to Stevens-Johnson syndrome which is a nightmare. The question can be answered with a yes and more specifically by explaining the two infection risks (specifically the picking with fingernail, and the ingestion of contaminated mucus). I think this question has been satisfactorily resolved, and exceeded the expectations set forth by the OP. Could we do better? Yes! I think the OP has been treated very diligently and will use this service again. I hope he enjoys his experience with wikipedia and will think highly of its hard-working editors. Sentriclecub (talk) 11:57, 3 August 2008 (UTC)[reply]

The answers thus far have failed to distinguish between infection and colonization. If MRSA is residing in your nose without causing disease, you're colonized, not infected. The usual reason for culturing someone's nose is not to determine if he's infected, but rather whether he's colonized. If a colonization is found, it can be treated with topical antibiotics to reduce the risk that MRSA will be transmitted to someone else, who (especially in hospital settings where this is done) may be less well immunologically equipped to deal with the organism and would develop an actual infection. - Nunh-huh 15:48, 3 August 2008 (UTC)[reply]


August 3

Amount of energy needed to reach the speed of light: Infinite. Amount of atoms to destroy the sun: 1?

Essentially the faster an object with mass is moving, the more kinetic energy it has. In order to reach the speed of light, it would require an infinite amount of energy. The amount of energy it would take to blow apart the sun, is a finite amount. With that in mind, would it be fair to say that you could (in theory) destroy the sun with just one hydrogen atom, as long as you throw it at an extremely high fraction of the speed of light? ScienceApe (talk) 00:04, 3 August 2008 (UTC)[reply]

You seem to be invoking relativity and quantum mechanics at the same time. I'm no expert in either field, but my initial guess would be that any energy high enough to destroy the sun would surely be enough to split your hydrogen atom before it even gets there. --Russoc4 (talk) 00:20, 3 August 2008 (UTC)[reply]
Not exactly; the hydrogen atom can be accelerated to any speed below c at any non-zero rate. This implies that any non-zero force would due. --Bowlhover (talk) 02:31, 3 August 2008 (UTC)[reply]
But velocity is relative. So it doesn't matter which one is moving, as long as it's relative. So alternatively the sun could be moving at that velocity and collide with a single hydrogen atom that's at rest. Also I'm not sure how a Hydrogen atom would split just because it's moving at high velocity in a vacuum. This is kind of a hypothetical question, we are ignoring how to accelerate the atom to such ludicrous velocities. ScienceApe (talk) 00:27, 3 August 2008 (UTC)[reply]
Very well, then you are just using the hydrogen atom as an example of something extremely small. Alas, I don't have any answer for this one. --Russoc4 (talk) 01:40, 3 August 2008 (UTC)[reply]
You could certainly give the hydrogen atom enough energy to destroy the sun, but that doesn't mean it would actually do so. In order to destroy something you have to transfer the energy to it. I expect such a fast atom would just go straight through the sun and keep going, rather than destroy it. --Tango (talk) 02:19, 3 August 2008 (UTC)[reply]
There is a similar situation with bullets. A faster, higher energy bullet can actually do less damage, by going straight through a body and essentially taking a "core sample", while a slower, lower energy bullet bounces around inside and rips the body apart. StuRat (talk) 02:31, 3 August 2008 (UTC)[reply]
From my very limited knowledge of quantum physics, once you impute that much energy into the small mass, and it approaches the speed of light, the matter takes on wave-like properties. That's what advanced general relativity is about. Sentriclecub (talk) 02:46, 3 August 2008 (UTC)[reply]
Sorry Sentriclecub, but that's completely untrue. The more energy something has, the less quantum-mechanically it behaves, and general relativity has nothing to do with the quantum mechanical world at this point. Special relativity can be applied in something called quantum field theory, which is the likely source of confusion. SamuelRiv (talk) 02:53, 3 August 2008 (UTC)[reply]
Admittedly, my knowledge of quantum physics is just from 6 TTC lectures. They said that quantum mechanics and general relativity has some overlap, but they also have parts the other can't explain. The most advanced stuff of general relativity is trying to expand it into something called unified field theory which explains everything that quantum mechanics seperately covers. So that's why I called it "advanced general relativity" because it is something I'm very interested in. Einstein spent 30 years of his life unsuccessfully working on relativity trying to explain natural phenomenon using the propositions he laid out in his theories of special and general relativity. I have skimmed his book Relativity: The special and general theory with commentary by Richard Geroch" and follow updates to other scientists continuing his search. I believe I may have misunderstood the application of at appreciable fractions of the speed of light, matter breaks down and subsumes purely wavelike properties and maybe not relevant to this guy's question. Again my answer reflects my limited knowledge which was hopefully prefaced well enough enough in my first answer, that the reader knows I'm just providing an educated guess. I don't quite get what you say with the more energy something has, the less quantum-mechanically it behaves could you explain? I thought it was related to the more mass something has? Sentriclecub (talk) 03:18, 3 August 2008 (UTC)[reply]
, so energy is linearly proportional to mass. —Preceding unsigned comment added by SamuelRiv (talkcontribs) 03:28, 3 August 2008 (UTC)[reply]


Thanks SamuelRiv, I used the wrong word also. Unified field theory is about expanding relativity into the field of electromagnetism, not quantum mechanics. I dont remember the term for unifying relativity with quantum physics. One last question, given the nature of mass-energy equivalence-- does a high energy object (with high mass) behave equally less quantum-mechanically than a particle with the same amount of energy (but exorbitantly less mass)? Thanks and I appreciate your response. Sentriclecub (talk) 03:43, 3 August 2008 (UTC)[reply]
The gravitational binding energy of the sun is around 1041 joules; I'll take that as an estimate of the energy needed to blow apart the sun (although stars are perfectly capable of blowing themselves apart without external input, so maybe I shouldn't). The mass of a hydrogen atom is about 10−10 joules. So we're talking a gamma factor of 1051. The sun's diameter is about 5 light seconds, so the hydrogen atom will pass through in about 10−50 seconds of its proper time. Compare this to the Planck time of 10−43 seconds. The orders of magnitude involved here make me think that you need a theory of quantum gravity to predict what will happen. My best guess is that you'd destabilize the vacuum, destroying the Sun, the Earth, and everything else. -- BenRG (talk) 11:54, 3 August 2008 (UTC)[reply]
Thanks for the followup, I was lurking awaiting followup so that I could go to sleep peacefully. Sentriclecub (talk) 12:00, 3 August 2008 (UTC)[reply]
You only need enough energy to disturb the balance between the sun's gravity and the radiation pressure to the point of causing an explosion - I have no idea how much energy that is, but it's presumably significantly less than the total gravitational binding energy. Either way, what does "destabilize the vacuum" mean? It sounds like something out of science fiction! --Tango (talk) 20:53, 3 August 2008 (UTC)[reply]
See False vacuum#Vacuum metastability event. Algebraist 21:01, 3 August 2008 (UTC)[reply]
A valid criticism of this hypothetical question is: we have never observed such high energy particles; while current theory may not prohibit them outright, it's conceivable that some upper bound on energy per particle will prevent this situation. Take a look at Tevatron - it's no easy task to get particles to have high energy, and you're suggesting that one may result from a random astrophysical process (or something). I conclude that it's not very likely, whether it's possible or impossible! Nimur (talk) 17:33, 3 August 2008 (UTC)[reply]
There's nothing in physics that prohibits it. There is no upper bound on energy per particle. According to special relativity, you can approach the speed of light as much as you like. You'll never hit it, but there's nothing stopping an object with mass from achieving a very high fraction of c. I never suggested that one may result from a random astrophysical process (or something) at all. Strictly a hypothetical question. ScienceApe (talk) 23:46, 3 August 2008 (UTC)[reply]
Such a high energy particle will strike a strong headwind of cosmic microwave background photons. These will appear to be ultrahard gamma rays in the frame of the proton and will cause massive reactions, as well as slowing it right down. This sort of effect Greisen-Zatsepin-Kuzmin limit puts an upper bound on the energy of cosmic rays. Graeme Bartlett (talk) 00:29, 4 August 2008 (UTC)[reply]
In this question we really aren't taking into account such things, nor am I asking about cosmic rays. Also I'm not sure how photons would significantly slow down a such a fast moving particle. The particle would have tremendous more momentum than a bullet. And photons don't slow down bullets very well. The Greisen-Zatsepin-Kuzmin limit applies to cosmic radiation, but not a particle that I'm referring to. Ultra-high-energy cosmic ray actually exceed that limit, but even they only have the energy of a baseball moving at 60 mph. But again, this is only referring to cosmic radiation, which isn't what I'm asking about. ScienceApe (talk) 02:13, 4 August 2008 (UTC)[reply]
Cosmic radiation just means really fast particles originating from space, like the one you suggested. — DanielLC 03:05, 4 August 2008 (UTC)[reply]
Originating from space, or from naturally occurring sources. In that case, it would be impossible for the particle to achieve such high velocities for the reasons mentioned above. ScienceApe (talk) 04:54, 4 August 2008 (UTC)[reply]

I don't understand Schrödinger's cat

I was trying to find out if there were any evidence of randomness in the universe and was led to Schrödinger's cat. I think I understand that the universe is random (right?)...but I don't understand why the cat would be both alive and dead. Louis Waweru  Talk  02:23, 3 August 2008 (UTC)[reply]

You're question is essentially one of microcausality. watch parts 1 through 5 and the answer is essentially that there isn't randomness, because the causal relationships between past and present are tied together and this must hold true at the quantum level. Also the guy who is prominently mentioned in I believe lecture 4, is a youtube professor from stanford.edu (the person who was invited to a physics meetup and got terrified by hawking's conclusion) you'll like it, since you are probably the right demographic that the video is aimed for. A pretty decent ability to comprehend concepts of higher science stuff. Sentriclecub (talk) 02:41, 3 August 2008 (UTC)[reply]
Professor suskind was his name. Sentriclecub (talk) 02:43, 3 August 2008 (UTC)[reply]
Thanks, just finished downloading this, going to watch it tonight =) Louis Waweru  Talk  02:11, 4 August 2008 (UTC)[reply]
Heh, everyone in that video is a complete ass, especially Hawking and Krauss. I'm not sure to what you're referring by saying there isn't randomness, because in our universe there is and has to be. It's simply fact. Now having not finished the videos, I think you might be referring to certain multiverse ideas to resolve the apparent paradox of randomness, but it really doesn't matter, because our universe is independent of that. SamuelRiv (talk) 03:38, 3 August 2008 (UTC) Edit: okay, it's about the information paradox, which has nothing to do with quantum randomness. SamuelRiv (talk) 04:11, 3 August 2008 (UTC)[reply]
Sorry Sentriclecub, but you have misunderstood what was being said in the video. It has nothing to do with quantum mechanics, but rather the information paradox applied to a general relativity problem. Causality must hold true, but this doesn't mean true randomness can't exist and particles can't appear out of nowhere. And Hawking is a pathetic self-promoter, as are many of the others in the video, and while the paradox is unresolved, it is mostly because we still do not fully understand what happens when black holes are created (a classical/statistical/QFT/GR problem) nor the nature of the singularity (a quantum gravity problem), either of which will likely resolve the paradox, for better or worse. SamuelRiv (talk) 04:07, 3 August 2008 (UTC)[reply]
Eh, it's more of an allegory than a principle of physics. There's too much "observation" that the cat has with itself (just through normal particle interactions that occur with any dense object) that would collapse any such uncertainty. If you'd like to understand the fundamental idea of randomness, maybe quantum mechanics is a good start. Let us know if you have questions, as it's a difficult subject.
Another type of randomness relies on a property called "sensitivity to initial conditions", and is formally known as chaos theory. Unlike quantum mechanics, it can be observed all the time in macroscopic systems (it's why we can't make accurate weather forecasts more than a day or two in advance). However, since such systems are all essentially Newtonian, it isn't true randomness. Rather, the data needed to predict what is going on is extremely difficult to obtain, and even a small error in the data is enough to catastrophically throw off predictions.
The final step in the randomness ladder is quantum chaos, about which I know nothing. Happy hunting! SamuelRiv (talk) 02:47, 3 August 2008 (UTC)[reply]
You speak of observation that the cat has with itself, but that's controversial, right? Doesn't your statement assume Objective collapse theory or something like that? Wouldn't others say that if the box was magically sealed from the outside world, the system would remain in superposition until the seal was broken? (Copenhagen interpretation?) --Allen (talk) 04:52, 3 August 2008 (UTC)[reply]
A common misunderstanding - which it sounds like you might be having from the statement in the original post - is that the cat is both alive and dead. The wavefunction does not express this, but rather it expresses our knowledge of the state of the cat. The cat is definitely one or the other, but we don't know until we look (observation). Therefore the wavefunction expresses our uncertainty (or lack of knowledge) about the state of the cat. After an hour, all we can say is that there is a 50% chance of it being alive. When we look inside (make an observation) we collapse the wavefunction to one state or the other. PhySusie (talk) 10:49, 3 August 2008 (UTC)[reply]
Again, you call it a misunderstanding... but are there not interpretations both ways? Are there not physicists who interpret the wavefunction as expressing something real about the system? --Allen (talk) 12:53, 3 August 2008 (UTC)[reply]
Thinking about this more, though my line of argument is similar (i.e. "hey, isn't what you're saying controversial?"), it doesn't seem to me that you're saying the same thing as SamuelRiv. It seems to me that SamuelRiv is saying that the wavefunction collapses without us looking from the outside -- the system observes itself. You seem to be saying that the wavefunction does not collapse until we open the box, but then you're also saying that the wavefunction doesn't imply a real superposition of the state of the cat. --Allen (talk) 13:02, 3 August 2008 (UTC)[reply]
PhySusie, you were right about how I was viewing it. Amcbride, what I think you're saying is what I was hung up on. Superposition is still really confusing, but I think what my friend told me about waveform collapse helps me make sense of it. Now I look at it as the collapsed waveform being what is reality...are you saying that the other possibilities are real as well? Louis Waweru  Talk  02:11, 4 August 2008 (UTC)[reply]
I'm no physicist so I don't know... I'm just saying it seems to me that a lot of physicists think that the other possibilities are real. So I'm not saying PhySusie is wrong; I'm just saying I think she's expressing one side of a controversy among physicists, rather than a consensus view. I could be wrong about that too, but so far no one has said so. --Allen (talk) 06:44, 4 August 2008 (UTC)[reply]
Schrödinger's cat was a silly parenthetical comment by Schrödinger in a much longer paper which took on a life of its own. The point Schrödinger was trying to make is that the rules of quantum mechanics imply that you can't confine it to the microscopic systems it was invented to describe; you can set up thought experiments where quantum behavior (modeled by Schrödinger's equation) is transferred from a microscopic system to a familiar macroscopic one, like a cat. In fact this always happens, since we can't measure properties of microscopic systems except by amplifying them. But in reality cats aren't fuzzy the way quantum systems are. (They're fuzzy in a different way—aren't you, widdle kootchie wootchums? Now get in the box.) People often get this backwards, thinking that Schrödinger was saying that the cat is, in reality, a superposition of alive and dead. He was really saying that the lack of alive-dead combinations in the real world means that Schrödinger's equation can't be the whole story. At any rate, he was wrong; cats are constantly interacting with their environment, and there's no way to prevent or shield this because of the second law of thermodynamics. When you model this interaction instead of treating the cat as an isolated system, you find that Schrödinger's equation does predict classical behavior for thermodynamic systems (see quantum decoherence). It might have been better all around if the cat had ended up on the cutting room floor. The whole cat paragraph could have been deleted from the paper without affecting the thrust of the argument at all, since it just reiterates the point of the previous paragraph in a sensationalistic way. And the point was wrong anyway. -- BenRG (talk) 13:35, 3 August 2008 (UTC)[reply]
When you talk about taking the cat as an isolated system, do you mean that as shorthand for taking the whole box as an isolated system? I don't see how any interpretation of Schroedinger's cat involves assuming that the cat itself is isolated from the rest of the box, though I'm sure I could be wrong. --Allen (talk) 18:25, 3 August 2008 (UTC)[reply]

Thank you for the help everyone! Honestly, I had to get a friend to interpret what you guys were saying for me. I think I understand now though...he had to explain things to me first though, like waveforms, the idea of superposition, waveform collapse, what happens when something is measured...the idea of an observer. All fascinating stuff, amazing... But I think in the end the point was that the Copenhagen interpretation is one of many interpretations and the cat is just meant to get people thinking. Louis Waweru  Talk  02:12, 4 August 2008 (UTC)[reply]

A bit near the question above...

Doesn't the cat live a normal lifespan? Assuming that if lives its entire life without ever being poisoned/shot, then wouldn't it eventually die? And, is the 50-50 chance that it lives applied to every second? Or nano second? Or microsecond? Eventually, if you narrow it down to smaller and smaller measurements of time (like Planck seconds), then wouldn't the trigger go off even without 2-3 seconds passing by with no incident? And, assuming that this argument is valid, wouldn't that ensure that the cat would die almost instantaneously after the experiment began?

Thanks! ECH3LON 03:38, 3 August 2008 (UTC)[reply]

Eventually, the cat would die, but that isn't the point of the experiment. It isn't a 50/50 chance of death. It is just that if you concern yourself with wondering about the hidden cat's state of being, you are uncertain. If you instead check the control of the trigger (decaying atoms in his example), there is no uncertainty. -- kainaw 03:48, 3 August 2008 (UTC)[reply]
First off, it's an allegory, not a real experiment, so it would indeed live a normal lifespan. Now how quickly the cat lives or dies depends on how you formulate the problem. Let's say that each ray of radiation emitted by a bit of radioactive material in the box is enough to kill to the cat instantly. So let's say it has a 1/2 chance of emitting one ray each second, then extrapolate the math from there, so the cat only has a 1/4 chance of surviving after two seconds, etc. Extrapolating to a singular point doesn't work, I'm afraid, with applied math, so you can never count an infinite number of probabilities in an infinitely small time to get a 100% chance of death. This is because in the physical world, radiation is well-defined, not some point-particle emitted in point-time as this exercise describes. SamuelRiv (talk) 03:56, 3 August 2008 (UTC)[reply]
The probability that the cat is alive at any given point in time is where h is the half-life of the trigger (for instance, a radioactive sample) and t is the time that has passed since you set the trigger.71.77.4.75 (talk) 04:10, 3 August 2008 (UTC)[reply]
(2 ec's) "Doesn't the cat live a normal lifespan? Assuming that if lives its entire life without ever being poisoned/shot, then wouldn't it eventually die?" Yes, but Schrodinger's cat is a thought experiment about when a system ceases being a quantum superposition of states (both alive and dead) and becomes one state (either alive or dead). Whether the cat eventually dies is irrelevant; the experiment involves its state at the end of the experiment.
"And, is the 50-50 chance that it lives applied to every second?" No, radioactive decay, the process used in the experiment, has a certain possibility of occuring within a certain time. Half-life is the time it takes half a sample to decay, so it's also the time in which any given atom/other particle has a 50% chance of decaying. When the time is shortened, the possibility of decay also reduces; it doesn't remain at 50%.
Of course, the experiment does not necessarily have to use radioactive decay. Any quantum mechanical process that introduces randomness would work.
SamuelRiv: Surely extrapolating to a single point would give a 0% possiblity of emitting one ray? Also, the thought experiment doesn't involve a point-particle emitted in point-time; real particles can be used too, and Schrodinger used 1 hour as his time. --Bowlhover (talk) 04:13, 3 August 2008 (UTC)[reply]
You might like looking at quantum immortality. Jkasd 04:45, 3 August 2008 (UTC)[reply]
You might also like looking at Quantum Zeno effect. I you kept monitoring the cat so quickly it would never die :) Dmcq (talk) 08:48, 3 August 2008 (UTC)[reply]

Thanks for all your answers!! ECH3LON 22:49, 3 August 2008 (UTC)[reply]

Double alkylation of carboxylic acid derivatives

I have in my notes that acid anhydrides, unlike the more reactive alkanoyl halides and the less reactive esters, do not undergo double alkylation in the presence of a Grignard reagent. Does anyone know 1. if this is correct and 2. why this is? Instinctively, I would assume that, since you're making a ketone with the first alkylation, the Grignard could alkylate the ketone to yield a tertiary (or possibly secondary, if you started with a methanoic acid anhydride) alcohol, because you can do that with regular ketones that you didn't prepare from an anhydride. --M1ss1ontomars2k4 (talk) 03:54, 3 August 2008 (UTC)[reply]

Your analysis is reasonable - however look at this page http://designer-drugs.com/pte/12.162.180.114/dcd/chemistry/p2p.grignard.ac2o.html see the diagram (intermediate product) note that acetate has not left as a leaving group, in fact the acetocy group is capable of chelating the Mg in a six membered ring here..

eg from acetic anhydride and 'RMgBr' the intermediate could be:

        Me
         \
          C=O
         /   \
        O     MgBr
         \   /
          C-O
         RMe

Breaking the C-O bond to make ketone plus Mg(Br)(acetate) won't be very favoured (cf acid chloride) —Preceding unsigned comment added by 87.102.5.5 (talk) 13:46, 3 August 2008 (UTC)[reply]

However if you boiled this I would expect it to decompose..
Look at this patent from the preparation of a methyl ketone http://www.wipo.int/pctdb/en/wo.jsp?IA=WO2000076948&WO=2000076948&DISPLAY=CLAIMS note that the grignard is added to excess acetic anhydride.
I'd guess that if you added excess grignard to acetic anhydride with heat you'd get full reaction.
The results quoted are often under specific conditions and may not be an apples to apples comparison. There's often a big difference say if you do a grignard reaction at -78C or 25C - the text books often don't make that clear..87.102.5.5 (talk) 13:05, 3 August 2008 (UTC)[reply]

Extremes of earth's atmosphere

I remember an article, very probably on wikipedia, telling there was a time in earth's history when the atmosphere had oxygen levels up to 35%. Another article (or the same?) told about the solar system flying through a denser region of space, resulting the earth in catching hydrogen from space and burning the atmosphere. Now I can't find the articles again nor anything similar. Was I dreaming? 93.132.182.221 (talk) 10:00, 3 August 2008 (UTC)[reply]

As to the first question, oxygen catastrophe has a diagram showing a max at just about 35 per cent if I'm not mistaken. You won't get much more, anyway, because of spontaneous fires breaking out (already at ~25% IIRC).
As to the Earth's previous path, I think that's quite hypothetical. Also today, the solar system sits in a dust bubble, too, from a previous nearby supernova we moved to, so something similar as you say should happen now, too, but doesn't, probably because of the magnetic field shielding us. --Ayacop (talk) 10:13, 3 August 2008 (UTC)[reply]
Thanks for the link. The article I remember was different, telling for example that those giant flying insects of the past where only possible for the high oxygen concentrations to keep the muscles going when air could only enter through the trachea. I couldn't have mad up stuff like that in a dream, could I? 93.132.182.221 (talk) 14:11, 3 August 2008 (UTC)[reply]
Perhaps Carboniferous is what you are thinking of? Try the section "Rocks and coal" in particular. I've never heard anything about "burning the atmosphere" with hydrogen though. That seems unlikely to me. Dragons flight (talk) 16:28, 3 August 2008 (UTC)[reply]
Yes, that is the article!!! Thanks a lot! 93.132.182.221 (talk) 19:00, 3 August 2008 (UTC)[reply]
As for the second article, was it related to the late heavy bombardment or the hydrogen-and-helium atmosphere that escaped around this time? --Bowlhover (talk) 20:26, 3 August 2008 (UTC)[reply]

Baked limestone

Can you tell me if mixing baked limestone with water to produce heat would cause irritation to human skin?. —Preceding unsigned comment added by 217.42.237.185 (talk) 13:03, 3 August 2008 (UTC)[reply]

Yes - baking limestone enough makes quicklime which reacts with water giving heat. —Preceding unsigned comment added by 87.102.5.5 (talk) 13:08, 3 August 2008 (UTC)[reply]
Assuming there is enough lime, the water will reach a pH of about 12. Such very basic solutions are bad for skin. Given enough time, this would go beyond irriation and cause real damage. ike9898 (talk) 17:08, 3 August 2008 (UTC)[reply]

Shining Metals dug from the mountains

I was wondering if there are any metals that actually shine enough for us to be able to see it in the dark?

Let's say you're in a dig-site DEEP INSIDE the mountains, and you find yourselves standing in a tunnel looking down in a really dark hole with a huge gold-ore or silver-ore or any metal (but especially and preferably more commonly known metals!) And the only reason you can see it is because it gives out a slight glimmer that you can spot in the dark..

I'm not really thinking that there is any metal that shines and glimmers that strongly by itself, but what if you stood with a torch in your hand and maybe the light from the torch reflected in the metal so you could see it glimmer, despite the darkness surrounding it down in the hole. Is that possible ?

I have read that gold is kind of shiny compared to other metals and that the "grayness" that most metals has to them when you dig them out isn't as present in gold. because the electrons and/or atoms are more different and varied or somesuch. (whatever - that part about why and how isn't that interesting to me at this point) But i don't know if gold ore could ever reflect such light, or if any other metal could..


You familiar with MITHRIL for example? The fictional type of metal created by Tolkien. It was supposed to glimmer quite strongly, and have the beauty of silver... I know that no real metal shines that strongly but I was looking for one that maybe shines a little as I said, like maybe reflecting lights from torches etc. in the dark..

Krikkert7 (talk) 15:28, 3 August 2008 (UTC)[reply]

Shining is the reflection of light. Darkness implies lack of or no light. Therefore, no metal can shine in complete darkness and it's unlikely for it to shine brightly with little light. —CycloneNimrodTalk? 15:51, 3 August 2008 (UTC)[reply]
They didn't say it would shine in darkness, but only that it would reflect a flashlight beam. Yes, gold does that, because it doesn't oxidize when exposed to air, as most metals do. However, you're not very likely to see an exposed gold vein in a cave wall. Other formations, like cave crystals, are far more likely to be sources of such a reflection. Then, to add a shiver to your spine, that reflection you see in the dark might be the eyes of a black panther hidden in the cave, stalking you. StuRat (talk) 16:37, 3 August 2008 (UTC)[reply]
What will you do? If you want draw your weapon and continue investigating the cave, turn to page 175. If you think the risks are too great and wish to leave this dark place, turn to 247.87.102.5.5 (talk) 17:04, 3 August 2008 (UTC)[reply]
If you do not wish to be eaten by a grue, put the book down and walk away.91.143.188.103 (talk) 21:15, 3 August 2008 (UTC)[reply]
If it is a real metal, it should abide by conservation of energy, and cannot shine brighter than the torch or flashlight you used to illuminate it. Assuming the only process is reflection, the "shininess" will necessarily be dimmer than the source flashlight. (This does not account for phosphorescence or chemiluminescence, which are photo-chemical effects - in these cases, the glow may be dimmer, although maybe longer in duration, than the source light. In some exotic cases it seems conceivable that you might even catalyze some chemical change which could release previously stored energy in the form of light, but I know of no such mineral which does this when exposed to a flashlight). Nimur (talk) 17:40, 3 August 2008 (UTC)[reply]
Well, Radium shines in the dark, and, as far as I know, some of it's salts shine brighter. I can see an Unholy Avenger of Oppenheimer that shines in an eerily green colour and causes disease and deformity... --Stephan Schulz (talk) 03:06, 4 August 2008 (UTC)[reply]
Radium in nature is found in vanishingly small concentrations. It does indeed have a deadly glow when concentrated, but it seems very doubtful that it would ever be concentrated enough to glow in the ore as found in a mine, with milligrams of Radium mixed with tons of ore. Edison (talk) 03:24, 4 August 2008 (UTC)[reply]

Why do we care about faces?

Why do we care about how our partners look like? As far as they are healthy, why does it matter to have a harmonic face, a hooked nose or big ears? Evolutionarily fit children can be perfectly born from ugly parents, can´t they? —Preceding unsigned comment added by Mr.K. (talkcontribs) 19:04, 3 August 2008 (UTC)[reply]

There are lots of theories around human mate selection, you may want to read them for a better idea. But it appears there are some obvious errors in your thinking. The first obvious flaw is that you seem to have forgotten there is no way we can know how healthy someone (or is going to be, remember for greatest probability of successful children we tend to want the mate to be around and in good health for 20 years+) is by magic, we have to use something. Bearing in mind human mate selection largely evolved before doctors or anything remotely similar, going on what people look like seems a decent choice even if far from perfect. Also, it's not just how healthy someone is, for example a female wants her mate to stick around to look after the children. And of course you can be as healthy as you want, it doesn't help if you can't hunt/raise children/successful give birth. Then of course it's all very well successfully raising children but if no one wants to mate with your children because they look ugly you're not doing very well in evolutionary terms so even if looking a certain way isn't actually a predictor of reproductive fitness in most others ways, the fact that it affects mate choice may be enough in itself (see Sexual selection). Finally remember that evolution is all about probabilities. Even if 'ugly' parents can have 'evolutionarily fit' (this is poor usage of terminology BTW) children, if 'beatiful' parents have a greater chance of having 'evolutionarily fit' children then the beatiful parents win in evolutionary terms Nil Einne (talk) 19:22, 3 August 2008 (UTC)[reply]
Beauty can also indicate age and injuries, both recent and past. For humans, living conditions affect one's looks drastically, and poverty causes problems such as disease, aggression, crime, and low intelligence. --Bowlhover (talk) 20:04, 3 August 2008 (UTC)[reply]
There are theories (and studies) which claim that facial symmetry can be indicators of mutations, pathogen load, and other indicators of mate quality. Here is one paper discussing issue of attractive faces. Also of interest is research into what fluctuating asymmetry can be an indicator of.
As Nil Einne does point out, this desire for attractive faces (which typically are the most symmetrical) is more complicated than just wanting the most attractive mate possible, since as the mate increases in attractiveness, competition will increase in suitors.
But in response to Nil Einne's claims about health not helping to raise children/hunt/etc, the idea isn't that facial symmetry needs to predict those behaviors (although they might). Those factors can be partially determined by actual interaction (ask fellow tribe members how reliable X is at hunting, or ask fellow tribe members how nurturing Y is with children). If beautiful faces are predictors of factors like pathogen load (especially during development) or number of detrimental mutations, then they would provide information that actual interaction may not (or additional information to supplement information received through normal channels). So, you wouldn't need to look at the face of someone to tell whether they have parasites on them right now; you'd look at the face of someone to tell how resistant they are to parasites (again, assuming the data supports the link between facial symmetry and parasite resistance, which it may well not).--droptone (talk) 12:46, 4 August 2008 (UTC)[reply]
You may have misunderstood what I was saying. I was not saying that health isn't important in helping determine other factors, like how good someone is at raising kids, but there are additional factors besides health that matter and that some of these factors may be partially predicted by how someone looks. Remember that someone can be as healthy as they want in every other respect, but if they are infertile, well they aren't a good mate (at least if you're male, things get a little complicated if your female where the person who helps you raise your kids does not have to be the person who impregnates you). Remember that looking at beauty from a more complete sense, it includes things like body type (which I haven't discussed much but I'm not sure if Mr. K really wants to restrict this discussion to facial type since he talks about ugliness and beauty in a generic sense for part of his question, in any case it doesn't make sense to ignore other factors completely when we talk about beauty). I agree it's a combination of interaction and what the person looks (and possibly smells) like as you point out. But of course, just because it may seem likely that it would be better to determine something in another way, doesn't mean beauty didn't play a role as well in telling us. Remember also that in some cases, e.g. if you kidnap people from another tribe, your interaction experience may be limited. And that a big part of our evolution happened before we had the level of communication we have now. In other words, it's almost definitely the case that beauty played a multifaceted role in telling primative humans which mate to choose and that it's beyond a simple predictor of current and future health (future health obviously includes issues such as resistance). For example, I'm sure I've read of a study which seem to show woman prefer a more feminine looking face (or body?) most of the time except when she was most fetile in which case she prefers a more masculine looking face (or body?). The idea here is that the female wants the masculine looking person to impregnate her, because his (particularly male) children may be the most successful in evolutionary terms but wants the femine looking person to care for her children, because he's the most likely to stick around. (This is an idea which comes up a lot in menstrual cycle linked differences.) Nil Einne (talk) 14:01, 4 August 2008 (UTC)[reply]

Mechanical pencil uses chuck?

What do you call the type of mechanical pencil that uses a chuck (engineering)-type mechanism?--Sonjaaa (talk) 21:06, 3 August 2008 (UTC)[reply]

Don't they all use a chuck? --Russoc4 (talk) 02:38, 4 August 2008 (UTC)[reply]
The types of mechanical pencils are listed in the article you cite under Mechanical_pencil#Types. Presumably you haven't found the answer you want there though? I would say the clutch type best describes a chuck type mechanism since the 'claws' clamp the graphite piece in place. Jdrewitt (talk) 07:35, 4 August 2008 (UTC)[reply]

I read the article, but I'm not an engineer, and I don't understand which type of mechanical pencil uses a chuck, or if like Russo says, they all use a chuck? It's the article about chuck that's not clear to me.--Sonjaaa (talk) 13:32, 4 August 2008 (UTC)[reply]

i seem to remember a term 'propelling pencil' from my younger days. This now redirects to mechanical pencil. So there you are!
That's not a very good article. Any kind of mechanical pencil I've ever taken apart uses the "clutch" type (which I would term a collet) to hold the lead in place. That's the little three-claw thing with a sleeve overtop, pushed down by a spring. The sleeve pushes the claws together and they hold the lead in place. A Faber-Castell or Staedtler drafting pencil would be chuck-type, gravity-based - you push the button at the top and let the lead drop out as far as you want. (I used those back when drafting was done by hand). The Pentel (and is that an article in need of cleanup!) mechanical pencils I use every day would possibly qualify as ratchet-based, you click-click-click and the lead advances a little bit at a time, if you hold down, you can push the lead all the way back in. They also use a collet/chuck mechanism to hold the lead in place. The screw-based and twist-based designs mentioned in the article undoubtedly exist, but I can't say I've ever seen one. I have probably 25 mechanical pencils in my house (yeah I know, geeky) and all of them use a collet (or chuck if you want) to hold the lead. Franamax (talk) 11:20, 5 August 2008 (UTC)[reply]
You can buy brightly-coloured disposable twist-end mechanical pencils for not much more than wooden pencils, so they're fairly ubiquitous to me :) No need for sharpeners, messy shavings, loose rubbers (since they have one on the end), and no worries about losing or lending them (unlike with reusable ones). Now, if only there were somewhere to send the used up pencils to be refilled and recycled. So yeah, look for brightly coloured plastic pencils if you want to see screw types. 79.66.32.107 (talk) 15:24, 6 August 2008 (UTC)[reply]

Plant defences and herbivory in the African savanna

I was watching a special on TV about life in the African savanna, where several animals, namely elephants, deer, giraffes, and monkeys, were going about their normal business maneuvering the limbs and branches of fairly thorny trees, and eating the leaves off very thorny twigs. The giraffes and deer seemed to eat the leaves with the greatest calm, not minding the thorns at all. The monkeys were jumping about just as easily, and the elephants were knocked down trees with no complaints. If theses thorns evolved in part to deter herbivory, then how come these animals eat off these trees so easily. And how come they can move about on the branches with no difficulty. I wondered if it was because they mightve had thicker skin, so the spines dont penetrate, or in the case of eating they might actually pull the leaves off with their tongue, and then chew. Any thoughts? —Preceding unsigned comment added by 189.34.103.78 (talk) 21:35, 3 August 2008 (UTC)[reply]

You're too absolute. Maybe the damage from those animals isn't enough to give the evolutional pressure that's necessary for really different solutions holding away the animals completely. Maybe the plant evolved in that direction but died out because the fruits weren't eaten any longer by apes, and so seeds couldn't be transported to new places? Maybe such evolved plants died out because the fleas that can be found on that giraffes no longer ate those ticks sitting on the plant and sucking. Nature is much more complex than you think, even if you don't dive into the chemistry of it. --Ayacop (talk) 09:59, 4 August 2008 (UTC)[reply]
You may also be interested in Plant defence against herbivory. Shyamal (talk) 10:03, 4 August 2008 (UTC)[reply]
Ah, that has it all, thanks. What's missing is that seeing an animal eating a plant doesn't always mean it's on its daily diet. It has been observed that apes use specific plants when they're ill or for digestion. The cost/benefit ratio of evolving against such minimal threats would be forbidding. --Ayacop (talk) 10:11, 4 August 2008 (UTC)[reply]

On the subject of human cloning (not sure if this is the right desk for this)

I assume that a human created by humans using cloning technology would be sapient and have his/her own separate consciousness, barring any unforseen flaws in the system, but would these 'manufactured humans' have a soul, as such? --84.71.59.70 (talk) 22:12, 3 August 2008 (UTC)[reply]

Do you? Seriously, there is no scientific basis for the existence of a soul, hence it is not a scientific question. I would suggest you are better off asking a religious or spiritual adviser. Baring that, you might try the humanities reference desk. Dragons flight (talk) 22:32, 3 August 2008 (UTC)[reply]
You might find the novel Never Let Me Go an interesting read related to this question. But no, it is not scientific. But if I were to speculate: a "soul", as it is usually understood, is something that surrounds human life. It does not seem to be something that is dependent on the combination of female and male gametes. So it shouldn't be a problem. A clone would still be a human in every way. --98.217.8.46 (talk) 23:05, 3 August 2008 (UTC)[reply]
Many experiments have been done to try and detect the soul but they have failed, e.g. killing a person in a closed room with just a small hole for the soul to get out and watching the hole, or putting people who are dying on scales to see if the scales move when they die. I can't find references to such experiments in Wikipedia, I'd have thought they were notable. We'd have to a) clone a person and b) be able to detect a soul to give a really definitive answer to your question. Until then I think we have to be content with what is said in the Philosophical zombie article. Dmcq (talk) 08:25, 4 August 2008 (UTC)[reply]
The experiments involving weghing dying patients on scales were carried out by Duncan MacDougall in 1907. By comparing results from human patients to results from a control group of dogs (which were assumed not to have souls) he concluded that the human soul had an average weight of 21 grams. MacDougall's methods and objectivity were called into question and his results have never been replicated.
In answer to the original question - if you assume that humans have souls, why should a clone not have its own soul too ? Identical twins are "natural" clones, with identical DNA, but each twin presumably has their own individual soul. Gandalf61 (talk) 09:17, 4 August 2008 (UTC)[reply]
This position, while common, is not universal. I don't know what the Yoruba think about cloning. Algebraist 11:58, 4 August 2008 (UTC)[reply]
That's very interesting about the Yoruba, thanks. I've looked it up and Frederick II, Holy Roman Emperor was the one who did the earlier experiment but it's not described on wiki, there's a small reference in http://www.youngscientists.co.uk/node/106 Dmcq (talk) 20:19, 4 August 2008 (UTC)[reply]

Anyone here a doctor?

Hi there. I am writing an essay regarding about the writing practices of physicians today. I am conducting an interview for physicians as part of the research of the essay. Here is the interview. It consists of 12 simple questions:

  1. Approximately what percentage of an average work week do you spend on written tasks?
  2. Please discuss how written communication skills are important (or not) to your field.
  3. What sorts of documents you write on a daily/weekly basis? What about longer term?
  4. Who are your primary audiences (internal? external? any international? If so, where?)?
  5. Do you write collaborative documents (as part of a team)? If so, please describe a common collaborative situation (number of people, length of document, situation, etc.).
  6. What software tools do you use to prepare documents in your field (Word? Excel? PowerPoint? Other?)? In other words, what (non-technical) general software should professionals in your field be prepared to use for written or oral communication?
  7. What online tools are important to your field (examples include virtual meeting tools like WebEx, collaborative tools like Google docs or wikis, communication tools like blogs, online research tools, and professional networking sites)?
  8. Do you see increasing use of multimedia tools (audio/video, for example) by professionals in your field? If so, please describe the use, tools, or applications.
  9. Do you see the internet impacting information sharing and/or communication in your field in other ways not yet described? If so, please explain.
  10. What is most difficult about the documents and/or presentations you write for your job?
  11. What top three essential writing skills should new graduates have when entering this field?
  12. Describe how especially written communication practices changed in your field in the past 5 years.

I would really appreciate if a physician could answer these questions. I will need them ASAP. Thanks in advance! NittanyLion08 (talk) 23:32, 3 August 2008 (UTC)[reply]

Keep in mind that it will be hard to verify whether any responses are from a legitimate physician, so if you value the data results very highly, you might want to use a non-internet-based survey method. Nimur (talk) 00:04, 4 August 2008 (UTC)[reply]
I'm waiting for the "I'm not a doctor, but I play one on Wikipedia..." answers. -- kainaw 01:12, 4 August 2008 (UTC)[reply]
I doubt you will receive more than a few responses, NittanyLion, but I may be surprised. --Bowlhover (talk) 02:26, 4 August 2008 (UTC)[reply]
I see a lot of professional surveys created by marketing / sociology researchers who have no concept of what a professional's time is worth. I remember one academic sending out a dollar with each survey questionnaire, and getting some of them (if at all) returned with one question answered, with comments that a dollar isn't sufficient compensation for even that one question. Good luck finding a physician with sufficient patience for your questions. Physicians are busy people, and their time is valuable. The questions you pose are too time-consuming to answer, unless you pay them for their time. It might help if you streamlined it, providing meaningful multiple choice responses. ~Amatulić (talk) 22:45, 4 August 2008 (UTC)[reply]

Heparin lock

I've been undergoing minor testing in the hospital for several days, during which I've had a Heparin Lock in my arm. I've gone looking online for information about it, which has led me to conclude that it's potentially notable, even though there's no article about it. Does anyone know much about such a device? The medical staff have answered all my medical questions, so I'm not asking for medical advice for my specific situation: I'm simply more interested in information about the device in general. I've learned somewhat about it from the websites I've looked at, but as a distinctly non-medical person, I don't understand too much about it, except as it applies to my testing. Could someone explain the basics of this device: what it was originally meant for, what it's used for (in general today), information like that? Nyttend (talk) 01:32, 4 August 2008 (UTC)[reply]

A heplock provides access to the bloodstream for easy injection or sampling. Heparin is an anticoagulant, and is used to prevent the port from clotting. The alternatives would be to poke you with a new needle each time or to require you to have an IV drip to keep the line open. DMacks (talk) 05:17, 4 August 2008 (UTC)[reply]
It looks like there's a passing mention in port (medical), but it could use some beefing up. It'd be dandy if someone with suitable expertise could have a look, and perhaps create appropriate redirects. TenOfAllTrades(talk) 13:46, 4 August 2008 (UTC)[reply]

five kindoms

tigar which filam? —Preceding unsigned comment added by 121.247.108.75 (talk) 02:43, 4 August 2008 (UTC)[reply]

Your question is so full of typos and missing grammar that it is not possible to answer. It appears that you are asking which Five Kingdoms movie has the Tiger - but that is only a guess. Try spelling actual words and using complete grammar and someone will probably be able to give you an answer. -- kainaw 02:53, 4 August 2008 (UTC)[reply]
You can learn which phylum the tiger is in by looking at: Tiger. Dragons flight (talk) 02:55, 4 August 2008 (UTC)[reply]
Look at the box on the right. You don't need to read the article. --Bowlhover (talk) 03:42, 4 August 2008 (UTC)[reply]
BTW, the Five kingdoms model, while still taught in some schools is generally considered a rather poor model nowadays. One of the most obvious flaws is it puts Archaea and Bacteria together into Monera. Nil Einne (talk) 13:27, 4 August 2008 (UTC)[reply]

chordee

can steroid help cure chordee? —Preceding unsigned comment added by 116.71.150.79 (talk) 06:52, 4 August 2008 (UTC)[reply]

If you look at chordee it says treatment is best carried out in infancy by surgical means. If there was a treatment involving steroids I guess that would be the preferred treatment. Richard Avery (talk) 07:38, 4 August 2008 (UTC)[reply]

A lot of quasi-related questions

Sorry, I know these questions are going to be odd, but I couldn't find the answers on the net, so please bear with me. First, when we submerge something in water, there will be an upward force (buoyancy) acting on the object, right. The magnitude of this force (as per the wikipedia article) will equal the weight of the water displace, but why, intuitively, should this be? Secondly, an object that floats at the surface will also float at the bottom of the ocean, but how is this possible considering that there's the weight of the entire ocean above it that it has to contend against? The only explanation for this that I can think of is that water is a liquid, and so because the object can pass through it, it's weight is meaningless. But if this was the case, why do people keep mentioning the weight of the atmosphere above us as important (i.e. puts stress on objects, causes pressure, etc.) And why does air pressure drop as we travel farther from the surface of the earth is the weight of the atmosphere above us isn't significant? Finally, is bouyance responsible for convection (why hot air rises)? I would think so (hot air expands, thus less dense), but I wasn't sure. Thanks in advance! —Preceding unsigned comment added by 76.68.246.7 (talk) 09:16, 4 August 2008 (UTC)[reply]

By pushing an object into water, you displace water, and raise the water level slightly, so you're work done to push the object into the water, is stored as gravitational potential energy. The water level, acted on by gravity, is pulled down, and this forces the object up, providing that it is lighter than the water it displaces, this is the origin of buoyancy. Secondly yes, the water above an object does provide pressure in the same way air above us does, but it does not pin us down does it? gravity does that, and in the same respect, objects underwater are not 'pinned down' by the weight of the water above them.An object that floats placed at a greater depth will in fact experience a greater up thrust, as the higher pressure means the weight of the water it is displacing is greater. Philc 0780 11:28, 4 August 2008 (UTC)
A correction - The weight of the water above a submerged object is realized as pressure, which acts equally in all directions on an object, thus canceling out, so the only relevant forces are the weight of the object and the buoyant force. Also, the thrust does not increase with depth because the weight of displaced water does not increase with depth, because water is incompressible. To answer the other questions, you can say air pressure drops as you climb in altitude for the same reason water pressure drops as you approach the surface - just less weight piled on top - but air pressure also drops because its weight decreases with height due to the declining pull of Earth's gravity (relevant well above the troposphere). The weight of the atmosphere is certainly significant as atmospheric pressure. Buoyancy, in a sense, is responsible for convection, but since convection is a well-defined statistical mechanical effect, it might be more accurate to say that convection is responsible for buoyancy (but I'm not sure on this). SamuelRiv (talk) 13:54, 4 August 2008 (UTC)[reply]
I think this is splitting hairs, but yes, I would prefer to say convection causes buoyancy. Buoyancy is the simple approximation of the net fluid motion effects acting as a single equivalent upward force; it also assumes the object is sufficiently small that there is not a significant pressure gradient between its top and bottom parts, etc. The more rigorous treatment yields a similar effect via much more horrible integral equations. Nimur (talk) 15:50, 4 August 2008 (UTC)[reply]
Thanks a lot guys, really. But sorry, just one last thing, why is the pressure increase caused by the weight of water/air above us distributed evenly?
Because in the case of a liquid or gas, the molecules are free to move in all directions. They are under pressure (the weight of all the molecules above them) so they will push whichever way they can go to escape the pressure. If they can get a little more room by pushing your body inwards a little bit, that's what they'll do. They're pushing back just as hard on the molecules above them, beside them, and below them. (That is the highly unscientific explanation, but it works for me :) Franamax (talk) 02:14, 5 August 2008 (UTC)[reply]

I was washing dishes...

And I noticed two things happen that I can't fully explain. First, if I were to plunge a glass into the water (with the open side facing it), the inside won't get wet. I'm not sure why this, and my only explanation is that the air pressure inside the glass is stopping it from entering (and if the water level were to rise, the pressure would increase and stop it eventually). Second, if I quickely remove the drain, a huge bubble will erupt. I not more or less why, but the details seem to be lacking. The water enters some sort of secondary container, and the air needs to escape. But what gives the column of air so much force? Thanks. —Preceding unsigned comment added by 76.68.246.7 (talk) 09:21, 4 August 2008 (UTC)[reply]

In response to the first, the air in the glass takes up the volume of the glass. We'd normally say that the glass is empty but it's not. It's full of air. The air would rise out of the glass if there were a hole in the bottom (now the top) of the glass. This would of course cause the glass to fill with water. You can demonstrate this with a straw though it may be hard to see. Plug one end of the straw with your finger and plunge it into water the same way as your glass into the sink. Then remove your finger. The water will push the air up the straw and out the top. Dismas|(talk) 10:05, 4 August 2008 (UTC)[reply]
To answer your second question, the air that is in the drain beneath the plug has to go somewhere when the water in the sink tries to go down. It can either follow the pipe or it can attempt to rise through the water. Water will attempt to fall because gravity is pulling it into the drain, forcing the air to attempt to rise because it will be displaced by the falling water. This the bubble you see. The reason the air doesn't just move down the drain is because the water falling into the sink does not form a solid uniform wall to prevent the air from escaping. EagleFalconn (talk) 15:38, 4 August 2008 (UTC)[reply]
This is more or less what I thought, but when I say huge, I really mean it i.e. water went everywhere.
If you release it quickly, there is a large amount of air that is rising. It has a bit of momentum when it reaches the water's surface and therefore lots of water gets spritzed everywhere. Dismas|(talk) 20:29, 4 August 2008 (UTC)[reply]
If you ever go SCUBA diving, you can take this experience to a whole new level. Just under the surface the water level in the up-ended glass will be near the lip. As you descend and pressure builds the water will force it's way further up into the glass. When you hit 33 feet under, the water will be exactly (by volume) half-way up the glass. At 66 feet it will be 3/4 and at 99feet it would be at 7/8. As you ascend, it will slowly "empty" in exactly the same manner. ---J.S (T/C/WRE) 23:44, 4 August 2008 (UTC)[reply]

Drinking water vs. pouring it over your head to cool down

If you had a small amount of room-temperature water, say 30 cL, on a hot, dry summer day (say 28 °C), would it help cool you down more/faster to drink the water, or to pour it over your head? What if it was hot or cold water? Or what if it was a humid day? --58.37.187.141 (talk) 10:11, 4 August 2008 (UTC)[reply]

I don't expect drinking the water will actually help to cool you down directly much, but it will help recover a small amount of the lost fluids from sweating. It doesn't matter much what the temperature of the water is (just don't burn your mouth!). Pouring it over you, provided the water is at a lower temperature then your surface temperature is obviously going to cool you down somewhat but it won't help you with the water lose due to sweating and given the small amount, isn't going to help that much. Obviously the colder the water, the more effective. However particularly if you have fairly cold water, I would suggest pouring it on a cloth and holding it over your head (or wherever you want to cool down) will be much more effective. Also, while 28 °C isn't exactly an extremely hot day whatever the case, you may feel there is greater benefit in humid conditions to using the water to cool you down as your sweating is less effective Nil Einne (talk) 13:15, 4 August 2008 (UTC)[reply]
If you drink it, 100% of the temp of the water will go into changing the temp of your body, while if you pour it on yourself, only a portion will, and the rest will go into changing the temp of the air. However, there is no evaporative cooling if you drink it, while there is if you pour it on yourself. Evaporative cooling works best on hot, dry, windy days. If the water is hotter than the air and you, I would expect it would be best to leave it to cool to the air temp before using it for cooling. Also, if you are already sweaty, pouring water on yourself won't help much, as you already have the benefit of evaporative cooling from the sweat. StuRat (talk) 13:23, 4 August 2008 (UTC)[reply]
I've heard that drinking 500 ml cold water will make your body burn 100 kcal to get back at its normal temperature. Regardless of the correctness of it, it implies that any cooling of the body is useless in itself unless you have elevated temperature because cooling when having normal temperature will be reversed by your body. So, the correct answer would be; it depends on the body's temperature. --Ayacop (talk) 13:32, 4 August 2008 (UTC)[reply]
The math: 1 kcal raises a L of water by 1C. 0.5L of cold water raised by 35C (that's from near-freezing to body temp) = 17.5kcal.
Whether the water is cold or warm will certainly affect the comfort of drinking or wearing it. If it's too warm, it could exacerbate the problem of overheating...humidity will reduce the ability of evaporative cooling to effectively remove heat from the body by sweating or evaporating doused water.
Furthermore...one may assume that drinking the water allows one to capture 100% of the coldness and it will be used for further cooling as sweat. Pouring the water on one's self will waste some of it as it falls off the body. In general, drinking the water would be more effective as a cooling mechanism, and to stay properly hydrated. — Scientizzle 15:59, 4 August 2008 (UTC)[reply]
Extra drinking water will only lead to extra sweating if it's hot and you were in a dehydrated state to beign with. Otherwise, water you consume will just be eliminated as urine. StuRat (talk) 20:21, 4 August 2008 (UTC)[reply]
Let's do the math a bit differently. Imagine that you drink 1% of your body mass as water which is 60°F cooler than your body temp. That would result in lowering your body temp by about 60/100 or 0.6°F. That might not seem like much, but the body temp is very closely regulated, and that can be the diff between sweating profusely and being quite comfortable. (And, of course, if that takes your body temp too low, your body would react and burn some calories to bring it back up.) StuRat (talk) 20:28, 4 August 2008 (UTC)[reply]
Intuitively, it seems to me that if you only have a small amount of water you should drink it, assuming it is cooler than your body temperature. As noted above, this way you capture 100% of the coldness, and it helps supply your body with water for sweating. If you have plenty of water, you should drink enough to become well hydrated, and then pour some over yourself.
Partly, this question is hard because you've asked the wrong question. Body temperature is tightly controlled, so unless you are experiencing hyperthermia you can't actually "cool yourself down" with water. What you really mean is "which will make you feel cooler?" There, the answer is less clear, because it involves questions about how the rate of sweating and the sensation of being cool depend on your state of hydration.--Srleffler (talk) 02:46, 5 August 2008 (UTC)[reply]
Well, body temp is controlled, but it's not constant, it varies a bit. And even a few tenths of a degree in body temp is what makes the diff between you feeling hot or cold, so actually the question is dead on, as lowering body temp (a tiny bit) will also make you feel cool. StuRat (talk) 03:20, 5 August 2008 (UTC)[reply]
See [6] for previous responses to the same question.
The rate of cooling determines apparent temperature, not the amount of heat removed. Although drinking water removes more heat than pouring it on the skin, the latter feels much more effective in the short term because a thin film of water heats and evaporates very quickly. --Bowlhover (talk) 07:39, 5 August 2008 (UTC)[reply]
But, as already stated, pouring water on yourself will only help by evaporative cooling if it's hot and dry out, and yet you aren't already sweating, which is an unusual combo. One reason for this might be dehydration, in which case drinking the water becomes more important. Also, if you lower your temp too quickly then the body will react by generating heat, via shivering and other means. So, lowering body temp steadily may be the best way to go, especially if the effect lasts longer. StuRat (talk) 17:15, 6 August 2008 (UTC)[reply]
Evaporative cooling helps whenever the relative humidity is less than 100%. Poured water cools the body via conduction while itself heating up, and after reaching body temperature, the water would cool the body as sweat would do. As for reflexes, shivering and other reactions to cooling are trigged when the core body temperature drops; a drop in skin temperature would not decrease the core temperature because the body is already generating too much heat. --Bowlhover (talk) 11:08, 7 August 2008 (UTC)[reply]
Yes, evaporative cooling helps when less than 100% relative humdity, but adding water won't help in that way if the person was already completely coated with sweat. And cooling from conduction is never as effective as if the person drinks the water, as a portion of the coolness of the water goes into cooling the air, and much of the water drops off the person before it can cool anything down. I don't agree on shivering, however, as people shiver instantly when exposed to something cold, like diving into cold water, long before the core body temp could have had time to drop significantly. StuRat (talk) 14:58, 7 August 2008 (UTC)[reply]

Existance of God

Is there some connection between finding life on Mars and the existence of God? Does finding life on Mars refute the existence of God or prove it? —Preceding unsigned comment added by 71.100.5.89 (talk) 12:23, 4 August 2008 (UTC)[reply]

No. Algebraist 12:24, 4 August 2008 (UTC)[reply]
At worst, it would imply that the Judeo-Christian account of the creation of life was leaving out a few things. (But it's not like that was exactly on the line from a scientific point of view.) --98.217.8.46 (talk) 12:26, 4 August 2008 (UTC)[reply]
If you carefully read the Bible's account of the creation of extraterrestrial bodies, you'll notice that only "greater light", "lesser light", and "stars" are mentioned as being created: "sun" and "moon" are not used, and nothing is said specifically about the planets at all. I doubt anyone believes that this account includes everything. Nyttend (talk) 13:44, 4 August 2008 (UTC)[reply]
The Bible was written from the understanding of the universe they had at that time. Similar events occur in science where completely new ways of thinking or seeing things develop. These are called paradigm shifts. --Russoc4 (talk) 13:47, 4 August 2008 (UTC)[reply]
Neither. If/when we start discovering life on other planets, for the religious it will simply be a matter of saying "well, God created the entire universe and *all* the life in it". As far as I am aware, the Christian religion (I'll assume for the sake of discussion that you are referring to this) has never claimed that their God is only the god of *our* planet. Nothing will really change at all. It will still be impossible to prove or disprove the existence of an 'entity' which presumably would exist (if he/she/it exists) somewhere beyond/outside the universe as we know it. --Kurt Shaped Box (talk) 14:24, 4 August 2008 (UTC)[reply]
Quite the opposite, really. Christians believe God created everything, not just the stuff on Earth. —CycloneNimrodTalk? 16:01, 4 August 2008 (UTC)[reply]
That was the point I was making in the first place. Or at least I was trying to... ;) --Kurt Shaped Box (talk) 22:59, 4 August 2008 (UTC)[reply]
The existence of God might be strengthened by similar life on different planets (i.e. similar species). Life in itself doesn't really say much, it could have formed the same way we think it did here. —CycloneNimrodTalk? 15:26, 4 August 2008 (UTC)[reply]
The discovery of intelligent extraterrestrial life with very similar religious beliefs to those of humans would certainly provoke heated debate. I don't know how much the discovery of (for example) a fish-shaped creature in an extraterrestrial ocean really would prove, other than that that a fish shape is a good shape for swimming in the ocean and that convergent evolution works. --Kurt Shaped Box (talk) 15:56, 4 August 2008 (UTC)[reply]

Whew... I have to say that is certainly a relief. I would call this consensus that the discovery of life on Mars would not have any impact on the question of whether God exists or not so there is no reason for believers to panic or for non-believers to put them down. —Preceding unsigned comment added by 71.100.7.91 (talk) 19:17, 4 August 2008 (UTC)[reply]

There are always the same reasons as before ;) —CycloneNimrodT@lk? 19:48, 4 August 2008 (UTC)[reply]
God of the gaps is an article generally related to this topic. It discusses the interaction of theism and science over the long-term. ---J.S (T/C/WRE) 23:30, 4 August 2008 (UTC)[reply]
The question of whether or not there is a God in the abstract sense cannot and will not be verified or disproved by science. That is fairly clear. The question of specific religious ideas though can vary—if religions make concrete claims about the natural order of the world, then they can (and do) easily overlap into the domain of science, and conflict can (and does) occur. But if things are discussed in abstract, non-testable, entirely "unnatural" (that is, not pertaining to operations of the natural world) ideas, then there is not even the possibility of conflict. --98.217.8.46 (talk) 03:00, 5 August 2008 (UTC)[reply]
The abstract and the concrete, however, are quickly proving themselves to be reliant upon each other and inseparable for that reason and being only different aspects of the same thing. For instance, I can have an abstract concept which a computer program can emulate such as logical equation reduction. Although an abstract concept the results can be implemented directly by computer hardware, just as people can think and believe a certain abstract thing and control their actions accordingly to comply with or to implement the abstraction they are thinking and believe. Conversely, they can adapt their abstract thoughts and beliefs to accommodate feedback they get from the concrete world just a computer sensors can collect information from the concrete world which software can use to modify or to update an abstraction. In other words if you belief that God did not create life on any other planet and life was discovered on another planet then you would have to update your abstract belief with the concrete information or suffer abstract invalidation. —Preceding unsigned comment added by 71.100.7.91 (talk) 07:40, 5 August 2008 (UTC)[reply]

Calculation of pipe dia. Formula Needed?

Flow of water at standard pressure of 3bar, Calculation of pipe dia. to be used? When water needs to be pumped for a distance of 3 kilometers, and a constant pressure of 3 bar is supplied what is the optimal diameter(mm) of pipe that can be used? What formula can be used to calculate the diameter of pipe to be used when the pressure ,substance (water) and distance are known. Can anyone provide me with information? Arjaa (talk) 12:53, 4 August 2008 (UTC)[reply]

There's some missing info:
1) What is the desired flow rate through the pipe ?
2) Is the pipe straight and level or some other configuration ?
3) Is the pressure at the far end important ? StuRat (talk) 13:28, 4 August 2008 (UTC)[reply]


If the pipe is horizontal, and without leaks or taps, there should be no loss of pressure, and the diameter will only affect the flow rate. You can calculate the flow rate from the exit-velocity; density times velocity times flow area will give you mass flux. Mass flow rate has some equations that may put you in the right direction. If you use a pump or something else to induce a pressure differential across the pipe, you can use that to calculate the fluid velocity. Nimur (talk) 16:52, 4 August 2008 (UTC)[reply]
Is your first statement actually true in a real pipe? I would have thought that in an actual pipe there would be a pressure drop with distance due to the drag of the fluid on the walls of the pipe. The pressure drop should decrease with increasing pipe diameter (since there is less wall per unit volume), and also should increase with increasing flow velocity.--Srleffler (talk) 02:58, 5 August 2008 (UTC)[reply]
I'd say it's a suitable approximation; it would really depend on the fluid velocity (a stationary fluid would exhibit no drag!) The discharge coefficient is the quantity which would measure such a loss, but that's not typically applied to pipes, it's usually applied to the entry or exit orifice. In a real pipe, that quantity should be very close to 1.0. Nimur (talk) 04:48, 5 August 2008 (UTC)[reply]

Medical appliances (?), early/mid-20th C.

This picture shows prostheses and similar devices taken from victims of the Auschwitz II (Birkenau) camp and exhibited in a museum on the grounds of the Auschwitz camp. I haven't succeeded in identifying these other objects and need to know how to describe or otherwise refer to them. Would "medical appliances" be appropriate, or is there a more suitable term you'd suggest? -- Thanks, Deborahjay (talk) 13:50, 4 August 2008 (UTC)[reply]

My understanding, based on many years as registered nurse in the UK is that 'prosthetic' refers to some sort of replacement of a body part, sometimes to restore function sometimes for aesthetic reasons. Examples would be a glass eye or false hand for aesthetic reasons and wooden leg as a functional prosthesis. There are nowadays many more much more complex prostheses available. A 'medical appliance' I have always understood to mean a contraption, device or gizmo to assist some failing part of the body such as a spinal support for someone with scoliosis, a metal sprung calliper for drop-foot, a hearing aid for deafness or the good ol' truss for a hernia, as three examples. I hope that helps. Richard Avery (talk) 14:23, 4 August 2008 (UTC)[reply]
"Medical appliance" is quite vague, and includes such things as catheters and stoma appliances. The picture appears to be of orthopaedic-type equipment. It would perhaps be appropriate to refer to them as prostheses and orthoses, or prosthetic and orthotic equipment. Gwinva (talk) 22:14, 4 August 2008 (UTC)[reply]
I have two hesitations in accepting this solution en bloc: (a) some may indeed be hernia trusses (not clear in the photo, but used in this population) which are not orthopedic, and (b) the readership is mainstream (including non-native speakers and "orthoses/orthotic" are probably unfamiliar. Is there another comprehensive term rather than "medical appliance"?-- Deborahjay (talk) 06:39, 5 August 2008 (UTC)[reply]
If you're writing for a general readership, then "appliance" might be misunderstood; most people probably imagine an appliance is something you plug in. (WP doesn't even have an article for "medical appliance", although appliance mentions the concept briefly.) How about "prosthetics and other medical equipment" or "medical/health care aids"? Gwinva (talk) 22:52, 6 August 2008 (UTC)[reply]

Post thunderstorm ionization

Is it true that after thunderstorms people and animals feel better or get a 'high' from the change in ions in the air? Is this ionization? —Preceding unsigned comment added by 72.171.0.146 (talk) 14:42, 4 August 2008 (UTC)[reply]

No. In fact, some evidence[7] suggests the opposite, due to ozone production from the lightning. You may want to read ozone, though. Nimur (talk) 15:54, 4 August 2008 (UTC)[reply]

Tree of life

Does the DNA of currently-living organisms provide enough information to construct a full phylogenetic tree back to the most recent common ancestor of all such life? Are we likely to have such a tree in the next 50 years? Thanks. --Sean 14:51, 4 August 2008 (UTC)[reply]

No. We can make educated guesses and probably create a pretty good approximation, but we will never have a complete tree free from error. Several problems prevent this. First, many ancestral species no longer exist, which means one has to estimate the ancestral relationships between modern species. This can be done fairly well for recent speciation events, but as one looks further back in time, these ancestral relationship become far more speculative and there isn't enough information in currently existing DNA to be definitive. Keep in mind, more than 99% of all species that have ever lived are extinct. Secondly, with bacteria (and very infrequently some higher species) lateral gene transfer violates the assumptions generally used to construct such trees. As one gets closer to the most recent common ancestor, the more likely it becomes that non-descendent mixing will make it impossible confidently define descendent relationships. Dragons flight (talk) 15:42, 4 August 2008 (UTC)[reply]
But you might be interested in Richard Dawkins' book The Ancestor's Tale. It follows human evolution back and discusses the splits (or, in that direction, merges) encountered. He describes about 35 or so common ancestors, and also makes clear how much is hypothesis and how much is fairly solid. I found it in an airport bookshop on sale before I ever heard of Dawkins, and I was hooked by the beautiful writing and clear but not oversimplified exposition. --Stephan Schulz (talk) 00:23, 5 August 2008 (UTC)[reply]
Some clues to ancient life may be found in meteorites on the moon, or floating in the asteroid belt. Some would be derived from the earth, particularly in it's early stages. It is possible that vacuum dried bacteria may still be present. This is not a whole tree, just another node preserved from the past. This sort of thing may be found in the next 50 years. Graeme Bartlett (talk) 00:46, 5 August 2008 (UTC)[reply]

Blood plasma

If you take blood and just centrifuge it you get blood plasma. Plasma still has the clotting factors in it (as opposed to blood serum). My question is will blood plasma clot or does clotting only work when the cells and or platelets are in there? ike9898 (talk) 16:09, 4 August 2008 (UTC)[reply]

Yes, you need the platelets (and some factor proteins, read coagulation) which are in the heavy part of the centrifuged blood. --Ayacop (talk) 17:59, 4 August 2008 (UTC)[reply]

Minor correction; If you simply centrifuge whole blood you'll end up with a pellet of cells at the bottom and serum on top; If you centrifuge blood treated with anticoagulant you'll have a pellet of cells with plasma on top. When you have a blood sample taken you may notice that different colour tubes are used. The colours indicate the additive in the tube. If you put blood into a blue (Lithium citrate) tube the coagulation cascade is inhibited such that after centrifuging the coagulation proteins are still present in solution and the yellow fluid on top of the pellet is Plasma(or strictly speaking Plasma+Artificial anticoagulant). Various diagnostic tests/assays test different blood components, hence the need for different tubes. [8]Od6600 (talk) 16:56, 6 August 2008 (UTC)[reply]

Help diagnose my ear deficiency!

This question has been removed. Per the reference desk guidelines, the reference desk is not an appropriate place to request medical, legal or other professional advice, including any kind of medical diagnosis, prognosis, or treatment recommendations. For such advice, please see a qualified professional. If you don't believe this is such a request, please explain what you meant to ask, either here or on the Reference Desk's talk page.
This question has been removed. Per the reference desk guidelines, the reference desk is not an appropriate place to request medical, legal or other professional advice, including any kind of medical diagnosis or prognosis, or treatment recommendations. For such advice, please see a qualified professional. If you don't believe this is such a request, please explain what you meant to ask, either here or on the Reference Desk's talk page. --~~~~

Earths core

This is a schematic diagram of earth's interior. Nimur (talk) 16:57, 4 August 2008 (UTC)[reply]

How do we know the core of the earth is molten? Has someone drilled right down. Is it possible there could be another planet at the center like in journey to the center of the earth? —Preceding unsigned comment added by 79.76.204.221 (talk) 16:47, 4 August 2008 (UTC)[reply]

This is what we use to determine the above layers. Nimur (talk) 16:57, 4 August 2008 (UTC)[reply]
We measure earthquakes and compare the data from many different places on the planet. Then we use that data to solve the wave equation to estimate what kind of material the wave traveled through. In some cases the waves appear to travel through solid rock, and other cases the wave appears to have traveled through liquid rock. Close data analysis lets us estimate the density, and thus the temperature and pressure, and other parameters. The deepest drilling humans have completed is probably deep petroleum exploration, which may go as far as 10 or 12 kilometers in some extreme cases; this does not even penetrate the crust. It is very unlikely there is another planet or anything like Journey to the Center of the Earth speculates about. Nimur (talk) 16:58, 4 August 2008 (UTC)[reply]
The Bertha Rogers well was the deepest petroleum exploratory well and reached 31,441 feet (9,583 m) encountering pressures approaching 25,000 psi (172,369 kPa). In 1974 the hole was abandoned after encountering molten sulpher that melted the drill pipe!
The Kola Superdeep Borehole made it to 12,262 metres (40,230 ft or 7.62 mi) deep in 1989. Saintrain (talk) 23:01, 5 August 2008 (UTC)[reply]
(edit conflict) We know because earthquakes travel differently through the different layers, and refract at the boundaries. As for drilling, see Mantle_(geology)#Exploration. Note that that's for drilling to the mantle, not the outer core (the liquid layer). Anything in the center of the earth wouldn't fulfill the definition of a planet. If you're asking about having a gas or vacuum layer (so the next layer down would look like a planet), then no. It wouldn't conduct earthquakes as well as it does. If having a liquid layer with a solid layer under it counts, then it's correct. The outer core is liquid, and the inner core is solid. If you could survive the temperature and pressure, the inner core-outer core discontinuity would seem similar to a liquid covered planet. — DanielLC 17:16, 4 August 2008 (UTC)[reply]
But the inner core would be covered by an ocean of liquid metal, and that would be opaque and probably red-hot. It's not quite the typical "liquid-covered planet" you might envision. Nimur (talk) 17:43, 4 August 2008 (UTC)[reply]
What inner core made of? If iron why isnt it molten? —Preceding unsigned comment added by OpticalSphincter (talkcontribs) 17:48, 4 August 2008 (UTC)[reply]

Who said the core was molten? The core is solid. In fact most of the layers of the Earth are solid. The pressure would force any matter near the core into a solid. Of course if you were to bring that hot matter to the surface, it becomes a viscous liquid. ScienceApe (talk) 19:46, 4 August 2008 (UTC)[reply]

The outer core is liquid. Dragons flight (talk) 19:49, 4 August 2008 (UTC)[reply]
Actually, OpticalSphincter asked why the core isn't molten. It's made of mainly iron and nickel, and is solid because the pressure on it prevents molecules from moving freely. The Sun`s core is even more interesting; it comprises mainly hydrogen and helium at 15 million degrees Celsius, yet the density is 150 g/cm^3. --Bowlhover (talk) 19:52, 4 August 2008 (UTC)[reply]
How do we know that?
Pretty much the same way we know about the Earth's core, but rather than studying earthquakes, you study sunquakes. See Helioseismology. --Tango (talk) 03:41, 5 August 2008 (UTC)[reply]
Fun (aside) fact—in his memoirs, Edward Teller says that working with George Gamov on questions relating to the core of the earth gave him the idea that you could use a solid sphere of plutonium inside an implosion design nuclear weapon, that under the pressures of a few hundred megabars even an apparently dense solid could get a LOT denser than it would be at a single atmosphere. --98.217.8.46 (talk) 02:55, 5 August 2008 (UTC)[reply]

Galactic Center of the Milky Way

Hypothetically, if there was no cosmic dust and such in the way, how bright would our galactic center would be to us earthlings? If anything, how would this extra light or whatever, affect us on earth? --Anilmanohar (talk) 18:57, 4 August 2008 (UTC)[reply]

Well there should be a super massive black hole at the galatic core of our galaxy, and it's surrounded by an immense accretion disk. This disk should be very hot and larger than any other object in our galaxy, so it should be very bright. ScienceApe (talk) 19:52, 4 August 2008 (UTC)[reply]
Actually by itself the black hole, official known as Sagittarius A*, is very dim (as galactic nuclei go). Only about 1000 times the luminosity of the sun, or so. For comparison, the neutron star at the heart of the Crab Nebula and some individual stars, such as Rigel, are at least 10 times more intrinsically luminous than this black hole. Since the black hole is 27000 light years away (give or take), it would not be visible to the naked eye (even if you could distinguish it from all the stars clustered around the galactic center). As a mature black hole Sgr A* has already consumed most everything in its immediate vicinity and so now it accretes matter only very slowly. Young black holes, which have lots of matter to eat, can power active galactic nuclei which are among the brightest objects in the universe, but our black hole thankfully became quiet billions of years ago. Also, the bulk of the accretion disk is only about the size of a solar system. The black hole itself is not huge on astrophysical scales. Dragons flight (talk) 20:40, 4 August 2008 (UTC)[reply]
No not necessarily. I guess you are talking about the black hole at the center of our galaxy. But the it depends on which supermassive black hole we are talking about because they vary in size by quite a lot. The one in our galaxy is pretty small at only about 3.7 million solar masses. But Q0906+6930 is much larger at 16 billion solar masses and has a event horizon volume that's 1000 times our solar system. And that's just the event horizon. The accretion disk is much much larger. ScienceApe (talk) 03:28, 7 August 2008 (UTC)[reply]
(Edit conflict) Okay, let's work through this problem - it's an interesting one that requires a bunch of assumptions and formulas. First, Galactic Center gives us relevant information on the area immediately around the supermassive black hole, and it tells us that within 1 parsec (3.something light years) you "thousands of stars... most of them are old red main sequence stars." Great, so we can ignore most of them, because such stars are very very dim compared to the "more than 100 OB (blue giant)... stars that have been identified so far". We'll use Rigel as a model for this type of star, which is at 40,000 apparent solar luminosity. One more thing: this central parsec is 7600 parsecs from us, which, using trig, gives us 27 seconds of arc, which is about 1/60th the diameter of the moon in the sky, so roughly the angular size of a planet.
So now to crunch numbers to find apparent magnitude of this galactic center from Earth. We can add luminosities since it's just a measure of power and get about 4*10^6 solar luminosities, which is an absolute magnitude of -11.7, which is an apparent magnitude at 7600 parsecs of 2.7, which would make it one of the brighter lights in the sky. With all that done, it would probably be more relevant to get a number for the whole galactic bulge, since the combined luminosity could probably outshine the moon. But I'm tired now. SamuelRiv (talk) 20:16, 4 August 2008 (UTC)[reply]
This webpage has luminosity data on bright galaxies. Taking the spiral galaxies M31, M77, M81, NGC 3521 and averaging the brightnesses of their inner 2 arcminutes, we find that galactic nuclei are around magnitude 8.7 per 4pi square arcminutes or magnitude 20.3 per square arcsecond. This corresponds to class 5 on the Bortle dark sky scale, described as "Milkyway washed out at zenith and invisible at horizon. Many light domes. Clouds are brighter than sky. M31 easily visible. Limiting magnitude about 5.6 to 5.9." The galactic nuclei would be conspicuous unless light pollution is heavy. --Bowlhover (talk) 10:28, 5 August 2008 (UTC)[reply]

No one ha explained how this will appear to us earthlings or the effect it would have on earth...... --Anilmanohar (talk) 14:06, 5 August 2008 (UTC)[reply]

Using SamuelRiv's calculations, we would see something about the same size as a planet (i.e. showing features when viewed through binoculars or a telescope) and just about bright enough to be seen in an urban night sky. Gandalf61 (talk) 14:24, 5 August 2008 (UTC)[reply]
SamuelRiv only calculated the size of the parsec around the supermassive black hole; his purpose was to give an indication of brightness. Anilmanohar: see this excellent photo of the Milky Way. Look at the bright region on the right and imagine the dust lane does not exist. The galaxy would continue to get brighter as one approaches the center of where the dust lane is now, and the region around Sagittarius A*, as SamuelRiv calculated, would be magnitude brighter than magnitude 2.7. On average, the bright region would be as bright as a suburban night sky, but it would be extremely luminous at the center and comparatively extremely dim at the edges. --Bowlhover (talk) 20:56, 5 August 2008 (UTC)[reply]
So, in summary, an unobscured galactic centre would not be at all spectacular when viewed from Earth - we are just too far out in the galactic 'burbs. And, to answer the other part of the original question, I don't see how it would have any effect on life on Earth. Gandalf61 (talk) 09:00, 6 August 2008 (UTC)[reply]
It wouldn't be particularly bright, but I still think it would be quite spectacular - it's pretty impressive with the dust cloud in the way, without it it would be even better! --Tango (talk) 09:12, 6 August 2008 (UTC)[reply]

Why can't bullets fly ?

(fly in the sense that its form causes a little lift to compensate the free fall) —Preceding unsigned comment added by 77.224.230.72 (talk) 19:47, 4 August 2008 (UTC)[reply]

It does. A little. It would need constant thrust like a rocket though. ScienceApe (talk) 19:53, 4 August 2008 (UTC)[reply]

A form that would create lift would also increase drag, decreasing the effective range of the weapon. You also have to consider stabilisation; how would a lift-creating shape work with the spinning motion that most modern firearms use to increase accuracy? FiggyBee (talk) 20:02, 4 August 2008 (UTC)[reply]
Back in the olden days of SciRefDesk, you could get a whole treatise about how angle of attack, not asymmetric air-foil shape, can be key to lift. So have its center of mass displaced relative to the center of lift: essentially weight down the rear, increasing the angle of attack. The bullet flies rotating on its axis (spin-stabilized) but the axis is pointed nose-up. And has enough surface area on the "forward-facing bottom" so that the air pushes the bullet up, but not so much that you lose all your forward motion. And still maintain stability in the forward (as well as axial) direction without starting to tumble. Pretty soon you're just firing a normal rifle nearly straight up:) DMacks (talk) 21:50, 4 August 2008 (UTC)[reply]
Now that sounds interesting. Can you point us to an article that says all that/ —Preceding unsigned comment added by MisterRSole (talkcontribs) 01:10, 5 August 2008 (UTC)[reply]
Indeed, I would like to see that treatise. It seems to me that after the bullet exits the barrel in a straight line, the extra weight at the rear would cause the bullet to (gyrate? pronate? precess?) go into a speed wobble. Franamax (talk) 01:54, 5 August 2008 (UTC)[reply]
I had the same reaction. Weighting the rear would precess the spinning bullet and it would wobble its way dramatically off course. I'm skeptical of any angle-of-attack-related lift for a spinning, axially-symmetric shape. Nimur (talk) 04:53, 5 August 2008 (UTC)[reply]
Bullets can and do wobble to some degree—a really wobbly bullet produces horrible forensic wounds if it hits someone. I believe the degree to wobbling depends on how the weight is distributed in the bullet itself. --98.217.8.46 (talk) 02:49, 5 August 2008 (UTC)[reply]
Note my conclusion: "Pretty soon you're just firing a normal rifle nearly straight up". I'm completely doing thought experiments, speaking from first-principles physics and not any specific training in balistics design. The angle-of-attack issue is real for airplane wings though. But back to the bullet...by the time you fix all the problems and have enough tilt and enough spin to maintain stability and a steep enough angle of attack, your gonna tumble or something else rather than "fly". I don't think you could get a stable design that gave enough push "up" enough to generate real lift without pushing "up and over backwards" and creating back-spin. Wobble really does occur (and pretty much makes hamburger of whatever it hits). An easy way to get it is to have the weight distributed off-center from the axis...rotation about the center of mass thus has the bullet itself spinning off-center. Dunno how badly pushing center of gravity forward or backward would lead to wobble (lift on one end of the axis of rotation would produce torque at right angles, sending the bullet wobbling or curving sideways?). Maybe could also off-axis weight to counteract it? DMacks (talk) 04:25, 5 August 2008 (UTC)[reply]
Yeah, if I remember my gyroscopes right (which I don't) changing the direction of the axis of rotation produces precession (hah, I got to use the word :). Since the direction of flight isn't changing, that would add up to a wobble. Now I don't think adding an off-axis weight could counteract that, because - where would you put the weight? The bullet is spinning, so the off-axis weight is spinning around too, seems to me you could only increase the wobble. And now to complicate things, bullets are pointy-shaped, so aren't they already heavier at the back end? Franamax (talk) 04:57, 5 August 2008 (UTC)[reply]
It's not "heavier at the back" (CoG relative to object's overall form) but (dim memory) where the CoG is compared to center of lift or something like that. There was something in an Estes catalog about finding CoG of a model rocket, then tying a string there (wherever it was front-to-back) and swinging it around. If it flew level (not pitching up or down) then you were all set and you wouldn't wind up launching at through the neighbor's window by accident. Well I'll be damned, I managed to learn some science from the toys I played with as a kid! (learned about torque from Capsela too:) DMacks (talk) 05:50, 5 August 2008 (UTC)[reply]
OK, I think I understand you, the rocket on a string analogy is good, I thought they were only good for poking your eye out. Now the rocket over its various shape and contours will have areas of greater or lesser positive and negative lift. Adding up these vectors will give you the centre of lift. But how do you do that with the spinning bullet? If a patch of the surface has positive lift, it will have negative lift in one-half a revolution. Spinning the bullet will put one centroid of lift onto the rotation axis, now how do we move the fore-aft centroid away from the CG? Add aerodynamic drag at the back end of the bullet with little extra furrows? Would that add net negative lift at the back? Franamax (talk) 06:38, 5 August 2008 (UTC)[reply]
If one could stabilize the spinning bullet with an angle of attack, the Coanda effect would cause it to curve way off course (see curveball).
A (perfect) flying bullet would have about the same range as a ballistic one but, when it got to that range, it would just drop out of the air with no impact speed, all its umph having been used up as drag.
(Side note:if you fire a bullet horizontally and drop one from the same height they'll both hit the ground at the same time! (Unless one of them is "flying" :-) How cool is that!?) Saintrain (talk) 22:23, 5 August 2008 (UTC)[reply]
  • 'Cause the distance the air has to travel on the top of the bullet is the same as the distance the air has to travel on the bottom of the bullet. Hence, no vacuum. For an airplane wing, it is curved on the top and straight on the bottom. Comparted to the wing bottom length, the longer air travel distance on the top results in vacuum, which sucks the airplane right off the ground. Of course, if you laid a bullet on the ground and put the end of a vacuum nozzle over it, the bullet would fly upward. And if you shoot a small airplane wing out of a gun, you might miss what ever you are aiming at. Suntag (talk) 17:28, 7 August 2008 (UTC)[reply]
    • "Back in the olden days of SciRefDesk, you could get a whole treatise about how angle of attack, not asymmetric air-foil shape, can be key to lift." DMacks (talk) 21:50, 4 August 2008 (UTC) Ah, good times. Good times. Practical experiments were recommended and everything. 79.66.38.215 (talk) 23:28, 7 August 2008 (UTC)[reply]

Plant identification: red berries

I saw these berries in High Park Toronto yesterday while I was out for a walk. I was wondering what kind of berries they are and if they are edible. They looked like red currants, but I'm not sure if that's what they are. Please see the photo I've linked bellow.

http://img.photobucket.com/albums/v244/shigil/berries.jpg —Preceding unsigned comment added by 99.254.23.238 (talk) 21:31, 4 August 2008 (UTC)[reply]

I don't live in Ontario anymore, otherwise I'd be running out to my little backyard paradise to look at the Honeysuckle bushes I planted. That was my immediate thought when I brought up the photo you link. Unfortunately, I no longer have the record of the exact species and my Audubon guide is not helping out, so maybe someone else can chime in here.
Our red currant article states "3-10 berries on each raceme" and I can't quite get to three when I count from your pic, so I'd say red currant is unlikely.
And of course, unless you're sure, assume they're not edible!
And looking a little more, a lot of people love High Park [9] [10], so I'm sure you can find some locals to ask if we can't pin it down. Franamax (talk) 01:35, 5 August 2008 (UTC)[reply]
They give me the impression of being from the dogwood family. This family is quite large and more expert knowledge will be needed to make an accurate identification. I certainly wouldn't put them on my pizza. Richard Avery (talk) 07:32, 5 August 2008 (UTC)[reply]
Lonicera morrowii I think. See http://www.ppws.vt.edu/scott/weed_id/lonmo.htm and http://www.illinoiswildflowers.info/weeds/plants/morrow_hs.htm William Avery (talk) 19:02, 6 August 2008 (UTC)[reply]
Aw crap - looks like I'm right about honeysuckle, but I always (tried) to take care to only plant native species in my yard. Maybe I wasn't clear with Dan at the garden centre that year. Any chance there's an equivalent native Ontario species? (I agree with your diagnosis, just trying to salvage my principles. You may have nailed the OP's plant right, it's my own I'm worried about now :) Franamax (talk) 09:41, 7 August 2008 (UTC)[reply]

Why did Newton think light speeded up in glass?

The Principle of least action was already a well known principle that explained Snell's law when Newton was investigating optics, so why did he come to the conclusion that light went faster when it entered glass rather than slower? —Preceding unsigned comment added by Dmcq (talkcontribs) 21:42, 4 August 2008 (UTC)[reply]

Did he? I hadn't heard of that, but it's a reasonable layman's interpretation to infer from observation that sound waves travel faster in denser materials, so perhaps electromagnetic waves would do the same. It's wrong, but understandable. ~Amatulić (talk) 22:29, 4 August 2008 (UTC)[reply]
Yes, it's how he explained refraction (thus accurate measurements of the speed of light in different materials were the first good evidence against Newton's ideas). I don't know how he came to this position, but it's probably in his Opticks. Algebraist 23:20, 4 August 2008 (UTC)[reply]
It is explained in [11]. Newton imagined that as light moved through air, the net attractive force on it from all air particles equaled zero. But at the interface between air and water, there was a net force pulling it towards the water, speeding it up. As its velocity along the surface remained unchanged, light would bend. this was Newton's explanation of snell's law and he was quite happy about it. :-) The different refractions of different colours were explained by the different colours being of different masses, so that the same force gave different changes in velocity. It wasn't until the 1850's that Léon Foucault got to compare the speed of light in air and water and discovered that light slows down in water.
As an apropos, [12] tries to speculate on what Newton would have made of simple experiments where the refraction of colours are opposite to the usual. EverGreg (talk) 07:12, 5 August 2008 (UTC)[reply]

Thanks very much. Very interesting. I must say I find his argument a bit unconvincing though and can see why he would be worried about Hooke! By his argument light of a single colour even if all the corpuscles were going a the same speed would have a different speed after entering glass depending on the angle they entered it at. But all the different speeds would have their vertical speeds leaving the glass changed by exactly the same ratio, that's a very strange idea for a particle theory - you'd expect the slower ones to change more if anything. I can see why he'd want a corpuscular theory with different shaped particles to try and explain polarization though. Dmcq (talk) 07:56, 5 August 2008 (UTC)[reply]

Sorry I misunderstood. He said the vertical attraction would change the velocity by the ratio rather than the vertical component. That makes a lot more sense. I guess he might have modeled it as two flat surfaces separated by a slope and the light particle going faster after it goes down the slope, the speed change wouldn't depend on the angle the slope was approached at. If the particles had differing speeds though they would bend differently so I guess he thought all light of a particular colour was going at the same speed - the obvious way then would be to have the different colours going at different speeds rather than have different masses. I must check if that is what he thought. Also I guess he liked having different particles for the different colours because Fourier was far in the future - who would have though different wavelengths could practically ignore each other like particles. Dmcq (talk) 18:27, 5 August 2008 (UTC)[reply]

Nope it looks like Newton believed all light moved at the same speed in space and he actually had quit a good estimate. I guess his reasonng was different speeds for different colours would give funny effects when looking at the moons of Jupiter. So he thought different colours were different masses. All a bit strange but it loks like he wasn't altogether sure of his theory himself because of Newton's rings Dmcq (talk) 19:13, 5 August 2008 (UTC)[reply]

Gas

Let's say I had a closed vessel containing only two liquid chemicals (no air), and this vessel had a known weight. If the two chemicals started a reaction which produced a gas, and I were to weigh the vessel afterwards, would the recorded weight constant (i.e. can a balance record the mass of a gas)? A second and unrelated question, how does water behave in a vacuum? —Preceding unsigned comment added by 76.68.246.7 (talk) 22:12, 4 August 2008 (UTC)[reply]

Assuming the vessel itself has mass, you can certainly weigh it "full of" something (liquid, gas, whatever), and then subtract the weight of the vessel itself to see what the effect of its contents are. If your reaction produces a gas, remember gas is still matter, and that matter has to come from somewhere--the liquid! See Conservation of matter. DMacks (talk) 22:15, 4 August 2008 (UTC)[reply]
As to how water behaves in a vacuum, the phase diagram for water shows that at zero pressure and some positive temperature, water will be in a gaseous state. The phase diagram for water also shows an anomaly in that for a certain range of temperatures, increasing the pressure will cause a transition from gas to solid, and then to liquid. Most other substances don't behave that way. ~Amatulić (talk) 22:27, 4 August 2008 (UTC)[reply]
The mass of the liquid, solid and gaseous contents of a sealed container would not change measurable as it went through chemical reactions or phase changes. In theory, it should weigh a tiny bit less (too small a change to detect on existing scales) if it emitted energy by heat or by radiation, and it should weigh more if it absorbed energy through the container, due to E=MC2. Edison (talk) 23:57, 4 August 2008 (UTC)[reply]
You'd need a very accurate scale to measure that! Even nuclear fissions yield mass discrepancies of tiny fractions of an atomic mass unit (relatively, on the order of hundredths of a percent). A chemical reaction has a far smaller energy change; as Edison said, no conventional mass measuring equipment could detect it (I am imagining some sort of emission spectrum fine structure analysis might reveal some indirect measurement which could reveal mass changes on this order of magnitude, but I've never seen anything like that in any lab I've been around). Nimur (talk) 05:01, 5 August 2008 (UTC)[reply]
I think you misunderstood my question. I know the total mass in the vessel will remain the same. I was wondering if a balance can detect the weight of a gas (afterall, it's not really touching the balance).
Yes, a balance can weigh a gas. Ultimately, the balance is reacting to pressure exerted, rather than "touch". Similarly, a truck full of birds weighs the same if all the birds begin flying within the truck. — Lomn 18:17, 5 August 2008 (UTC)[reply]
Whether the gas touches the balance or not, gravity is still acting on it, and it in turn presses down on the other substances in the vessel. If you stand on a scale, carrying a backpack would increase the weight reading even though the backpack is not in contact with the scale.
Yes, but if you let go of the backback (i.e. it's in freefall), then the scale won't take it into account. From my knowledge of gasses, molecules are touching each other and thus spend a considerable portion of their time in freefall, and so the balance shouldn't be able to detect their weight. So wouldn't the total weight measured be slightly less, because some of the atoms of gas are in freefall? —Preceding unsigned comment added by 76.68.246.7 (talk) 13:04, 7 August 2008 (UTC)[reply]
If you're trying to actually weigh a gas, however, you must take buoyancy into account. It has a significant effect on apparent weight when dealing such low-density substances. --Bowlhover (talk) 20:29, 5 August 2008 (UTC)[reply]
Buoyancy shouldn't have a meaningful impact on this example, though, with the gas in question enclosed in a vessel. — Lomn 23:20, 5 August 2008 (UTC)[reply]

Viscious liquid

i read lot of science pages. I notice lot of mention of vicious liquid. Why is it vicious. will it attack me/ —Preceding unsigned comment added by MisterRSole (talkcontribs) 23:07, 4 August 2008 (UTC)[reply]

Yes. It is a a very dangerous liquid. Simply vicious. It will tear your heart out if you give it half a chance. On the other hand a viscous liquid is a very thick, slow flowing liquid, such as maple syrup. Though they may lead to sticky situations, most viscous liquids are not plotting to kill you. Dragons flight (talk) 23:16, 4 August 2008 (UTC)[reply]

Viscosity is a measure of the resistance of a fluid which is being deformed by either shear stress or extensional stress. In general terms it is the resistance of a liquid to flow, or its "thickness". - Viscosity

(vicious edit conflictx2)Does that help? ---J.S (T/C/WRE) 23:17, 4 August 2008 (UTC)[reply]
Of course, viscosity is definitely at its finest with Non-Newtonian fluids. -- Captain Disdain (talk) 01:38, 5 August 2008 (UTC)[reply]
...Funny you should say that, because one of the definitions of a non-Newtonian fluid is one that doesn't have a single, well-defined viscosity. —Keenan Pepper 02:15, 5 August 2008 (UTC)[reply]
Yeah, and that's what makes them cool. -- Captain Disdain (talk) 10:02, 5 August 2008 (UTC)[reply]
Like cold custard? —Preceding unsigned comment added by 79.76.159.153 (talk) 20:47, 5 August 2008 (UTC)[reply]
According to this[13] the danger is the sharks. Julia Rossi (talk) 02:16, 6 August 2008 (UTC)[reply]

Expansion on shaking container of hot water

If I take a plastic food storage container and partially fill it with cold water, snap the lid on and shake it, nothing happens. If I repeat the procedure with hot water, very shortly after I begin shaking, the lid will vigourously pop off due to pressure buildup inside the container. If I do the experiment with a mason jar, there's a "Pfff!" of escaping air if I loosen the lid after shaking, but again, only with hot water. Why? The addition of dishwashing liquid seems, if anything, to amplify the effect, though I'm guessing this is due to the volume of suds produced. —Scheinwerfermann (talk) 23:20, 4 August 2008 (UTC)[reply]

Hot water heats the remaining air in the container. Warm air takes up more space than cold air. Shaking the container mixes the water with the air, warming it more quickly. Pressure builds up until the pressure difference between the contained warm air inside and the free, cool air outside is enough to pop the lid. 79.66.32.107 (talk) 23:30, 4 August 2008 (UTC)[reply]
Also, if the air is quite warm and the water is cold, you might notice the container collapse a bit. ---J.S (T/C/WRE) 23:45, 4 August 2008 (UTC)[reply]
"As the temperature is raised, gases usually become less soluble in water" <-- Maybe the heated water is releasing dissolved air. --JWSchmidt (talk) 03:55, 5 August 2008 (UTC)[reply]
Today's safety tip: Be very careful when blending hot liquids, basically for the reasons 79 gave above. -- Coneslayer (talk) 12:10, 5 August 2008 (UTC)[reply]
@JWSchmidt: Not likely, for two reasons. One; the water isn't being heated in the container and two; the dissolved gasses take up very little volume. ---J.S (T/C/WRE) 17:27, 5 August 2008 (UTC)[reply]

Here's a nice simple experiment to demonstrate the expansion of air with heat. All you need is a sink with hot water and an empty lightweight plastic bottle (like a big soda pop bottle). If your hot tap water is not very hot, you might start by heating some with a stove, microwave oven, or electric kettle instead of using it straight from the tap, in which case a funnel might be handy for pouring safely. Okay, first fill the bottle with hot tap water to warm the bottle, then dump out the water. Next put in just a little fresh hot water from your hottest supply in the bottom. Cap the bottle and shake it vigorously so the water heats the air, then uncap it and drain the water. Repeat with a little fresh hot water again, to make sure the air is really hot. After draining the water this time, quickly recap the bottle and close tightly. The heated air will now cool to room temperature and you will see how much it shrinks. --Anonymous, 18:36, August 5, 2008.

August 5

Apparent Incandescence of Planets

When viewed from earth, why do the planets of the solar system appear to be incandescent, as if they were stars? [This paragraph has been corrected by the originator. Sorry, folks, I didn't express myself right.]

We know that all the planets are below the temperature of incandescence.

Similarly, the earth's moon has a glowing appearance, rather like a fluorescent lamp. But men have walked on the moon and found it did not glow when the observer is close to it.

Things should get dimmer as they become farther away, not brighter. We know that from observation of things on earth. That also conforms with the principle that radiant energy decreases with the square of the distance.

What is the solution to this paradox? —Preceding unsigned comment added by Andme2 (talkcontribs) 04:26, 5 August 2008 (UTC)[reply]

Planets and moons shine because they reflect the light of the Sun, in varying amounts according to their albedo. Watch the moon throughout a month though, you will see large parts of it that don't glow brightly - those are the parts turned away from the Sun. (Oh and they're not the Earth's planets, they revolve around the Sun) Franamax (talk) 05:01, 5 August 2008 (UTC)[reply]
Are you suggesting that we don't own the entire solar system? I find it strange that your definition of ownership would depend so heavily on gravitational attraction. Does that mean that the Earth owns your car and your house? Human-made paperwork notwithstanding, Earth is certainly outcompeting you on net induced gravitational force acting on everything you think you possess. Or, you can adopt a philosophy that ownership transcends gravitational attraction... Nimur (talk) 05:06, 5 August 2008 (UTC)[reply]
Well on the few occasions when I've taken my car off the Earth's surface, Earth has exercised its prior claim to ownership pretty darn fast. I haven't tried launching a house yet, for one thing if I didn't have gravity, I would have wasted all that money on floors. Not to mention toilets with pipes that go downwards... Franamax (talk) 05:57, 5 August 2008 (UTC)[reply]
And since there is a chance you're serious: 1) ownership implies control, so Earth tells the Moon what to do, but it sure doesn't tell Ganymede when it's bedtime; 2) ownership implies relevance, if my car doesn't have me around it's not much good (since I'm the only one who has the keys), whereas if Earth's not around, Jupiter still works just fine; and 3) philosophize all you want, the Earth is what you're made from and the Earth is where you're gonna end up, all nice and molecule-like. Write your paperwork up, it's gonna end up as dirt too eventually. Of course, there will always be a Wikipedia :) Franamax (talk) 06:14, 5 August 2008 (UTC)[reply]
Wait, wait a second... How can you be so sure we came from Earth when even NASA is trying to find out if we really came from Mars? —Preceding unsigned comment added by 71.100.162.249 (talk) 20:58, 5 August 2008 (UTC)[reply]
They seem bright because you are looking at them contrasting with a black night sky. If you look at the moon in the day time (easy) or at the planets in day time - you need to have binoculars at least and know where to look you will find that the moon and planets are much dimmer looking. Only Venus is really bright, and that is because it is very white and closer to the sun. Graeme Bartlett (talk) 06:50, 5 August 2008 (UTC)[reply]
"But men have walked on the moon and found it did not glow when the observer is close to it." Actually, the Moon's surface reflects about 12% of incident light, giving it an albedo similar to that of worn asphalt.
Anyone who has looked through a telescope knows that the planets are far, far dimmer in terms of surface brightness than any star. No amateur telescope can show the disc of any star except the Sun; hence, stars appear to have about the same surface brightness regardless of magnification. Planets rapidly dim when the magnification is increased. --Bowlhover (talk) 07:23, 5 August 2008 (UTC)[reply]
First, each point on a planet or Moon does get dimmer as distance increases. However, you also can see more of the surface of such an object as you back off, up to the point where it all fits in your view. Backing up beyond that makes the object seem smaller, but this essentially squeezes all the reflected light down to a smaller and smaller area, such that the total amount of light that hits one rod or cone in your eye, in theory, remains the same, if there's no dust in between to absorb the light. This continues until the light from the planet or Moon only hits one rod or cone. From that point on the total amount of light hitting that point in your eye decreases with distance. However, the nearby planets and our Moon aren't far enough away for that to happen, while all stars except the Sun are. StuRat (talk) 12:18, 5 August 2008 (UTC)[reply]
"However, the nearby planets and our Moon aren't far enough away for that to happen" Just to clarify, all planets except Earth are seen as points with the naked eye. There are exceptions, but those are rare. --Bowlhover (talk) 21:13, 5 August 2008 (UTC)[reply]
Using my def of a point light source as one where all the light hits a single rod or cone in the eye, the nearest planets don't qualify. StuRat (talk) 17:09, 6 August 2008 (UTC)[reply]
By the def, not even the stars are point sources. The lens can't focus light perfectly, but even if they could, the Airy disk would spread out the light. These two factors are the reason bright stars like Sirius appear larger than dim ones like Gamma Trianguli. It's true that planets should appear much larger than stars, but only the actual appearance of stars and planets is relevant to the question. --Bowlhover (talk) 20:28, 6 August 2008 (UTC)[reply]
Another effect to consider is that our eyes dilate or contract in an attempt to make everything appear equally as bright, so that something like the Moon, which isn't very bright at all, seems bright when viewed at night. When you see the Moon during the day, you can really compare it with the Sun, and see how dim it really is. StuRat (talk) 12:22, 5 August 2008 (UTC)[reply]
Yet another effect is the night vision that only becomes fully effective after 30 minutes in the dark. Judging from http://webvision.med.utah.edu/imageswv/KallDark1.jpg, the eye's sensitivity threshold decreases by a factor of 60 000 after half an hour. Pupil dilation has a significant effect only in the first few minutes of dark adaptation, in which it allows 4-16 times the light-adapted amount of light to pass through (http://sightresearch.net/files/adaptation.htm). --Bowlhover (talk) 20:07, 5 August 2008 (UTC)[reply]

Height of mountains

how are the height of mountains calculated? how accurate are these?Shraktu (talk) 04:39, 5 August 2008 (UTC)[reply]

It could be done by dead reckoning, by pressure altimeters, or by triangulation. In modern times, GPS and RADAR are some other options. Nimur (talk) 05:03, 5 August 2008 (UTC)[reply]
Here's an article from Slate, but it's not actually all that informative... Nimur (talk) 05:23, 5 August 2008 (UTC)[reply]
When I was in high school I was taught that 0 m starts at the geoid level. --Kjoonlee 06:03, 5 August 2008 (UTC)[reply]
Yes it is not that easy to calculate exactly where the geoid is for a mountain a long way inland. This has been a diffulcult job for surveyors. It will involve taking precise measures of the vertical and comparing it with the known 0m datum at many points on a path to the top of the mountain. See if geodesy#Heights has anything. Graeme Bartlett (talk) 06:44, 5 August 2008 (UTC)[reply]
You may want to read Mount Everest for some info on the height and various ways it was measured Nil Einne (talk) 08:52, 5 August 2008 (UTC)[reply]
I believe triangulation is the most frequently used method. It has the advantage of not requiring a physical presence on the top of the mountain - don't forget that even today some mountains have not yet been climbed. Professional triangulation can also be amazingly accurate - the Great Trigonometric Survey of India in the 19th century measured the height of Mount Everest to be 8,840m, even though they were measuring from the border of India, about 150 miles away. This is within 0.1% of the current official height of 8,848m. Gandalf61 (talk) 08:58, 5 August 2008 (UTC)[reply]
You need some kind of zero point base to measure from, usually "sea level", which is a vague term at best. See Datum (geodesy)#Vertical datums. Pfly (talk) 09:18, 5 August 2008 (UTC)[reply]
Well, yes, but that is true of all the methods mentioned - they only actually give you a height relative to a base line. You then need some convention for defining the elevation of the base line - so you use a geodetic system. In the 19th century you put your base line near the coast (the Great Trigonometric Survye of India established its base line at Chennai) and measured its elevation relative to local mean sea level, which you averaged over several years. A modern survey would use the World Geodetic System. Gandalf61 (talk) 09:40, 5 August 2008 (UTC)[reply]

The height of mountains being above sea level means that on Mars or the Moon the mountain hieghts would be undefined, and that on Earth, as the hypothesized sea level rise progresses, all mountains would need periodic adjustments down in their reported heights. Seems odd. Edison (talk) 13:40, 5 August 2008 (UTC)[reply]

Yea it is non-ideal. Another thing is that a mountain like Mount Hood seen from Portland looks much taller than Pikes Peak seen from Colorado Springs, even though Pikes Peak is actually higher. But Portland is nearly at sea level and C-Springs is something like a mile high. But then, measuring elevation from sea level makes sense given human history and culture. Measuring from the center of the Earth might be more... scientific? Still, it's more useful to know how far above sea level your house is as a hurricane approaches, for example, than how far your house is from the center of the Earth. I agree it is odd though. "Sea level" is not the most stable and well-defined baselines. On the other hand, mountains are not exactly stable either, with their height changing not only from erosion but from larger scale uplift and sinking of land. This kind of height change may usually be slower than sea level change, but in some cases it happens catatrophically fast! Pfly (talk) 16:22, 5 August 2008 (UTC)[reply]

Semiconductor Resistivity

What is the "Four Point Probe Method" in measurement of Semiconductor Resistivity?Shraktu (talk) 06:27, 5 August 2008 (UTC)[reply]

I think you are looking for Four-terminal sensing --Dr Dima (talk) 06:33, 5 August 2008 (UTC)[reply]

Applications of Chem kinetics

do you know any websites on the applications of chem kinetics? Jaydilite2008 (talk) 12:10, 5 August 2008 (UTC)[reply]

Chemical kinetics would be a good place to start. EagleFalconn (talk) 13:18, 5 August 2008 (UTC)[reply]

Kissing scenes in movies - health risks?

In movie production, how do they prevent diseases from being passed from one actor/actress to another when they kiss? It doesn't look like there a lot that can be done when you see open mouths touching each other. On the other hand, if you kiss enough random people, sooner or later you'll catch something. Are actors and actresses really risking their health when shooting kissing scenes or do they have some kind of high-tech tricks not visible to the viewer that prevent disease transmission? —Preceding unsigned comment added by 71.185.73.240 (talk) 14:26, 5 August 2008 (UTC)[reply]

They are really kissing, if that's what you're asking. They probably do occasionally pass colds on to one another that way. But it's not much of an epidemic—the sum total of people kissing in movies per year is really just a handful. --98.217.8.46 (talk) 15:06, 5 August 2008 (UTC)[reply]
Plus during the filming of any one movie an every-day character I wouldn't expect will kiss more than 5 or 10 of their fellow actors on the movie. Given that shooting a movie can take anything from weeks to years it's hardly like the actor will be kissing significant numbers of individuals so I doubt the risk will be notably higher than the general population. 194.221.133.226 (talk) 15:14, 5 August 2008 (UTC)[reply]
Yeah, kissing is not risk-free, but it's not much different than shaking hands or other forms of close contact. Kissing#Disease_transmission is pretty scant but touches on the health aspects. Friday (talk) 15:15, 5 August 2008 (UTC)[reply]
The diseases spread via kissing are usually upper respiratory tract infections such as the common cold and influenza. Most people in good shape will recover from these within a week or two. Slightly more rare are the diseases such as infectious mononucleosis (kissing disease, mono etc) which are still usually non-fatal but take longer to recover. The risk is there but I suppose it's part of the job. —CyclonenimT@lk? 17:12, 5 August 2008 (UTC)[reply]
Lots of movies have prolonged tougue-wrestling scenes. There are maladies more serious and persistent than colds and flu which could be spread by such contact. I wonder if standard contracts at mainstream studios require medical certification that the co-star is free of herpes, Infectious mononucleosis, Hepatitis B and venereal disease in general? The the Columbia University health information site "Go Ask Alice" says these could be transmitted by kissing [14], although they call it "highly unlikely." Edison (talk) 19:19, 5 August 2008 (UTC)[reply]
How do they prevent that? For the most part, they don't. I mean, if someone has a terrible cold, I'm sure they avoid filming the kissing scenes to try and ensure they don't have two sick actors on the set, which'd be a waste of time and money. Likewise, if someone has a herpes flare-up, I'm sure they avoid kissing for obvious reasons. And of course, actors and directors tend to get medical check-ups before filming begins (for insurance purposes), so if they have any even remotely serious communicable diseases, chances are that they're known and being dealt with. But apart from situations like this, I don't think it's a major concern, any more than kissing someone on a date is. I mean, I'm sure some people just don't kiss anyone if they can avoid it, but for most people -- whether they're in front of a camera or not -- kissing is not first and foremost thought of as a health risk. I mean, if you're going to be worried about that kind of thing, chances are that you'll also want to avoid shaking hands with people, eating out, engaging in many sports, etc. -- at which point one kind of has to wonder if you're suffering from mysophobia or something. It's not really a very reasonable concern, all things considered. -- Captain Disdain (talk) 00:41, 6 August 2008 (UTC)[reply]

Do newspaper inks contain cyanide?

Do the inks commonly used to print newspaper and newspaper inserts contain cyanide? —Preceding unsigned comment added by 71.185.73.240 (talk) 16:18, 5 August 2008 (UTC)[reply]

This may help you. —CyclonenimT@lk? 17:09, 5 August 2008 (UTC)[reply]
Cyanide is a very dangerous chemical. I wasn't able to find any reference to it on any website discussing the dangers of newspapers/inks/etc. On the other hand, I did not find an itemized list of the chemicals in ink. ---J.S (T/C/WRE) 17:47, 5 August 2008 (UTC)[reply]
According to the link I just gave (albeit not in the free abstract), ink used in newspapers in the U.K. contains acetonitrile which is a cyanide.
Saying that acetonitrile contains cyanide is like the old trope about salt being made up of sodium, an caustic and explosive metal, and chlorine, a poisonous gas. While true in the sense that salt contains sodium and chlorine atoms, they exist in salt as (mostly) harmless ions, not as the poisonous elemental forms. Likewise with acetonitrile. Though it contains a cyano group, which is like the cyanide ion, the cyano group is covalently bonded to another carbon atom, which completely changes its reactivity versus the cyanide ion. While not completely harmless, acetonitrile is very different from the cyanide ion in the way it behaves in your body. It's like the difference between putting a cotton ball (cellulose) into your coffee and putting a teaspoon of sugar (sucrose). They may both be carbohydrates containing glucose groups, but what those glucose groups are attached to and the way they are attached makes a world of difference. -- 128.104.112.147 (talk) 20:49, 5 August 2008 (UTC)[reply]
Is it possible that Potassium ferricyanide or Prussian blue are used?87.102.5.5 (talk) 17:52, 5 August 2008 (UTC)[reply]

is this snake poisons —Preceding unsigned comment added by 71.3.150.136 (talk) 18:40, 5 August 2008 (UTC)[reply]

Strictly speaking, yeah, I guess its possible. However, acetonitrile and cyanide should not be confused. Acetonitrile is a very common organic solvent, cyanide is an ion. Acetonitrile has been known to breakdown into cyanide in human metabolic processes, however, and thats likely where any toxicity from newspaper ink would come from. EagleFalconn (talk) 20:00, 5 August 2008 (UTC)[reply]

Seems that nearly all the solvent from the ink would evaporate (otherwise the ink would never dry!), and that toxicity from the ink could be due to all the pigments too. Even the cited article above talks about a very different toxic chemical than acetonitrile. DMacks (talk) 20:58, 5 August 2008 (UTC)[reply]

The answer may be YES - see http://holmfirthtypo.blogspot.com/2006/04/complete-history-of-typography-part-3.html 87.102.5.5 (talk) 21:54, 5 August 2008 (UTC)[reply]

Pigments that may test positive for “free cyanide” under some test protocols include CI Pigment Blue 27, CI Pigment Red 169, as well as PMTA green and violet pigments

I'll leave the rest to you.87.102.5.5 (talk) 21:55, 5 August 2008 (UTC)[reply]


In my completely unprofessional, non-warrantied opinion: Don't get too worked up about tiny doses of cyanide. Cyanide is everywhere; the body has ways to deal with it. It's very toxic in the sense that it doesn't take much to kill you, but in the sense of nasty things happening to you from doses that don't cause symptoms, I'd be much more worried about benzo(a)pyrene, which one of the links suggests is in the ink. --Trovatore (talk) 22:03, 5 August 2008 (UTC)[reply]

I really should emphasize here that I'm not at all sure that what I write above is true -- this is a vague impression not based on anything solid. If anyone actually knows something about it I'd be interested to hear. --Trovatore (talk) 22:51, 5 August 2008 (UTC)[reply]
Toxicity from newspaper ink has been related to Antimony -- whether it still is, don't know and the article doesn't say. Julia Rossi (talk) 02:09, 6 August 2008 (UTC)[reply]
I think that must be slight wrong - antimony can be toxic - but surely would only affect those that worked with it ie the printers/typsetters/typemakers - occupational hazards.
I doubt there is (or was) any risk from antimony to the average reader.
But I could well be wrong. —Preceding unsigned comment added by 87.102.5.5 (talk) 02:27, 6 August 2008 (UTC)[reply]
Not the reader, but in other ways, so not for wrapping fish and chips or using on the body because it can be metabolised. Julia Rossi (talk) 06:07, 6 August 2008 (UTC)[reply]
Well, here's what I meant about cyanide. If someone forced me to make a choice between taking 100 mg of cyanide, or 100 mg of benzo(a)pyrene, I'd take the latter. It would quite possibly (probably? certainly?) give me cancer at some later time, but the cyanide would be certain death immediately. On the other hand, if the choice were between 1 mg of cyanide or 1 mg of benzo(a)pyrene, based on my current state of knowledge and belief, I'd take the cyanide, expecting it to be essentially harmless, whereas the hydrocarbon could still give me cancer even in the much lower dose.
So what I'm kind of curious about is, am I right about that? Does anyone really know? --Trovatore (talk) 08:21, 6 August 2008 (UTC)[reply]
It seems you want to avoid cancer, but not as much as instant death - I think your analysis is correct - though of course nobody really knows how much an increased chance of cancer from eating 1mg of benzopyrene - but 1mg of cyanide is not lethal (Cyanide poisoning). Actually you quite a good chance of surviving 100mg of cyanide - but not guaranteed. It's a probability choice and science can only help so much.87.102.5.5 (talk) 10:29, 6 August 2008 (UTC)[reply]

Logical human thought

I have a computer program which reduces multiple state equations to minimum form. When I record sensor data into a table as states of an independent variable for each sensor concurrent with some event and use the program to create a rule from the data, in some cases the rule that is created is very easy for a human to comprehend. I consider this process, therefore a form of simulated [logical] human thought. Is there any published objection to defining this process in that way? —Preceding unsigned comment added by 71.100.162.249 (talk) 19:27, 5 August 2008 (UTC)[reply]

The Chinese room argument? --Bowlhover (talk) 21:21, 5 August 2008 (UTC)[reply]
All of what I have read and skimmed through so far suggests Searle knew nothing of the process of logical equation reduction or that the human mind can apply it to memory of past events to formulate rules. Thanks for the reference. The student I am dealing with is at that age where whatever is read is accepted on face value without question as to whether it is true. Thanks. —Preceding unsigned comment added by 71.100.162.249 (talk) 22:12, 5 August 2008 (UTC)[reply]
True, but Searle used speaking Chinese as a random example of a human activity that computers may eventually do following code. Speaking English or reducing equations would have worked just as well. --Bowlhover (talk) 22:25, 5 August 2008 (UTC)[reply]
But conversely Searle's argument makes a different point: What might a human do in such a situation to prove they are human and not computer? "For if we believe that Searle has proven that the man in his room (or more precisely, the system of the "rulebook-plus-man") doesn't understand Chinese, then Searle has also proven that a Chinese person doesn't understand Chinese. After all, no individual cell in a Chinese person's nervous system understands Chinese, and all of these cells obey the "blind" laws of biology (This argument doesn't work so well against dualists, but Searle, and many of his fans, claim not to be dualists.), just as surely as the man in Searle's room blindly follows the rulebook." -- A Refutation of John Searle's "Chinese Room Argument" by Bob Murphy. Murphy's refutation of Searle's argument applies regardless of the human activity. —Preceding unsigned comment added by 71.100.162.249 (talk) 06:31, 6 August 2008 (UTC)[reply]

electro etching of copper with copper suphate soln

can it be done? —Preceding unsigned comment added by 79.76.159.153 (talk) 20:50, 5 August 2008 (UTC)[reply]

With a current and an electrode most any conductive solution can be used to effect electro etching. Ferric hydrochloride, however, works without any current at all. —Preceding unsigned comment added by 71.100.162.249 (talk) 21:03, 5 August 2008 (UTC)[reply]

Rise time

What is the shortest mans rise time recorded? —Preceding unsigned comment added by 79.76.159.153 (talk) 21:24, 5 August 2008 (UTC)[reply]

Did you mean 'mains rise time' or 'shortest man's rise time'?87.102.5.5 (talk) 21:35, 5 August 2008 (UTC)[reply]
Haha, there are so many things this could mean! Is it about climbing something? Springing to one's feet? Getting an erection? We may never know... —Keenan Pepper 00:46, 6 August 2008 (UTC)[reply]
Even the earliest ever time the world's shortest man got out of bed...  ; )) Julia Rossi (talk) 01:48, 6 August 2008 (UTC)[reply]
Maybe the OT has drowned someone and fears early detection? --Ayacop (talk) 08:45, 6 August 2008 (UTC)[reply]
Mr. He Pingping claims [15] to be the world's "shortest man" at 2.4 feet (73 cm). How quickly could one get a rise out of him? Perhaps by pointing put that Mr. Lin Yik-chic is only 67.5 cm tall [16]. Edison (talk) 14:08, 6 August 2008 (UTC)[reply]

No no. I mean shortest time for man to get eerection —Preceding unsigned comment added by 79.76.167.200 (talk) 23:31, 6 August 2008 (UTC)[reply]

One of Kinsey's studies probably has the answer.[17]. Suntag (talk) 17:21, 7 August 2008 (UTC)[reply]

Question of utmost seriousness

Scenario: You take out a 500ml can of beer from the fridge where it has been sitting for the last 24 hours. You notice that its temperature is exactly +2 deg C. Unfortunately, because of the condensation on the can, it slips from your hand and falls a distance of exactly 1m to the floor. Miraculously, the can does not burst (although it may be dented), and all seems well. However, you quickly realize that if you try to open the can immediately, it will almost certainly mostly be lost in froth due to out gassing of the CO2.

Now the Q: If you were to return the can in question to the aforementioned fridge (with a temp of +2 deg C), how long would the can need to remain there before you could once again get it out and open it without any frothing and enjoy it at its best?

BTW its not homework-- I just done it ! :( —Preceding unsigned comment added by 79.76.159.153 (talk) 21:42, 5 August 2008 (UTC)[reply]

I'm guessing at this but I believe it is the same problem that requires cooling in the first place to remove the heat (in this case caused by the fall to the floor) it has acquired. Once the heat is lost there may be an additional time period for the molecules to forget their trauma and to go back to sleep since some of the energy from the fall was not turned into heat but into "nervous" molecules. I'd say give them another week. —Preceding unsigned comment added by 71.100.162.249 (talk) 22:20, 5 August 2008 (UTC)[reply]
A week? That's pessimistic! It's not really to do with heat or energy, it's just the dissolved gas coming out of solution, you need to give it time to dissolve again. I'm sure someone has done a detailed study on the problem, but in my experience about 20 minutes should do it. If it's been really shaken up, then maybe a little longer, but a simple fall wouldn't normally shake it up much. --Tango (talk) 22:54, 5 August 2008 (UTC)[reply]
I have spent many years pondering this issue. The first question is why shaking (agitating) the can makes it froth. The increase in temperature is a red herring (the increase in temp from dropping it is orders of magnitude less than the variability in the fridge). Frothing requires nucleating agents: in this case, this is small bits of dust etc that are present inthe liquid. Under calm circumstances, these particles adhere to the sides of the can; but if you shake or drop the can they become suspended and become much more effective nucleating agents: FROTH! Anyone got any comments? Robinh (talk) 07:15, 6 August 2008 (UTC)[reply]
Or, could it be that shaking it causes bubbles of air to get mixed into the liquid and those act as nucleation sites? --Tango (talk) 08:00, 6 August 2008 (UTC)[reply]
Both small bubbles and particulate matter can act as nucleation sites. Most commercial mass-market beers are filtered to remove the vast bulk of particulate matter. (Exceptions exist, of course. Trappist ales are perhaps the most dramatic example, though you'll only find those in glass bottles, not cans.) That leaves bubbles. Most will rise to the top of the beer after the can is allowed to stand undisturbed for a few minutes. I would also recommend flicking the can sharply a few times around its circumference with your finger; this will dislodge some of the bubbles which would otherwise remain stuck to the side of the can. TenOfAllTrades(talk) 13:17, 6 August 2008 (UTC)[reply]

From my experience, I'd think a few hours should be more than enough. --Kjoonlee 11:19, 6 August 2008 (UTC)[reply]

From my experience, holding your mouth over the opening as you pull the tab stops waste and saves time. DuncanHill (talk) 11:36, 6 August 2008 (UTC)[reply]

I don't have a clue how but tapping the beer (if in a can) several times on the top seems to prevent fizzing up by a large degree. I'd love to know the science to that, actually, but it always works for me even if I shake it up loads. —CyclonenimT@lk? 14:05, 6 August 2008 (UTC)[reply]

I used the "tap the can" method as the basis for a high-school science experiment. I did an exhaustive study with different types of taps, different lengths, different amounts of shaking (I even built a shaker to ensure all cans were shaken just as much) and I had controls that were not shaken, but sat there for the same length of time as those getting tapped. Just out of a pure guess, I also used a "turn the can upside down" think my grandmother used. I measured spray by opening the cans in a small fishtank with a lid (I cut holes in the plastic lid and used rubber gloves - so I could trap all the spray). In the end, I found that tapping the can for about 5 seconds stopped most of the spray. Also, turning the can upside down slowly and then upright again had the same effect. Finally, leaving the can sitting there for about 5 seconds had an equivalent effect. So, my conclusion was that it takes about 5 seconds for much of the fizzing to cease inside the can. If you like, you can spin around three times on one foot while singing Old McDonald. What you do during those few seconds isn't the key. It is merely the length of time. In the end - I got second place. I'm still a bit pissed about it. I was told that I got second place because the science teacher disagreed with my conclusion. She was certain that tapping the can had some sort of physical effect on causing the bubbles inside to burst and reduce fizzing. -- kainaw 17:20, 6 August 2008 (UTC)[reply]
Ahahahaha! Welcome to the world of research :) Now you just need to replicate your results, publish your research, wait for (or encourage) others to independently replicate your results, and then you're home dry. But your teacher still probably won't believe you for another 20 years. 79.66.38.215 (talk) 23:25, 7 August 2008 (UTC)[reply]

OK AS the OP, i have to say that none of the above time estimates seem to concur with my experience. I have found that if I drop a can of lager from 1m or less, I have to leave it in the fridge for a considerable number of hours (may be 5 hours) before I can open it without frothing. Is lager different from other beers in this respect? —Preceding unsigned comment added by 79.76.225.183 (talk) 01:00, 8 August 2008 (UTC)[reply]

Harmful bacteria

I heard it said recently that the human mouth contains more harmful bacteria then the rectum or colon. Is that true or false? —Preceding unsigned comment added by 79.76.159.153 (talk) 23:16, 5 August 2008 (UTC)[reply]

Define harmful. There are normally orders-of-magnitude more bacteria near the exit of your alimentary canal than near the entrance, but I do not know how you would like to determine which are harmful and which are not. You would not want any of them in your blood stream, that's for sure. It depends on your immune system just how much harm any of them would do when given the opportunity. --Dr Dima (talk) 00:18, 6 August 2008 (UTC)[reply]
Well, oral bacteria in the bloodstream can possibly cause myocardial infarction. How much shit would you need to eat cause that? —Preceding unsigned comment added by 79.76.159.153 (talk) 00:31, 6 August 2008
Eating something does not automatically put it into your bloodstream. For a fair comparison you'd need to be injecting it. Probably a bad idea. APL (talk) 02:05, 6 August 2008 (UTC)[reply]
I'm not sure. I've been told that there are more bacteria per square centimeter on one's tongue and teeth than anywhere else in a healthy human body. Maybe that's totally wrong, but it is widely circulating idea at least. Dragons flight (talk) 00:29, 6 August 2008 (UTC)[reply]
It all depends whether you mean "(more harmful) bacteria" (i.e. bacteria that are more harmful than others), or "more (harmful bacteria)" (i.e. a greater number of bacteria that are harmful at all, even if only slightly). —Keenan Pepper 00:42, 6 August 2008 (UTC)[reply]
To 79.76: How much shit you would need to eat depends on how deep a shit you are in right now. Please use more appropriate terminology next time. To other readers: you may want to read Gut flora article. It clearly states the usual numbers of bacteria in human gut, as well as the more common genera of bacteria found normally in the gut. Clostridium, in particular, is a well known human pathogen, and many others are opportunistic pathogens. So, colon wins! --Dr Dima (talk) 01:13, 6 August 2008 (UTC)[reply]
Clostridium species, especially the dangerous ones like C. botulinum and C. perfringens, are pretty much everywhere in nature, though. They aren't harmful without special circumstances, though in those situations they're quite lethal. SDY (talk) 02:04, 6 August 2008 (UTC)[reply]

The gut flora article is missing all the plant viruses see doi:10.1371/journal.pbio.0040015 . And probably bacteriophages as well. --Ayacop (talk) 08:39, 6 August 2008 (UTC)[reply]

August 6

Shoe polish and solubility

Are there any household liquids that can dissolve shoe polish? It's obviously not very soluble in water. SDY (talk) 01:49, 6 August 2008 (UTC)[reply]

Warm water with detergent works just fine. If that didn't help for some reason, try petroleum ether. --Dr Dima (talk) 01:59, 6 August 2008 (UTC)[reply]
Hi, in the article Shoe polish there's this[18]. Being made up of waxes and oils, then a solvent for these would be turps as a cleaner for oils. Another solvent is Kerosine. Julia Rossi (talk) 02:01, 6 August 2008 (UTC)[reply]
Swarfega is fine for removing polish from the hands. Algebraist 12:58, 6 August 2008 (UTC)[reply]
I don't know bout household liquids, perhaps fingernail polish remover? I keep a very inexpensive product called 'Goop' hand cleaner. It was originally used by auto mechanics, but I use it after oil painting. Also, it's great for grease stains if you haven't washed the clothing yet.Kissnmakeup (talk) 04:14, 7 August 2008 (UTC)[reply]

Recycling plastic

Recently read about plastic refuse not only creating an island of junk the size of Texas or New South Wales but breaking down to particles the size of plankton without the eco benefits. I forget the name for them, but they end up eaten, then as part of the food that ends up on our plates. Given this process, rather than just landfill or ocean jetsam, is there an better option to crumble and mix plastics back into semi-permanent materials such as concrete, or road surfaces, or building foundations? Is anyone doing this already? Julia Rossi (talk) 01:53, 6 August 2008 (UTC)[reply]

To some extent. Did you check out plastic recycling?--Shantavira|feed me 06:43, 6 August 2008 (UTC)[reply]
With today's and tomorrow's energy costs, also incineration with energy recovery is one of several waste-to-energy (WtE) technologies. In summary, plastics are like biomass, you can reuse it or burn it. --Ayacop (talk) 08:32, 6 August 2008 (UTC)[reply]
Do you mean the Great Pacific Garbage Patch? Much of it is in tiny pieces, not yet plankton size, but too small to strain back out. Franamax (talk) 09:05, 7 August 2008 (UTC)[reply]
In what I read, it mentioned that as well as the synthetic plankton stuff. Maybe someone will sell real estate on the patch and it will just sail around on the currents something like a floating Howl's castle. I was more interested in what's being done in the gap between the big stuff that's already out there, the stuff that's being recycled up to a point, and the stuff that slips through the system and simply distintegrates without going away. Julia Rossi (talk) 11:42, 7 August 2008 (UTC)[reply]

Gasoline Expiration Date?

I own an automobile that hasn't been driven since 2001. It has 80% of a tank of gas left in it from the last time I filled it up, when gas was somewhere around $1.30 a gallon. Now, I'm wondering if I could siphon it to another gas tank and it work just like new, $4 per gallon gasoline. Thanks. Danthemankhan 03:51, 6 August 2008 (UTC)[reply]

According to Gasoline#Stability: "When gasoline is left for a period of time, gums and varnishes may build up and precipitate in the gasoline, causing "stale fuel." This will cause gums to build up in the fuel tank, lines, and carburetor or fuel injection components making it harder to start the engine. Motor gasoline may be stored up to 60 days in an approved container. If it is to be stored for a longer period of time, a fuel stabilizer may be used."
There are many other Internet users with similar questions. Here is an anecdotal account in which 18-month-old cans of gasoline still powered a car perfectly. Also see these Yahoo! Answers responses. The Straight Dope has this article claiming gasoline has a shelf life of months to years, and degradation is due to oxidation, evaporation, and contamination by water. --Bowlhover (talk) 05:00, 6 August 2008 (UTC)[reply]
There are several microorganisms living on fuel, the German language WP has de:Kerosinpilz (kerosene fungus, Amorphotheca resinae, also growing in diesel) and a short mention exists in microbial corrosion#Aviation fuel about some other bacteria. Both types, however, need a certain percentage of water, so they appear only at specific places in the tank, where water condenses or collects on the inside or within the fuel at the bottom. Kerosene fungus is a big problem for the aviation industry. Probably much of what the links above claim as degradation is due to microorganisms. --Ayacop (talk) 08:15, 6 August 2008 (UTC)[reply]
There's also the problem of exactly when you filled it up. If you live somewhere with major seasonal temperature variation, then the winter fuel might not be suitable for summer use. Algebraist 12:55, 6 August 2008 (UTC)[reply]
It is important to know if this is diesel or unleaded. Diesel is considered to have a much shorter useful life than gasoline as diesel has problem with bacteria. Using winter gas in summer might cause vapor lock - but the extra higher volatility compounds may have already evaporated, effectively turning a winter mix into a summer one or worse. Rmhermen (talk) 14:19, 6 August 2008 (UTC)[reply]

I would also have reservations because gasoline is made up of many different components. Some of the hydrocarbons in there are light and could well have evaporated by now, with the heavier ones sticking around. This will make combustion very difficult and could cause the gumming up issues mentioned earlier. In short, as expensive as gasoline is now, given the risk and the conflicting information on the matter, safe is better than sorry. EagleFalconn (talk) 19:08, 6 August 2008 (UTC)[reply]

If you decide to throw it away, don't just pour it on the ground; although its properties of combustion may have degraded, it's still plenty potent enough to contaminate ground water.
Now, I'm sure you knew that, and the only reason I wrote it is to provide another alternative. Siphon it out, if you wish, and dilute it with newer gas, 5:1 or maybe 10:1. OR, use it in a lawnmower -- the gumming-up-the-innards stuff has already been done, and lawnmowers are ridiculously ignorant of octane and fuel quality.
You still have to deal with any potential damage done by the old product in the old vehicle. For that, I strongly recommend replacing the old fuel with new as soon as possible after emptying the tank, the goal being to minimize "drying out" the internal parts. If the car starts, it will probably not run well for a while, but the only way it's going to improve (in the short term) is to run more fuel through it
HTH! --Danh, 63.231.153.235 (talk) 21:10, 6 August 2008 (UTC)[reply]

Genital herpes Outbreaks

I dont have herpes, but was wondering that if someone does, and they take their medication everyday, do they ever have an outbreak again? —Preceding unsigned comment added by 76.168.89.122 (talk) 04:52, 6 August 2008 (UTC)[reply]

You might like to plod through the treatment section of the Herpes article or try google. Julia Rossi (talk) 05:21, 6 August 2008 (UTC)[reply]
(ec) There is more than one strain of the herpes virus, but immunity to even one of them will make a second herpes infection very unlikely (see http://chealth.canoe.ca/channel_section_details.asp?text_id=1370&channel_id=16&relation_id=27918). Infection is still possible as the 1980 study showed. --Bowlhover (talk) 05:27, 6 August 2008 (UTC)[reply]
By my understanding, it's common for a herpes infection to never be completely eradicated and just to lay dormant and then break out again later. That would suggest getting an infection doesn't yield immunity for the future. --Tango (talk) 05:35, 6 August 2008 (UTC)[reply]
Or as Demetri Martin would say - Glitter is the herpes of craft supplies... You think you've got rid it of it then go out into the sun then flare-up. 194.221.133.226 (talk) 08:00, 6 August 2008 (UTC)[reply]

Windowboxes

moved from the misc desk for better information Julia Rossi (talk) 10:03, 6 August 2008 (UTC)[reply]
Which are the best flower plants to use in windowboxes that are in full sun?TerryBohle (talk) 08:03, 6 August 2008 (UTC)[reply]

Now where were we, have you looked at cactus, geraniums, succulents? Julia Rossi (talk) 10:06, 6 August 2008 (UTC)[reply]
Wallflowers could work. DuncanHill (talk) 10:11, 6 August 2008 (UTC)[reply]
This site [19] suggests herbs and vegetables. DuncanHill (talk) 10:20, 6 August 2008 (UTC)[reply]
Other suggestions include miniature roses, dwarf gladioli, sunflowers, and daisies. DuncanHill (talk) 10:22, 6 August 2008 (UTC)[reply]

how to submit a picture with an question?

i need to submit a picture for detail of a question in help desk. how to do it ? —Preceding unsigned comment added by Shamiul (talkcontribs) 10:04, 6 August 2008 (UTC)[reply]

You scroll to the top of this page, click on Help desk on the right hand side under See also, and ask there for how to use wikipedia. Julia Rossi (talk) 10:34, 6 August 2008 (UTC)[reply]
Click on "Upload file" on the left. --Bowlhover (talk) 10:56, 6 August 2008 (UTC)[reply]
This assumes that the pic isn't already on the Internet. If it is, use a link like this: [20]. You can do the same thing for an image you've uploaded. It's also possible to have the pic display directly on this page, instead of by picking a link, but that's generally discouraged, because it slows down the page loading speed to a crawl. StuRat (talk) 13:33, 6 August 2008 (UTC)[reply]

Buoyancy on the ocean floor

If I take a cylinder (with its axis vertical) and immerse it in water, it will experience an upwards force due to buoyancy. As per the article, this is cause by the difference in pressure acting on the top and bottom surfaces of the cylinder (there are also pressures on the vertical walls of the cylinder, but these cancel out, and in any case they only act normal to the surface, ie in a horizontal direction). If I now push the cylinder all the way to the bottom of the body of water (eg the ocean floor), so that it is in contact with the (assumed to be smooth and flat) surface of the ocean floor, will the buoyancy force suddenly reverse direction and point downward?

Experience says no, but from the way I understand buoyancy, if the cylinder is sitting on the ocean floor, there is no more water underneath it exerting an upwards pressure force, so the only pressure forces acting on it are on its top and its sides. Since the sides are vertical and pressure acts normal to the surface, these forces act horizontally, leaving only a downward pressure force from the water on the cylinder's top surface. Therefore, the cylinder on the ocean floor has both its own weight and water pressure pushing it down, hence no more buoyancy, which would mean that even a light, air-filled cylinder would stay on the ocean floor. Where have I gone wrong? — QuantumEleven 12:24, 6 August 2008 (UTC)[reply]

It'd still be less dense than the water it displaces; thus it will still rise a little. --Kjoonlee 12:34, 6 August 2008 (UTC)[reply]
If you create a water-tight seal between the cylinder and the sea-floor and pump out any water in between them, then yes, the cylinder will be held on the sea floor by water pressure - you have, in effect, created an underwater suction cup. Gandalf61 (talk) 12:39, 6 August 2008 (UTC)[reply]
That's true, but it's not what Kjoonlee was asking. If the object is just resting on the ocean floor, and there's no seal (and partial vacuum) to make a suction cup, then it will rise off the floor and water will rush in under it. The reason is that the solid material that forms the ocean floor is also under pressure (the same pressure as the water touching it, because they are in mechanical equilibrium), so the floor itself exerts an upwards force on the object. 128.165.101.105 (talk) 14:03, 6 August 2008 (UTC)[reply]
I'm not sure that's true at all. You're assuming a semi-fluid sea floor. But the cylinder would still float even if it landed on solid rock. The reason is that the cylinder without the seal floats upwards because there is pressurized water underneath it, even if only a tiny bit. APL (talk) 16:34, 6 August 2008 (UTC)[reply]
Kjoonlee, You've just described exactly how suction cups work. The obvious practical problem is making sure that fluid does not get between the sea floor and your cylinder, which is why suction cups are typically made of rubber so that they can deform and create a seal with the surface they're sticking to. APL (talk) 16:34, 6 August 2008 (UTC)[reply]

I think Gandalf is right. In highly skilled metalworking, such as tool and die making, they use Johannsen (spelling?) blocks. These are metal blocks that have an exceedingly smooth and flat surface. If one surface of two of these blocks is made very clean and free of grit, and the two surfaces are then pressed together and rubbed together a bit, it will squeeze out all air between the blocks. One block can then hang below the other solely because of air pressure. The blocks are not magnetized.66.52.8.128 (talk) 16:58, 6 August 2008 (UTC)[reply]

That sounds like a cold weld. StuRat (talk) 17:06, 6 August 2008 (UTC)[reply]

Maybe I'm missing something here, and correct me if I'm wrong, but I think I'm detecting a slight error in logic. Assume the cylinder is less dense than water. If the cylinder is submerged in the middle of the water, the net force due to the pressure of the water existing is going to be zero because the water pressure will be equal in all directions (assuming still water of homogenous temperature etc etc). The model you guys are using for buoyancy only works at the surface. When submerged, the cylinder rises because of a difference in density. If it is more dense than water, it sinks. If it is less dense, it floats. As buoyancy explains (slightly poorly) the issue is weight difference manifesting as density difference. Because an object displaces a volume of water equal to its own volume, if the weight of the object is less than the weight of the water it displaces the water flowing downward decreases the overall potential energy of the system more. Therefore, water goes down, cylinder goes up. In the case where the (perfectly smooth) cylinder is on the (perfectly smooth) bottom, the cylinder will not rise because of the effect that 66.* mentioned. There is no path for the surrounding water to be under the cylinder to push up on it, so it stays put. If neither the cylinder or the bottom are perfectly smooth, water has access to the area underneath so it can push up. (Clarification: Collisions are pushing force only). EagleFalconn (talk) 19:21, 6 August 2008 (UTC)[reply]

The second part of your explanation is fine, but the first part is incorrect. When the cylinder is in mid-water the net force due to water pressure is not zero. It is equal in magnitude to the weight of an equivalent volume of water at that depth, but opposite in direction i.e. upwards - this has to be so, because the water pressure must be sufficient to support an equivalent volume of water in equilibrium. The net upwards pressure force arises because the water pressure at the bottom of the cylinder is greater than the water pressure at the top because pressure increases with depth. And this upwards pressure force is what we call buoyancy. This is explained in the Forces and equilibrium section of our buoyancy article. Gandalf61 (talk) 19:42, 6 August 2008 (UTC)[reply]
My mistake, let me rephrase. I did not mean net force due to water pressure. I meant the net force on the cylinder due to the water molecules colliding with the cylinder. A subtle difference, but my point was that the water colliding with the cylinder has nothing to do with the motion up or down, and that the motion was due entirely to the weight displacement of the water. EagleFalconn (talk) 14:22, 7 August 2008 (UTC)[reply]
Water pressure is due to water molecules colliding with the cylinder - these are just macroscopic and microscopic descriptions of the same phenomenon - see this section of the kinetic theory article. Water molecules colliding is pressure; pressure difference between bottom and top surfaces gives rise to net upward force that we call buoyancy; buoyancy force equals weight of equivalent volume of water. The displacement of the water has nothing to do with buoyancy - when you pour water from a jug into a glass you displace the water, but you don't create mysterious buoyancy forces. Gandalf61 (talk) 14:44, 7 August 2008 (UTC)[reply]

Let us suppose that the cylinder is a cylinder of water enclosed by an infinitely thin weightless skin. Even though the pressure of the surrounding water is greater at the bottom, the cylinder will not rise because its weight is such that the total pressure exerted by the cylinder on the water beneath it is equal to the pressure on it. The difference in pressure between the water under the cylinder and the bottom of the cylinder occurs when the cylinder weighs less than water. Then the total weight of the cylinder and the water above it is not the same as the weight of the water beside it. The difference in pressure is not just caused by the difference in pressure of the water between the top and bottom of the cylinder. It is also caused by the difference in the weight of the cylinder and an equal amount of water. Even when the cylinder sits on the bottom of the body of water, it weighs less than water. The pressure of the bottom of the cylinder on the bottom of the body of water will not be the same as the pressure of the water around it on the bottom of the body of water because it doesn't weigh as much, therefore the weight of the the cylinder and the water above the cylinder is not the same as the weight of the water beside it, regardless of where it is. So picture a free body diagram of the cylinder. The normal force of the bottom on the cylinder, the weight of the cylinder and the weight of all the water above the cylinder are acting on the cylinder. Are there any other forces in the vertical direction? Are these forces equal but opposite? I believe that if I took a wooden block in freshman physics lab and placed it on the bottom of a beaker full of water, its gonna float to the top because it's lighter than the water that it's in. If I take an ice cube and put it at the bottom of a glass of water, it might stick to the bottom of the glass because of dangling surface bonds, but I think it would float to the top. What do you guys think?Kissnmakeup (talk) 03:58, 7 August 2008 (UTC)[reply]

The problem with the idea that buoyancy force = weight of displaced water regardless of whether the object is on the bottom or not is that you then have to invoke some sort of negative pressure theory to explain the action of a suction cup, which will remain on the bottom despite weighing less than an equivalent volume of water. Returning to the block in a glass example, if the bottom of the block and the glass were sufficiently smooth to exclude water molecules from their interface (not going to be possible with a wooden block, but maybe possible with a plastic block ??) then, yes, the block would stay on the bottom, held there by its weight and the unopposed water pressure on its top surface. Our intuition is at fault here because our experience comes from objects that are not sufficiently smooth to observe this phenomenon. Gandalf61 (talk) 09:28, 7 August 2008 (UTC)[reply]
Yes, after further consideration, I agree. By sufficiently smooth, what you mean is that there cannot be even as much as the width of a water molecule of space between the surfaces of the container and cylinder, because if there is even that much space, the pressure will force the water up under the cylinder and there you will have it, via Pascal's Principle, buoyancy.Kissnmakeup (talk) 11:00, 7 August 2008 (UTC)[reply]

Epileptology and Neurophysiology

A quick question about titles in the UK. Are the above 2 titles (Epileptologist and neurophysiologist) restricted to Medical Doctors only, or can a Clinical Physiologist (Neuro) call themselves either of those. Obviously they'd have to have further study in epilepsy, but if they had a masters in epileptology, would they be allowed to call themselves an epileptologist? Thanks 92.4.122.75 (talk) 14:33, 6 August 2008 (UTC)[reply]

I don't know if epileptology is a term regulated or licensed by a medical board, but it probably varies by region. Nimur (talk) 17:10, 6 August 2008 (UTC)[reply]

Name for when skin hurts

I've heard about something where someone's skin hurts as though it had a sunburn underneath the skin. First it starts around the chest. Each day, it is in a lower position and moves from around the chest, to around the stomach, to around the groin area, then down one or both legs. There are no visible marks, but there is tingling pain. What might something like that be called and does Wikipedia have any articles related to it? Thanks. Suntag (talk) 17:33, 6 August 2008 (UTC)[reply]

It may be an unusual form of paraesthesia. Fribbler (talk) 21:56, 6 August 2008 (UTC)[reply]
For me, when my skin hurts, that's a sure sign I'll be hit with severe flu symptoms a few hours from the time I notice the pain in my skin. Not sure about your localized moving patch, though. ~Amatulić (talk) 00:34, 7 August 2008 (UTC)[reply]
There is also a rare genetic condition which causes extreme sensitivity to touch on the front of the torso (as well as pronounced curvature to the shoulder blades). I remember the symptoms, but I have no recall of the name. Tried googling, but found nothing. -- kainaw 02:22, 7 August 2008 (UTC)[reply]
  • I've read that the skin pain caused by herpes zoster moves over time as the old chicken pox virus makes its way along the nerve endings around a rib and near the skin surface. However, that is a horizontal movement rather than an up and down movement. Per the article, herpes zoster could result in residual nerve pain, but it doesn't say anything about the residual pain moving in any direction. Suntag (talk) 16:37, 7 August 2008 (UTC)[reply]
If I had a recurring pain that seemed to be progressing through my body, I would go to see a doctor and explain my problem, since they would be far better able to help than random people on the internet. If I couldn't see a doctor for some reason, I would visit a pharmacist as pharmacists are also trained and experienced in these things and able to give an idea of whether it is likely to be serious. 79.66.38.215 (talk) 23:18, 7 August 2008 (UTC)[reply]
I doubt any pharmacist with his head on right would give you any kind of diagnosis or anything other than some over the counter crap that will do nothing to cure a serious problem. A doctor would provide a credible diagnosis.--El aprendelenguas (talk) 00:20, 8 August 2008 (UTC)[reply]
Pharmacists are not trained in medical diagnosis. And certainly no licensed pharmacist in the US would provide medical advice except on lotions and potions he's pushing. However, to answer Suntag's question, I have no clue, short of a full medical exam. Zoster does not match your symptoms, but that's about all I'll say. This is not the place to get a diagnosis. OrangeMarlin Talk• Contributions 04:26, 8 August 2008 (UTC)[reply]

Dental membranes?

Can anyone explain to me what dental membranes are? I understand that their purpose is to help regenerate gum tissue, but what are they exactly? What are they composed of? Is it man-made or does it contain animal parts? Are there any similar membranes used on the human body? —Preceding unsigned comment added by Cap'n Louise Redbeard (talkcontribs) 22:22, 6 August 2008 (UTC)[reply]

Guided bone regeneration might help. Julia Rossi (talk) 03:45, 8 August 2008 (UTC)[reply]

Heat rub on the testicles?

I was told today by someone who used to be in the army that it was a common practice for soldiers to smear heat rub (Deep Heat or Tiger Balm type stuff) on their testicles before embarking on long marches in cold environments, so that they wouldn't feel the chill as much. I'm pretty sure that this he was just trying me out to see if I would be dumb enough to actually do it (I'm not going to, no way!) so that he and his buddies could have a good laugh at my expense, but just to clarify -- would this actually work as stated? Or would you just keel over in burning agony after a couple of minutes? --84.70.255.204 (talk) 22:44, 6 August 2008 (UTC)[reply]

Well (sorry I cant resist it), it's possible that the person who told you was talking complete bollocks! Sorry ;( —Preceding unsigned comment added by 79.76.167.200 (talk) 23:26, 6 August 2008 (UTC)[reply]
You would keel over in burning agony after a few seconds. The Menthol and Methyl Salicylate will cause a very bad burning sensation due to the large number of nerves in the general area. Q T C 23:30, 6 August 2008 (UTC)[reply]
So would that prevent frostbite? —Preceding unsigned comment added by 79.76.167.200 (talk) 23:33, 6 August 2008 (UTC)[reply]
I doubt it. Stimulating nerve endings to feel a burn isn't the same as actually warming up flesh. There may be a small benefit through increased blood circulation that may prevent freezing. ~Amatulić (talk) 00:36, 7 August 2008 (UTC)[reply]
So, could this be the same nonsense that says: Have another whiskey to warm you up? When it doesn't at all really? —Preceding unsigned comment added by 79.76.167.200 (talk) 01:10, 7 August 2008 (UTC)[reply]
Whiskey does warm up your skin, by increasing blood flow from the warm core of your body to the cool skin. The problem is that if the room isn't warmer than your skin temperature, your core body temperature will fall. --Carnildo (talk) 20:15, 7 August 2008 (UTC)[reply]
One reads from time to time about mountaineers or cold climate travellers having suffered frostbite of the fingers or toes. Never in my life (fairly long) have I heard of frost bite of the testicles, or indeed associated anatomy. Richard Avery (talk) 07:25, 7 August 2008 (UTC)[reply]
Well that is hardly surprising is it? would you go out in cold weather with your testicles hanging out of your trousers or would you keep them snugly cossetted close to your body inside your nice warm underpants/ I know which Id choose. —Preceding unsigned comment added by 79.76.225.183 (talk) 00:43, 8 August 2008 (UTC)[reply]
Even testicles themselves don't go outside in the cold[21]. Julia Rossi (talk) 03:42, 8 August 2008 (UTC)[reply]

Please identify this spider

File:Dsc 4670.jpg
Mystery spider. Normally has eight legs; a few have been lost due to an unfortunate traumatic experience.

I have a bit of a spider problem. I believe I know what this spider is, as I've seen them plenty of times back home (Georgia), however nobody here (North Carolina) seems to believe they live in this area. This includes the exterminators, which puts me in a bit of a pickle. I've had five of the specimens I caught and preserved in alcohol taken by the exterminator and supposedly shipped to Atlanta for identification. They claim that Atlanta couldn't identify my spiders. Now the spiders have supposedly been shipped to Raleigh so whomever is in Raleigh can take a crack at 'em.

I find all this pretty darn silly since I'm nearly certain of what these spiders are after a few minutes of inspection with a magnifying glass. However, I'm not an entomologist, so I need other opinions (I may end up sending some of my other samples to the department of agriculture).

Just for kicks, what do you think this spider is? All of the spiders I've caught look just like this one (except with more legs), and vary in size from roughly 4 mm to 20 mm (leg span). If you happen to have studied entomology, please do mention it since that would give me a greater level of confidence. I won't tell you what I think the spider is because I'm hoping someone will provide a feasible alternative to my amateur identification.

Thanks. -- mattb 22:49, 6 August 2008 (UTC)[reply]

Does your university have a zoology department? They might enjoy a challenge. DuncanHill (talk) 22:53, 6 August 2008 (UTC)[reply]
Eh... UNC-G and Wake Forest University both have biology departments, but I can't tell whether they have anyone who specializes in entomology in particular. My best bet is probably NCSU in Raleigh, but me being in Greensboro that's a rather long drive. -- mattb 23:15, 6 August 2008 (UTC)[reply]
I assume you're afraid it's a brown recluse. It might be, but (a) Greensboro is well outside the brown recluse's range; (b) I don't think it's as easy to identify brown recluses as our article suggests; and (c) they're not as dangerous as they're often made out to be. The information here might be helpful. (Incidentally, you technically want an arachnologist and not an entomologist, although UC Riverside seems to lump them together.) -- BenRG (talk) 23:36, 6 August 2008 (UTC)[reply]
And I should add that I know zilch about spiders, I just happen to have been reading about this subject a few days ago. -- BenRG (talk) 23:41, 6 August 2008 (UTC)[reply]
I can only add, that it does resemble a brown recluse (-4 legs) unless it's a close relation to it. Spiders can migrate through soil importing, or other vehicles of transport (building materials, climbing into cars etc), so you never know. There could be reasons why they're turning up elsewhere. Out of interest, what do the North Carolignians call them? Julia Rossi (talk) 23:48, 6 August 2008 (UTC)[reply]
I'm not really afraid that IT is a brown recluse so much as I'm afraid that all fifteen or so of the identical-looking spiders I've caught are brown recluses. Far more than that I'm afraid that the spiders I have not seen or caught are brown recluses, especially since this little guy sustained its damages due to being violently swatted off my face in the middle of the night. Anyway, I've done my home work already and I'm more interested in knowing whether anyone can suggest another spider that this might possibly be. -- mattb 00:43, 7 August 2008 (UTC)[reply]

Or maybe you discovered a new species of spider. Just throwing it out there. --harej 23:49, 6 August 2008 (UTC)[reply]

A photo taken looking straight down at the body, and in daylight, would improve the accuracy of identification. It has the fiddle marking. Also, a macro closeup of the eyes would be diagnostic, since recluses have a different pattern of eye arrangement than most other spiders. They do indeed travel around outside their usual habitat, hitching rides in any old item like a box of junk, to a remote location. They seem to be getting a grand rehabilitation these days, with claims they are not very dangerous. I know 3 people who had bites from them, proven by the spider carcass being preserved, and who had a slow recvery with a depressed scar after the nasty result of the bite. They love to live in houses, and are aggressive about climbing into shoes and even into beds, then biting when we unwittingly come into contact. Yuck. Edison (talk) 18:38, 7 August 2008 (UTC)[reply]

I have no adequate macro lens, and this is quite a small spider anyway. Still, you should be able to see the eye arrangement from the full-size photo (I can). Nevertheless, I'll see whether I can take a better photo of one of the larger ones today.
I'm certainly no arachnophobe, but I'm equally unconvinced by claims of this spider being mostly benign. I too have a friend who was bitten by one and who lost a tangerine-sized chunk of flesh as a result. Honestly I'd rather be bitten by a black widow than one of these. Even if less than 10% of bites result in necrotic wounds, I don't like playing the odds after having one crawl on my face. -- mattb 19:32, 7 August 2008 (UTC)[reply]

August 7

Please try to disprove this theory

In the Alaskan Psychic Newsletter recently there was an article stating that George W Bush was an alien from 20 million light years away implanted here on our earth to cause havoc. How can this theory be disproved? —Preceding unsigned comment added by 79.76.167.200 (talk) 00:06, 7 August 2008 (UTC)[reply]

Maybe if he starts behaving hisself? Julia Rossi (talk) 00:17, 7 August 2008 (UTC)[reply]
It can't. Essentially you're saying, "How do I prove that Bush is not an alien?" (Or maybe, a humanoid Roomba belonging to an alien.) That doesn't mean the article is correct; it just means we have no credible evidence of an extraterrestrial civilization, especially one that would travel 20 million light years to swap babies with George H. W. and Barbara Bush. They've known George W. for quite some time, and don't really seem the Jonathan and Martha Kent sort. Those writing the article either knew it to be complete codswallop, or believe in; in the former case, they're not going to admit the joke, and in the latter, you can't get through the tinfoil.
I could as easily write an article claiming that Vladimir Putin is the illegitimate child of Elizabeth I and Rasputin, conceived as a result of time-travel technology that was hidden in the temple of Hatshepsut until brought to Paris by Freemasons, only later to be pocketed by Peter the Great after his visit to Rheims (and his reading of Anna Yaroslavna's prayer book). Equally difficult to disprove (though possibly more entertaining). OtherDave (talk) 00:24, 7 August 2008 (UTC)[reply]
To idly quote those who are wiser than I am: "Logic cannot remove what logic did not put there." SDY (talk) 00:36, 7 August 2008 (UTC)[reply]
Surely that is most illogical, Captain: after all I am a Vulcan. You're a mere human! —Preceding unsigned comment added by 79.76.167.200 (talk) 01:14, 7 August 2008 (UTC)[reply]
You can't falsify something unless it makes testable predictions. If the aliens in question are bug-eyed, for example, then this claim is falsified by a large amount of eyewitness evidence. Without that kind of detail you can't prove the claim wrong, but it also fails to qualify as a scientific hypothesis (or theory). It's no good to say that the aliens have the technology to hide their evil plot from us; that's still an unpredictive statement. -- BenRG (talk) 01:38, 7 August 2008 (UTC)[reply]
When superstitious theories are raised by those who are serious, skeptics often use Occam's razor. For jokes like this one, on the other hand, even Occam would groan. --Bowlhover (talk) 03:06, 7 August 2008 (UTC)[reply]

I might be opening up a can of worms, but I have a follow-up question. We know that this ridiculous theory can't be disproved. But isn't it also fair to say George W. Bush isn't an alien? Similarly, isn't also fair to say that god doesn't exist for the same exact reason? ScienceApe (talk) 03:40, 7 August 2008 (UTC)[reply]

It would be fair to say "we have no reason to believe that George W. Bush is an alien." which is typically as close as science ever gets to these sorts of unprovables. APL (talk) 06:46, 7 August 2008 (UTC)[reply]
What I was disproving was a theory that included "to cause havoc". Without that item, it is impossible to disprove. With that item, it would be "disproved" if he indeed stopped what he's doing and say, made daisy chains instead, because logic does depend on what is put there (or not) -- so there y' go... Julia Rossi (talk) 07:01, 7 August 2008 (UTC)[reply]
For a second there I thought you had it Julia, but no. If he was sent to cause havoc and didn't accomplish that, it would only mean that he was incompetent as a from-20 million light year-away alien. Franamax (talk) 08:07, 7 August 2008 (UTC)[reply]
Also, how would you know the aliens didn't order him to stop wreaking havoc? --Bowlhover (talk) 08:19, 7 August 2008 (UTC)[reply]
I can see only two possibilities. Alien GWB fails at causing havoc, terrestrial GWB causes havoc. Either way, EPIC FAIL. /rant Franamax (talk) 10:07, 7 August 2008 (UTC)>[reply]
Actually, I think we can logically disprove this theory. Start from the assumption that there is a lone alien who (a) can flawlessly impersonate a human being; (b) can manipulate human affairs to insert themselves into any desired position of power; (c) is rationally pursuing an objective of causing as much havoc on Earth as possible. Given this, the alien would insert themselves into a position with as much individual power as possible. The head of state or head of government in a democracy has relatively little personal power, so the position of President of the United States is not a rational target for our hypothetical alien. On the other hand, the leader of a military dictatorship in a country with a nuclear capability has a much greater potential for causing havoc ... Gandalf61 (talk) 09:09, 7 August 2008 (UTC)[reply]
You're assuming that the alien is omniscient, which may not be the case. --Bowlhover (talk) 09:33, 7 August 2008 (UTC)[reply]
Well, let's at least assume they have access to a comprehensive electronic encyclopedia of human knowledge - think I've seen one around here somewhere ... Gandalf61 (talk) 09:45, 7 August 2008 (UTC)[reply]
Ahem, you've seen the ceeohk, but have you seen the alien? Let's not assume the alien is godlike, but only acts that way, and who says he looks flawlessly human? and if he was incompetent might that be our secret weapon agaiinst invasion then? Logical presups are proliferatiing like nuclear armaments here... Julia Rossi (talk) 10:07, 7 August 2008 (UTC)[reply]
See wishful thinking --Shaggorama (talk) 17:32, 7 August 2008 (UTC)[reply]

The disproof is that there is no evidence space aliens could be that cruel. Edison (talk) 18:31, 7 August 2008 (UTC)[reply]

Will going faster than the speed of light create a black hole?

I was having a conversation about the speed of light with someone I knew and he said that according to theory, if you go faster than the speed of light, it will create a black hole. Can someone please explain this theory to me, and if this is even true. Monkeynator03 (talk) 00:11, 7 August 2008 (UTC)Monkeynator03[reply]

No, that's not true. (Well, I suppose you could argue that it's vacuously true because you can't go faster than light in the first place.) I've never heard that one before. I have heard people say that something going very fast (but slower than light) will become a black hole, which is also not true. -- BenRG (talk) 01:18, 7 August 2008 (UTC)[reply]
I'm going to echo vacuous truth in non-small font, because I think it's the most useful thing you're going to get out of this question. As the first half of the statement ("things that go faster than light [in vacuum]") is an empty set, the second half of the statement can be anything and, from a logical standpoint, the collective statement is true. "If I go faster than light, I create a black hole" is the logical equivalent of "If I go faster than light, Wikipedia tastes like candy".
On the other hand, when that "[in vacuum]" caveat I added is removed, you can make a meaningful statement: when a particle exceeds the speed of light in a given medium, Čerenkov radiation is produced. But not a black hole. — Lomn 02:17, 7 August 2008 (UTC)[reply]
That's a misconception. I just tried it. Wikipedia tastes like a LCD screen. See also "Why cant I type?" below. :) Franamax (talk) 08:00, 7 August 2008 (UTC)[reply]

I think the misconception may stem from the fact that relative mass increases the faster you go. People confuse relative mass with rest mass. You can travel at 99% the speed of light, and your relative mass will be immense, but your rest mass is the same as it was at rest. Creating a blackhole depends on immense rest mass coupled with immense densityScienceApe (talk) 03:34, 7 August 2008 (UTC)[reply]

TeleDeltos paper

Where can I get it? I need to plot. —Preceding unsigned comment added by 79.76.167.200 (talk) 00:15, 7 August 2008 (UTC)[reply]

Why cant I type?

Why cant I type after 8 pints. I mean, my brain knows what to do, but my fingers just dont obey me? Why is that? —Preceding unsigned comment added by 79.76.167.200 (talk) 01:20, 7 August 2008 (UTC)[reply]

Alcohol is a central nervous system depressant, so although your brain sends the signals, they don't quite make it there in good shape. Q T C 01:34, 7 August 2008 (UTC)[reply]
20 20 24 hours a day, I want to be -- typing???
Duuud! I woz sooooo wasted I couldn't even -- type???
O man! She sooooo wanted it but I had to -- type???
Youth is wasted on the wrong people! Saintrain (talk) 04:58, 7 August 2008 (UTC)[reply]
Checking my email is the first thing I do when I get in, drunk or otherwise, and I agree typing is a significant challenge. It's not just more typos, it's different typos as well... I don't generally think about it much, though - I'm drunk at the time, after all! (I do have to be careful about *what* I'm typing though!). --Tango (talk) 18:29, 7 August 2008 (UTC)[reply]
True. I find I type similar-sounding words rather than the word I want when I'm drunk or tired. Which is frankly bizarre. It's like my fingers are taking dictation from my mind, but are themselves drunk... 79.66.38.215 (talk) 23:05, 7 August 2008 (UTC)[reply]

You might also want to check out short-term effects of alcohol, specifically this bit. —CyclonenimT@lk? 11:22, 7 August 2008 (UTC)[reply]

If sufficient pitchers of beer are consumed, it may be perceived to be an insoluble problem to pour the next glass from a newly arrived pitcher, because they put the handle on the wrong side of the pitcher! Just saying. Edison (talk) 18:30, 7 August 2008 (UTC)[reply]

Helium lifting gas

What isotope of Helium provides the most lift? ScienceApe (talk) 03:35, 7 August 2008 (UTC)[reply]

The lightest one, naturally, i.e. Helium-3. The gain isn't much, though, and you'll have trouble getting it in useful quantities. Algebraist 03:40, 7 August 2008 (UTC)[reply]
(e/c) I assume you mean "the most lift if you put it in a balloon". Helium-3 would probably provide more lift than Helium-4 because it's lighter, but it's ridiculously rare. Paragon12321 (talk) 03:41, 7 August 2008 (UTC)[reply]
Unless we start gas mining the moon or Jupiter. ScienceApe (talk) 04:55, 7 August 2008 (UTC)[reply]
Nah - make some tritium and wait for it to decay .. Philip Trueman (talk) 11:29, 7 August 2008 (UTC)[reply]
If you wanted to fly an airfoil through a purely helium atmosphere, wouldn't the more massive isotope provide more lift? Nimur (talk) 19:30, 7 August 2008 (UTC)[reply]
No. Aerodynamic lift depends on air pressure, not density. So lift should be the same for all reasonably gases (various caveats not discussed to keep it simple). --Stephan Schulz (talk) 03:48, 8 August 2008 (UTC)[reply]
And wouldn't hydrogen be even better?, in a purely helium atmosphere there is no chance it would burn. Graeme Bartlett (talk) 02:11, 8 August 2008 (UTC)[reply]

Curl of F

If the curl of F is not zero, what does it represent besides the presence of friction? For example the curl of E is the time rate of change of B. So when the curl of F is not zero, what does the quantity represent? —Preceding unsigned comment added by Kissnmakeup (talkcontribs) 04:58, 7 August 2008 (UTC)[reply]

The circulation. Have you seen Curl (mathematics)#Interpreting the curl? 81.174.226.229 (talk) 08:33, 7 August 2008 (UTC)[reply]

I see that I need to be more specific. According to Keith Symon in his text, Mechanics, if the curl of F (and consequently the line integral about a closed path by Stokes' Theorem) is zero, then the force is conservative. If it is not zero, energy is not conserved. When energy is not conserved does the quantity of the curl of F indicate anything quantitatively about the lost energy? If you do a line integral around a closed path and you don't get zero, is the quantity that you get equal to the energy dissipated by, say, heat from friction? Does the number that you get have a physical meaning specificly with regard to energy not being conserved? (By the way for those of you who are interested, I chose this strange handle after reading some of the discussion pages for articles like entropy.)Kissnmakeup (talk) 11:32, 7 August 2008 (UTC)[reply]

No, it doesn't always represent friction. In the example you gave of the electric and magnetic fields, it obviously doesn't represent friction. It represents the EMF induced by the changing magnetic field. You say "energy is not conserved", but of course the total energy of the system is always conserved. For example, if the changing magnetic field produces 1 V of EMF, and I let one electron go once around the loop, it gets accelerated by 1 eV of energy. This energy is exactly the energy that came from the magnetic field. Other non-zero curls of forces similarly represent transfers of energy. 128.165.101.105 (talk) 14:15, 7 August 2008 (UTC)[reply]

Cats and their waste

Cats are generally good about disposing of their waste – they dig a hole in loose soil or sand, do their business in it, and then meticulously cover it up. Do big cats, like lions and tigers, do the same? --Bruce1eetalk 06:07, 7 August 2008 (UTC)[reply]

Instinctively domestic cats bury their waste to protect their trail from predators. This usually only applies to subordinate cats however, and sometimes a dominant cat will actually display it's waste as a sign that "this is my territory, keep the hell out". I would imagine that this behavior is the same in big cats, as "the various species of cat are amazingly similar in both structure and behavior." Of course, some cats use other methods, though I'm not sure how a lion would look doing that! 20I.170.20 (talk) 13:24, 7 August 2008 (UTC)[reply]
It might be of interest to note that Christian the Lion used a litterbox scrupulously when he lived in London. - Nunh-huh 20:34, 7 August 2008 (UTC)[reply]

Air brakes

I know that fast cars, especially race car and high end road cars, have spoilers to create massive downforce. Hundreds and hundreds of pounds.

Why do I not see more cars with Air brake (aircraft)? The Veyron has one [22] --mboverload@ 06:22, 7 August 2008 (UTC)[reply]

You do. That "something" must be rather large to have a significant braking effect at the low velocity of race cars (relative to jet planes, that is), so a parachute is used, to help stop dragsters at the end of their runs. For race cars that go around a curved track, there's no hurry to slow down, as they could just go around the track a few times and coast to a stop if they wanted to. StuRat (talk) 06:31, 7 August 2008 (UTC)[reply]
So the issue is that at even high road speeds air braking would have little effect? And any effect would be mitigated by the increase in weight of the air braking system? --mboverload@ 06:49, 7 August 2008 (UTC)[reply]
If you used the same sized mechanism as on a plane it would have little effect, yes. I still say that a parachute IS a type of air braking, however, just one designed to use a much larger surface area and yet still have a low weight. StuRat (talk) 14:49, 7 August 2008 (UTC)[reply]
Air brake only makes sence if you are fast 100km/h is not enough to give you the benefit you want. But in racing the technice is old and was established in formula 1 racing through the Mercedes 300 SLR in the 1950s. This modification was forbidden, partly due to the fact that you block the view for all cars behind. --12:46, 7 August 2008 (UTC)

Loud booms

I suppose this goes in here. Hello, I'm from Temecula, California, and I've been hearing successive loud booms coming from a west by south-western direction for over an hour and it's freaking me out a bit. From a scientific standpoint, what is causing the booms, and why are there so many of them over a period of about an hour and a half? Camp Pendleton is in that direction, though I think it's too far away for anything to be heard from here if something is going boom over there. And this started around 11:00 PM local time.-- 07:41, 7 August 2008 (UTC)[reply]

You might try reading about the Sonic boom, although I don't know why they would cause such a disturbance that late at night. Kissnmakeup (talk) 11:07, 7 August 2008 (UTC)[reply]
Speaking for my area, which is near a military test facility, we're subject to destructive testing. While the base normally tries to blow things up during normal work hours, some tests don't quite go as planned -- we've had unusually large (heard across multiple counties) explosions past 11 PM on occasion. Some similar event could be the cause. However, there's no possible way for us to ascertain the cause from a scientific standpoint, as there's not enough information available. — Lomn 13:11, 7 August 2008 (UTC)[reply]

Čerenkov radiation/speed of light/follow on from previous question

Someone said earlier that:

when a particle exceeds the speed of light in a given medium, Čerenkov radiation is produced

How is this possible? I was always taught, albeit I'm only an A-Level student, that things cannot go faster than the speed of light, especially if they actually have mass. Particles have mass, don't they? —CyclonenimT@lk? 11:27, 7 August 2008 (UTC)[reply]

Things cannot go faster than the speed of light in a vacuum. Čerenkov radiation is seen in media such as water. William Avery (talk) 11:51, 7 August 2008 (UTC)[reply]
Let me clarify my point of emphasis: there is c, the "speed of light in a vacuum". c cannot be exceeded, ever. However, much like the speed of sound varies in different materials, so does the speed of light. Light travels through water ("the speed of light in water", if you will) at some rate less than c, and particles can exceed that speed. Exceeding c in water remains impossible. — Lomn 13:08, 7 August 2008 (UTC)[reply]
Incidentally, the differing speed of light in different mediums is the cause of refraction, as given by Snell's law, and diffraction. --Bowlhover (talk) 19:36, 7 August 2008 (UTC)[reply]

Keeping Cool

How can your body keep itself cool when the outdoor temperature is 37C and there is 100% humidity?

Surely there is no way for the body to keep cooler than 37C and the temperature will therefore continue to rise, how come people do not die? And yet people live in such conditions, not necessarily with air conditioning.

I am going on holiday next week and it is very likely I shall face this problem (although I will have central air most of the time). 62.24.129.68 (talk) 11:56, 7 August 2008 (UTC)[reply]

Well, firstly people do die from hyperthermia. The best way to stay alive is simply drinking plenty of water (plenty!) and to use sunscreen to help keep sunburn away. Avoid drinking alcohol and caffeine during the day, these all play games with your homeostatic mechanisms. Oh, and don't do too much vigorous exercise in the heat if you're not well-trained to! —CyclonenimT@lk? 12:10, 7 August 2008 (UTC)[reply]
The last response didn't seem to address the core issue. Let me try to state it: "If the humidity is 100% and the temperature of everything in the person's environment is significantly hotter than normal human body temp, what mechanisms does the body have to cool itself ?" In such a case evaporative cooling shouldn't work, neither should conduction of heat into a cooler object, as no cooler object is available, and neither should convection of air around the person, as that requires cooler air to replace the hot air. Perhaps radiation can still work, if the person is somehow able to radiate heat more than the objects around them, provided this difference in thermal radiation can overcome the additional heat the body absorbs due to the other forms of heat transfer.
I'd say that the way people are able to survive is that they avoid such environments except for brief periods. In places where such conditions exist, they likely hide from them until they pass, such as in undergound dwellings which remain cooler or in an adobe abode which retain the coolness of night well into the heat of the day. However, if conditions remained like that, day and night, permanently, I'd say such a place would indeed be uninhabitable. StuRat (talk) 14:41, 7 August 2008 (UTC)[reply]
I don't think radiation is going to help - the rate at which you radiate heat depends on your colour (black radiates most), but the same colours which radiate a lot absorb a lot, so the affect is going to cancel out. Even if it doesn't cancel out exactly, it's still going to be far to low to help much. I think people would soon die in such conditions - you have to just get out. The human body can survive in very high temperatures (I'm not sure how high, but certainly much much more than body temperature) if the humidity is very low, but in high humidity, heat can easily be fatal. --Tango (talk) 18:36, 7 August 2008 (UTC)[reply]
I'd classify that as "not survivable": the heat index of 37C/100% humidity is 83C, far above what the article gives as the "extreme danger" point of 54C. --Carnildo (talk) 20:33, 7 August 2008 (UTC)[reply]

Wormholes and momentum

If wormholes exist, then wouldn't their existance allow one to violate the law of conservation of momentum, ie by placing the two wormhole mouths at a right angle to each other (if you go through one vertically, and then exit somewhere else horozontally, then momentum's not conserved because momentum is a vector). —Preceding unsigned comment added by 76.68.246.7 (talk) 13:09, 7 August 2008 (UTC)[reply]

Wow, where to begin? First of all, a real wormhole solution of general relativity is not just some magical portal where you go in one "mouth" and instantly pop out the other. There is no clearly defined "mouth", and every point of spacetime (even inside the wormhole) has a little patch of spacetime around it that looks locally flat. That's one version of the equivalence principle, and it's extremely important for general relativity (it means spacetime is actually a manifold, so it can't have any corners, edges, or abrupt changes of any kind). So it could be that if you go in one end of your wormhole, you have to fire your rockets and turn inside it in order to come out the other end. Conservation of momentum is certainly not violated.
Getting into a little more detail than perhaps you're looking for, momentum actually isn't conserved globally in GR. That statement actually makes no sense, because momentum vectors at different points in spacetime aren't even in the same vector space, so you can't compare them. Instead, if an object with a certain momentum moves through spacetime, its momentum vector undergoes parallel transport, and if the spacetime is curved, you can go around a loop and back to the point you started from, and have the vector be pointing a different direction. See holonomy. So even if you can go through your wormhole and back around to where you started and it seems like momentum isn't conserved, it's actually not a problem. Momentum is always conserved locally, that never changes. —Keenan Pepper 13:31, 7 August 2008 (UTC)[reply]
Thanks a lot! —Preceding unsigned comment added by 76.68.246.7 (talk) 20:54, 7 August 2008 (UTC)[reply]

Project Management

WHAT IS PROJECT MANAGEMENT ALL ABOUT? WHAT DOES PROJECT MANGEMENT REALLY ENTAILS?

REGARDS, J.J. HARRY —Preceding unsigned comment added by 66.178.116.10 (talk) 14:17, 7 August 2008 (UTC)[reply]

First, stop SCREAMING (typing in all caps means you are angry and screaming at everyone) and then try reading project management and come back if you still have questions. -- kainaw 14:19, 7 August 2008 (UTC)[reply]

Why would a pencil sharpener need microbial protection?

I saw a pencil sharpener advertised in an OfficePro flyer that said, "ACME PENCIL SHARPENER/ERASER COMBO \ With Microban antimicrobial protection." Why would it need this? --205.174.162.243 (talk) 14:43, 7 August 2008 (UTC)[reply]

We have an article on Microban. Supposedly, it "inhibits the growth of odor and stain causing bacteria", according to the packaging I've seen. I think it's kind of a pointless exercise unless you really dislike the smell of your pencils and/or don't wash your hands regularly. —CyclonenimT@lk? 15:00, 7 August 2008 (UTC)[reply]
Control of bacteria is sometimes critical in health care settings with vulnerable patients such as nursing homes. An object that is used by a large group of people is a good way to transmit germs (door handles are the usual suspect for this problem). The ironic twist to this, of course, is that most medical records must be permanent and indelible and are therefore not written in pencil in the first place.
It's also possibly an advertising gimmick, since many pencil sharpeners are used in schools and defending children from nasty bacteria, well... anything for the children. Too bad it increases the risk of things like MRSA. SDY (talk) 15:20, 7 August 2008 (UTC)[reply]
What I worry more about, is what do all those "anti-microbial" chemicals do to the environment, if they actually work, and if they don't, it's just snake oil. I can't picture it actually doing any good to have your pencil sharpener treated with some chemical when snot-nosed kids are lined up using it one after another. (I don't mean that in a derogatory way, just realisticly.) On the other hand, how would any of us build up any anti-bodies to harmful stuff if we OCD sanitize everything like Mr. Monk? I think it's wise to be prudent either way. Balance, moderation, and just plain old common sense are in order.Kissnmakeup (talk) 16:17, 7 August 2008 (UTC)[reply]
Is it for the benefit of people that suck on the end of their pencil while they think? I think the most likely answer, however, is that it's just a gimmick. I wouldn't buy one. --Tango (talk) 22:47, 7 August 2008 (UTC)[reply]

111-year-old reptile

Today, I read a news story about a 111-year-old reptile (lizard) that mated for the first time in decades. (See [23].) Also, I have previously read about turtles being 200+ years old. (Question A:) How in the world do "they" (scientists) know the age of such creatures? These scientists do not say "approximately" 111-years-old. They give an exact number, as if the birth were actually recorded somewhere. That was also the case on the oldest documented turtle ... it was like 225 years old or something like that. (Question B:) Is there some article in Wikipedia or on the Internet that compares average life spans of various animals (example: gnat = life span of 3 hours; turtle = life span of 125 years, etc.)? (Question C:) In terms of evolution, can anyone see the reason / purpose that some animals live for such extended periods (100+ years)? Thank you. (Joseph A. Spadaro (talk) 15:59, 7 August 2008 (UTC))[reply]

(B) Maximum life span or one of its external links [24], which has a bit more extensive table. Oops, these are max. values, not averages. Clarityfiend (talk) 16:12, 7 August 2008 (UTC)[reply]
Getting down to (C) while avoiding notions of "reasons" for evolution, it's quite clear that an extended lifespan gives a creature more opportunity to produce offspring. There will be more turtles with potentially-200-year-old parents than those with potentially-80-year-old parents, all other things being equal. — Lomn 16:36, 7 August 2008 (UTC)[reply]
To the question (C): in general, yes, longer lifespan may give an animal an opportunity to produce more offsprings; and/or may allow an animal to afford slower metabolism; and/or provide for better survival rates for long spells of unfavorable conditions; and/or allow an animal to rear its young longer so that the offspring survival rate is high; and/or allow for better selection of the fittest individuals before they have a chance to reproduce. Obviously, only some of these apply to any given long-lived organism. Shorter life-span allows for faster metabolism, higher adaptability for unexpected changes in the environment, higher rate of population increase under favorable conditions, etc. Hope this helps. --Dr Dima (talk) 18:08, 7 August 2008 (UTC)[reply]
Thanks. I guess with regard to Question C ... what I was asking was ... why would the period be so long as to be 100+ years? Certainly, these animals can reproduce / survive harsh conditions / etc. in shorter time frames that that? These very long time spans (100+ years) seem excessive, no? (Joseph A. Spadaro (talk) 19:43, 7 August 2008 (UTC))[reply]
Geez. I hope that when I hit 100+ years, my lizard still wants to get up and go for a walk. --Drop Dead Ed (talk) 19:57, 7 August 2008 (UTC)[reply]
Hi. How is it possible that a lizard could mate over 100 years? Was this a tuatara by any chance, because it did say it was endangered, and related to dinosaurs (sphenodontians, to be exact)? Do they have a slow reproductive rate or something? Thanks. ~AH1(TCU) 20:08, 7 August 2008 (UTC)[reply]
Humans are actually fairly unusual in having a menopause, and even that only affects women. Assuming that the lizards and tortoises don't invest a lot of time and energy in raising young, what would be the advantage in them living that long and not being able to reproduce? 79.66.38.215 (talk) 22:56, 7 August 2008 (UTC)[reply]
Again, the "why" is problematic. Evolution isn't concerned with motivation. For this case, though, so long as the critter isn't going sterile, its lifespan is tied to a competitive edge in offspring quantity, and thus those genes make up a larger portion of the gene pool. — Lomn 20:45, 7 August 2008 (UTC)[reply]
Living longer is obviously better, all else being equal, since it gives more time for reproducing. The real question is: Why do other animals not live as long? Take a look at Senescence for some possible reasons. --Tango (talk) 22:56, 7 August 2008 (UTC)[reply]
As for part A, the exact ages of many animals are known because they are pets handed down from one generation to the next and the date of birth (or hatching) was noted. There may be older creatures of that species in nature, but maybe not, as they aren't as likely to be taken care of in their old age and live as long. StuRat (talk) 01:18, 8 August 2008 (UTC)[reply]
There's a joke about a fossil being 152 million and 43 years old. Why? Because the museum had been told it was 152 million years old 43 years ago. Maybe the keeper of the lizard was told that the lizard was 50 years old 61 years ago. --Kjoonlee 02:14, 8 August 2008 (UTC)[reply]

Tu'i Malila, Adwaita and Harriet might be helpful for information on old turtles/tortoises. --Kjoonlee 02:20, 8 August 2008 (UTC)[reply]

(C)They're in for the long haul so the tortoise wins the race. Julia Rossi (talk) 03:23, 8 August 2008 (UTC)[reply]

Why are their bellies cut open?

 ?? —Preceding unsigned comment added by ScienceApe (talkcontribs) 19:34, 7 August 2008

See exploding whale. --Carnildo (talk) 20:49, 7 August 2008 (UTC)[reply]
Well, as the picture is about whaling, that's done because that's what you do with things you're killing for food. Gutting is a pretty necessary process if you're going to eat an animal or prepare it for eventual eating; when you take out the bowels and internal organs, you also take out a lot of the bacteria that'll make the fish (or any animal, really) spoil more quickly. Most of that stuff is inedible anyway. Also, as Carnildo points out, not cutting in a couple of holes for ventilation can also result in the build-up of gases, but I think at that point the animal is going to be pretty much unfit for consumption anyway. -- Captain Disdain (talk) 20:55, 7 August 2008 (UTC)[reply]
Slight correction: They're not fish. —Pengo 22:43, 7 August 2008 (UTC)[reply]
No, I know. I was trying to say that just like fish have to be gutted, so do whales, but that particular point didn't really come through here. -- Captain Disdain (talk) 23:29, 7 August 2008 (UTC)[reply]

Well, there's a Chinese saying:

We eat everything with four legs, except for tables and chairs. We eat everything with wings, except for airplanes and helicopters. We eat everything with fins, except for submarines.

And there's also a Korean saying:

There's nothing to waste from a bovine.

If you try hard enough, almost anything is edible. Whale intestines are consumed for food in Korea. --Kjoonlee 01:53, 8 August 2008 (UTC)[reply]

Vectors

When dealing with vectors (ie displacement, velocity, acceleration, etc.), is it always needed to write the arrow above it? Now I know probably few people do always bother; my question is more asking if there are certain times (ie when dealing with vector components) when writing the arrow would be improper. —Preceding unsigned comment added by 76.68.246.7 (talk) 21:02, 7 August 2008 (UTC)[reply]

As long as you use consistent notation, then technically you can write it however you want. However, marking vectors (with a small arrow, or a boldface font, or a squiggle, or whatever you want to use) can be helpful especially when they're used in expressions with scalar quantities to make sure that you only perform vector operations on vectors and scalar operations on scalars. For example, writing v = xi + yj + zk is much clearer about what's going on than v = xi + yj + zk. Confusing Manifestation(Say hi!) 22:37, 7 August 2008 (UTC)[reply]
Personally, I underline vectors, but whatever notation you use, it's important to use it consistently. You could not mark vectors at all, but if you're going to mark some, you need to mark all, otherwise it will certainly cause confusion. If you don't mark any, people can probably work out from context when something is a vector and when it's not, but it's a waste of effort when you could just have marked them in the first place, and there is a risk of confusion. Also, marking them allows you to use x and x as two different variables - x often means position as a vector and x is the x-coordinate, so x=(x,y,z). If you didn't mark them, that would be extremely confusing. --Tango (talk) 22:52, 7 August 2008 (UTC)[reply]

August 8

Color of Water

What color is it? Some one said on TV tonight it was slightly blue. Is that correct? —Preceding unsigned comment added by 79.76.225.183 (talk) 00:45, 8 August 2008 (UTC)[reply]

Yes. See our cunningly-named article color of water for more information. Algebraist 00:47, 8 August 2008 (UTC)[reply]
Yes I ve just seen it. No more replies needed thank you! —Preceding unsigned comment added by 79.76.225.183 (talk) 00:49, 8 August 2008 (UTC)[reply]

Monitor colors keep changing

The colors on my CRT monitor keep changing very slightly (over a periods of a few seconds), giving pink, pale green or pale blue shades. What could be causing this please? Is it the monitor of the video card? —Preceding unsigned comment added by 79.76.225.183 (talk) 00:54, 8 August 2008 (UTC)[reply]

It kind of sounds like there is a strong but distant magnetic field near your monitor that is turning. When did this start happening? Also, colors changing on a CRT monitor usually means that it is going bad/about to die. --mboverload@ 00:57, 8 August 2008 (UTC)[reply]

Anywhere on earth?

Is there any where on earth where your urine can freeze before it hits the ground (assuming your are standing up when releasing it) —Preceding unsigned comment added by 79.76.225.183 (talk) 01:03, 8 August 2008 (UTC)[reply]

No. That's a silly myth / hyperbole. The density and thermal conductivity of air is so low that even if its temperature was absolute zero it wouldn't be able to freeze your urine in the ~1 second it takes to reach the ground. Dragons flight (talk) 01:07, 8 August 2008 (UTC)[reply]
How about if I wizz off a mountain ? :-) StuRat (talk) 01:11, 8 August 2008 (UTC)[reply]
Trust StuRat to be always thinking outside the box. I often thought he was out of his box anyway ;) —Preceding unsigned comment added by 79.76.225.183 (talk) 01:16, 8 August 2008 (UTC)[reply]

The thermal conductivity of urine should be considered. If a man is relieving himself off the edge of a VERY high cliff, as the urine descends a tube of frozen urine would form around a liquid core. After a certain amount of fall, the urine would become a solid rod. The rod would continue to fall, with a tube above it. The walls of the tube would become thicker in the downward direction until a sold rod is formed. This assumes that the fall is large enough and the temperature is low enough. AndMe2. 66.52.8.251 (talk) 04:55, 8 August 2008 (UTC)[reply]

Paradox?

A train is traveling at 100mph in a westerly direction. A small fly is traveling at 20mph (wrt to the ground) in an easterly direction along the rail track. At the moment of impact, the fly's velocity is instantaneously changed from +20 to -100 mph. At the precise moment of change of velocity, the fly must be at rest wrt ground. But since fly is in contact with train, train must also be instantaneously at rest. So, does the fly stop the train? —Preceding unsigned comment added by 79.76.225.183 (talk) 01:46, 8 August 2008 (UTC)[reply]

One second before contact, at that precise moment, must the fly be at rest with regard to the ground? --Kjoonlee 01:56, 8 August 2008 (UTC)[reply]
The error is in assuming the change is instantaneous and that the fly and the train each have a single velocity. The change will be gradual due to deformation of the fly and the train. That deformation resolves the apparent paradox - parts of the fly and parts of the train will be travelling at varying velocities. Each part of the fly will, at some point, be stationary, which I guess means some tiny amount of the train will also be stationary for a time, but that's perfectly possible if you allow for deformation of the train. --Tango (talk) 02:05, 8 August 2008 (UTC)[reply]
The problem with this 'paradox' is that it can't decide how physically realistic it's being. If it's being realistic, then the train and fly are not particles, they undergo deformation, and the change is not instantaneous, as Tango points out. Thus there is no paradox in this case. If, on the other hand, we're being very unphysical, and model the train and fly as particles, unable to deform, then the change of velocity is instantaneous, and it makes no sense to ask what the velocity of the fly is at that moment; it simply isn't well-defined in this crude model. Again, there is no paradox. The paradox arises if we unphysically model the fly and train as particles and then impose the physical condition that the fly, in changing velocity, must go through the velocities in between. Algebraist 03:05, 8 August 2008 (UTC)[reply]
Each atom of the fly is at some point stationary because its velocity has to change sign. No part of the train ever needs to come to rest. Someguy1221 (talk) 04:17, 8 August 2008 (UTC)[reply]

The statement of the problem speaks of "the moment of impact". A moment, as the word is used here, means a point in time of zero duration. So yes, both the train and the fly are stationary for that zero duration. AndMe2. 66.52.8.251 (talk) 04:29, 8 August 2008 (UTC)[reply]

Also in a real world situation the train is pushing air out of its way, and this air will greatly slow the fly to a stop and declerate it almost very close to the trains speed well before the fly hits the train. The deformation would amount to a microscope region of metal or glass being tensioned. No part of the train would necessairly need to come to rest, if it was inside a vacum this would happen and the impact would be more severe.--Dacium (talk) 04:43, 8 August 2008 (UTC)[reply]
At the moment of change in velocity, the velocity wrt the ground is not zero. Instead, it cannot be calculated because both the distance travelled and the time are zero and 0/0 is indeterminate. --Bowlhover (talk) 04:50, 8 August 2008 (UTC)[reply]

Bowlhover states (and I paraphrase) "At the moment of change in velocity, the velocity of the train is zero." Doesn't that mean the train is stationary? Also, how can there be "change" in a zero time duration? AndMe2 66.52.8.251 (talk) 05:14, 8 August 2008 (UTC)[reply]

Thermocouples

The flame failure devices in gas fires are usually called thermocouples, but how can a thermocouple act to shut off a gas valve in the absence of an amplifier? Are they, in fact, some other sort of heat activated mechanical device by any chance? —Preceding unsigned comment added by 79.76.225.183 (talk) 01:58, 8 August 2008 (UTC)[reply]

Why does quantum mechanics only work on a small scale?

And what is the cutoff point? One atom = Spoooky! Two atoms? Hello, Newton!Hey, I'm Just Curious (talk) 04:13, 8 August 2008 (UTC)[reply]

Quantum mechanics applies to all objects, but for most purposes, Newton's laws are sufficiently accurate. To build a rocket, for example, it would not be necessary to calculate the wave functions of every atom in every component. Not only is that level of accuracy not required, but parts cannot be engineered with such precision. --Bowlhover (talk) 05:02, 8 August 2008 (UTC)[reply]
See also: correspondence principle. Quantum mechanics would not have been accepted as a sound theory if it conflicted with the results of classical mechanics. -- 98.26.182.245 (talk) 05:38, 8 August 2008 (UTC)[reply]

cooling/heating vortex

Does anyone know the name of the vortex shaped device where you put in compressed air and cold air comes out one side and hot air out the other side and it has no moving parts or power. And can they work on water?--Dacium (talk) 04:38, 8 August 2008 (UTC)[reply]

AFAIK water cannot be compressed substantially, so if device works on expansion of compressed air, then it wouldnt work with water. —Preceding unsigned comment added by 79.76.221.121 (talk) 04:41, 8 August 2008 (UTC)[reply]

Do any types of dementia result in loss of tact

I have noticed thatin many people as they age they loose the social limits most people put on their communications. This results in older people being much more likely to make insulting statements that 1) they don't seem to notice they have made and 2) would never have made in their earlier years. I wondered whether this is a form of dementia or loss of societal imposed constraints. —Preceding unsigned comment added by 202.92.75.130 (talk) 05:11, 8 August 2008 (UTC)[reply]